You are on page 1of 209

MRCP PACES by Jansen Koh

(http://jansenkoh.com/MRCPnotes.htm)
CARDIO
1. Mitral regurgitation 2
2. Mitral valve prolapse 7
3. Mitral stenosis 8
4. Aortic regurgitation 10
5. Aortic stenosis 12
6. Mixed mitral valve disease 14
7. Mixed aortic valve disease 15
8. Prosthetic heart valves 16
9. VSD 18
10. ASD 20
11. HOCM 22
12. Approach to central cyanosis and clubbing 24
13. Dextrocardia 25
RESPI
14. Bronchiectasis 26
15. Interstitial lung disease 30
16. COPD 33
17. Pleural effusion 35
18. Collapse 38
19. Consolidation 40
20. Lobectomy/Pneumonectomy 44
21. Approach to lateral thoracotomy scar 45
22. Lung transplant 46
ABDO
23. Chronic liver disease 48
24. Hepatomegaly 54
25. Splenomegaly 56
26. Hepatosplenomegaly 58
27. Ascites 60
28. Unilateral enlarged kidney 63
29. Bilateral enlarged kidney 64
30. Transplanted kidney 67
NEURO
31. Approach to examination of the face 70
32. Cranial nerve syndromes 71
33. Isolated III nerve palsy 74
34. Isolated VI nerve palsy 76
35. VII nerve palsy 79
36. Myasthenia gravis 82
37. Approach to examination of the eyes 85
38. Gaze palsies 86
39. Unilateral ptosis 87
40. Bilateral ptosis 89
III, IV and VI cranial nerve palsies (33, 34, 35)
41. Assessment of higher cortical function 90
UPPER LIMBS (NEURO)
42. Upper limbs overview 92
43. Radial nerve palsy 94
44. Median nerve palsy 96
45. Ulnar nerve palsy 99
46. Wasted hands 101
Peripheral neuropathy (54)
47. Syringomyelia 104
48. Dystrophica myotonica 106
49. Cerebellar signs 109
50. Chorea 114
Parkinsonism (61)

LOWER LIMBS (NEURO)


51. Lower limbs overview 116
52. Flaccid paraparesis 118
53. Spastic paraparesis 123
54. Peripheral neuropathy 129
55. Charcots Joint 131
56. Proximal myopathy 133
57. Brown-sequard syndrome 135
58. Footdrop 136
59. Hemiparesis/Hemiplegia 137
60. Gait assessment 140
61. Parkinsonism 141
RHEUMATO
62. Rheumatoid arthritis 144
63. Gouty hands 146
64. Psoriatic hands 149
65. OA of the hands 151
66. Scleroderma 153
67. Ankylosing Spondylitis 155
68. Marfan syndrome 157
69. Dupytrens contractures 160
70. Clubbed fingers 160
71. Painful/swollen knee joint 161
72. Stills disease/Juvenile chronic arthritis 161
73. Enteropathic arthropathy 161
74. Old rickets 162
ENDOCRINE
75. Acromegaly 163
76. Cushings syndrome 165
77. Goitre 168
78. Pagets disease 174
79. Panhypopituitarism (Simmonds disease) 176
80. Addisons disease 177
81. Gynaecomastia 177
DERM
82. Dermatology overview 178
83. Psoriasis 181
84. Lichen planus 183
85. Neurofibromatosis 185
86. Purpura 187
87. Dermatomyositis 189
EYE
88. Diabetic retinopathy 191
89. Hypertensive retinopathy 195
90. Optic atrophy 197
91. Papilloedema 199
92. Central and branch retinal vein occlusion 201
93. Central retinal artery occlusion 203
94. Retinitis pigmentosa 204
95. Visual field defects 205
96. Visual acuity 207
97. Cataracts 207
98. Nystagmus 207
99. Pupillary defects 208
OTHERS
100. History-taking station 209

CARDIO!
1. Mitral Regurgitation
Presentation
Sir, this gentleman has mitral regurgitation that is moderately severe in nature.
There is a pansystolic murmur heard best at the apex which radiates to the axilla. (If it radiates to the carotids posterior mitral
leaflet rupture) This is a grade 3/6 murmur and is not associated with a systolic thrill. The first heart sound is soft and there is
presence of a third heart sound(S3). I did not detect any mid-diastolic murmur. The apex is thrusting and displaced, located at the
th
6 IC at the anterior axillary line.
This is complicated by pulmonary hypertension as evidenced by a palpable and loud pulmonary component of the second heart
sound associated with a left parasternal heave. There are no clinical signs of heart failure.
On the peripheral examination, patient is in atrial fibrillation with an irregularly irregular pulse which is rate controlled at 80 beats per
min. There is no bruising to suggest overanticoagulation. There are also no stigmata of infective endocarditis.
To complete the examination, I would like to take the patients blood pressure, as well as temperature chart for any fever. (Mention
abdominal examination, urine dipstick and fundoscopy if clinically suggestive of IE)
In summary, this gentleman has mitral regurgitation that is moderately severe in nature, with complication atrial fibrillation and
pulmonary hypertension. There are no complications of heart failure or infective endocarditis. My differential diagnoses include IHD,
MVP and Rh heart disease. (If thoracotomy scar, think of mitral valvotomy for MS)
Questions
How do you grade the severity of mitral regurgitation clinically?
Mild No Pulm hypt
Moderate Pulmonary hypertension
Severe LVF, S3
What are the causes of mitral regurgitation?
Common causes are MVP, IHD, Rh heart disease and endocarditis
Left ventricular dilatation, cardiomyopathy, Marfans, Rheumatoid, AS
Acute causes: MI, IE, Trauma, Surgery, spontaneous rupture
Anterior leaflet: radiates to axilla and back
Posterior leaflet: radiates to carotids
Mitral valvotomy if a thoracotomy scar seen
If elderly and mild to moderate, typically due to annular calcification
What are the differential diagnoses for a pansystolic murmur?
MR
TR
VSD
What congenital conditions can be associated with MR?
Corrected TGA
Partial AV canal
Ostium primum atrial defect (cleft mitral valve)
What causes a third heart sound?
Rapid filling of the left ventricle from the large volume of blood from the left atrium occurring in early diastole
Why is the pulse jerky?
Pulse is sharp and abbreviated due to lack of sustained forward stroke volume with a reduced systolic ejection time because of
regurgitant leak into the left atrium
How do you differentiate an MDM from severe MR vs MS?
MS has an opening snap
Severe MR associated with S3
MS murmur is longer
MS has loud S1
How do you differentiate between MR and TR murmur?
Mitral Regurgitation
Triscupid Regurgitation
PSM heard best at Apex
PSM heard best at the LLSE
Radiates towards the axilla
Radiates towards the right of sternum
Louder on expiration
Louder on inspiration
Displaced apex beat
Apex beat not displaced
Jerky pulse character
Normal pulse character
Normal JVP unless complication
Giant V wave with pulsatile liver

How do you differentiate between an MR murmur and that of a VSD?


MR
VSD
Loudest at the apex
Loudest at the LLSE
High pitched
Harsh, low pitched
Soft S1
Normal S1
What are the types of dynamic manoeuvres that you are aware of and what are their uses?
Respiration
Murmurs on the right side louder on inspiration due to increased venous return and blood flow to the right side of heart
Converse is true
Valsalva manoeuvre (decrease preload)
3 phases
Phase 1 beginning of maneuver
Rise in intrathoracic pressure and a transient increase in LV output
Phase 2 Straining phase
Systemic return falls
Reduced filling of the right and left heart chambers
SV and BP drops while HR increases
Most murmurs become softer and shorter except
MVP Systolic click and murmur begins earlier (LV size is smaller), ie longer and louder
HOCM murmur is louder as LV volume is reduced
Phase 3
Release of maneuvre
Right heart murmurs becomes louder followed by left heart murmurs
Squatting (increases venous return and systemic arterial resistance)
Most murmurs are louder
MVP click occurs later and murmur is shorter because LV size increased
HOCM LV size increased which reduced the obstruction to outflow and systolic murmur is softer
Standing
Most murmurs are softer except
MVP louder and longer and HOCM - louder
Isometric exercises (increases afterload)
AS Softer murmur as there is reduction of pressure gradient across the valve
MVP click occurs later and murmur is shorter because LV size increased
HOCM LV size increased which reduced the obstruction to outflow and systolic murmur is softer
MR/AR/VSD louder
Amyl Nitrite inhalation
Initial relative hypotension
MR/AR/VSD decrease
AS increases because of increased stroke volume
Later tachycardia phase
MS and right murmurs increase
Can use to differentiate Austin Flint from MS
What are the signs of severity for MR?
Presence of S3
Short MDM
Apex thrusting and displaced
Pulmonary hypertension
CCF
What is the pathophysiology of MR?
MR leads to LV overload
Compensatory LV dilatation
Eventually, decompensate resulting in heart failure and increased risk of sudden death
Also, regurgitation into the LA leads to enlargement of LA with AF and elevated pulmonary pressures
Should all murmurs be investigated?
All should be Ix except 1. mid-systolic, grade 2 or < murmurs with no associated findings or symptoms 2. continuous murmurs
of venous hum or mammary souffl of pregnancy

How would you investigate this patient?


ECG
LA enlargement P mitrale (II P >0.12s, Limb; Bifid P waves in limb leads with inter-peak > 0.04s, terminal P negativity
in V1)
LVH Sokolow & Lyon Criteria (S in V1 and R in V5 or 6 >35mm)
AF
Pulmonary hypertension
CXR
CCF pulmonary congestion, enlarged heart
Left atrial enlargement
Pulmonary artery enlargement
Echocardiogram
Dx of MR
Severity EF <60% and LV end-systolic diameter >45mm
Cause
Complications eg IE
Cardiac catheterisation
Not indicated in most patients but useful if there is discrepancy between echocardiographic and clinical findings
Useful to stenosis, regurgitation and intracardiac shunting
How would you manage this patient?
Education
Medical therapy
Antibiotic prophylaxis
Treatment of underlying cause eg IHD, dilated CMP (Rx of CCF and afterload reduction)
Treatment of complications eg AF, IE, CCF
Surgical
Indications
Symptomatic or
EF<60% or
LV end-systolic dimension >45mm
Types of surgery
Mitral valve repair if technical feasible is best
Mitral valve replacement if technically not feasible provided EF >30%
Controversial
Varied causes for MR
If due to IHD or dilated CMP, then Sx is controversial
If due to MVP, timing of surgery
Indicated if symptomatic, AF, pulmonary hypt, EF<60% or ESD>45
If asymptomatic, risk stratify according to regurgitant orifice(doppler)
<20mm2
20-39mm2
>40mm2 (this affects Px and closer follow up necessary)
How do you diagnose infective endocarditis?
Dukes criteria
2 Major, 1 Major 3 Minor or 5 minor
Major
Persistently positive blood c/s with typical organism
Persistently positive blood c/s
2 or more positive c/s > 12h apart
3 or more positive c/s each 1 hr apart
if 4 or more taken, >70% positive
Typical organism
Strep viridans, Strep bovis, enterococci, Staph aureus
HACEK: Haemophilus, Actinobacillus, Cardiobacterium, Eikenella, Kingella
Endocardial involvement
Positive echocardiogram: vegetations, abscesses, valve perforation, dehiscence
New valvular regurgitation
Minor
Predisposing heart condition
Fever
Vascular phenomena: arterial emboli, septic pulmonary emboli, mycotic aneurysm, ICH, Janeway lesion
Immunologic phenomena: GN, Oslers nodes, Roth spots, Rh factor
Positive blood c/s not satisfying major criteria
Positive echocardiogram not satisfying major criteria

What are the types of endocarditis?


Native valve endocarditis
Strep, enterococci, Staph
Rh, Cong HD, MVP with murmur and degenerative valvular disease
Prosthetic
Early(<60 days): Staph aureus or S. epidermis
Late(>60 days): Similar to native endocarditis
Fungal :IVDA and ICU
IVDA: TV involvement, AV also; Staph aureus, MRSA, fungi, Strep, GNB
How would you investigate?
Bloods
Blood C/S (as above)
FBC (NCNC anaemia, raised TW with left shift), ESR, CRP
2D echo
CXR, ECG
How do you treat infective endocarditis?
General measures
Eg oxygen, treat fever
Antibiotics
IV CP 12-18MU/d 4H for 4 weeks
Can also add IV gentamicin 1mg/kg 8H for first 2 weeks
If allergic, use vancomycin 30mg/kg/d in 2 divided doses for 4 weeks
HACEK organism: IV ceftriaxone
MSSA: IV cefazolin or nafcillin or cloxacillin
Surgery
Heart failure
Failure of medical therapy
Presence of fever and inflammatory syndrome after 1 week of appropriate and adequate antibiotics
Presence of mobile vegetation >10mm with 1 major embolism 1 week A/B
Presence of mobile vegetation >15mm with 1 week of A/B
Valvular complication eg valvular abscess, valvular obstruction, rupture into the pericardium, septal formation, fistula
Fungal endocarditis
Prosthetic valves esp if unstable or early(<60 days) or caused by S aureus

When and how should you prophylax against IE? (3 steps: Risk stratify, Type of Procedure and Type of antibiotics)
Risk stratify
Highest risk: prosthetic valves, both bioprosthetic and mechanical; previous IE; congenital cyanotic heart disease; or
surgically produced systemic/pulmonary shunts.
Moderate risk: (1) all other congenital cardiac conditions, except isolated secundum atrial septal defects and surgical
repairs of an atrial septal defect or patent ductus arteriosus or ventricular septal defect more than 6 months ago; (2)
acquired valvular dysfunction (eg, rheumatic heart disease, calcific aortic stenosis); or (3) hypertrophic cardiomyopathy
and mitral valve prolapse with valvular regurgitation and/or thickened leaflets. Thickening of the anterior leaflets of the
mitral valve correlates with significant mitral insufficiency, especially in men older than 45 years.
Low risk: mitral valve prolapse without significant regurgitation or thickened leaflets on echocardiography, implanted
cardiac PMs, implanted defibrillators, implanted coronary stents, or "innocent" murmurs. An important caveat is that in
elderly individuals, an innocent murmur may not be hemodynamically significant but may signify the presence of a calcified
leaflet that is susceptible to infection during a transient bacteremia.
Procedures that require antibiotic prophylaxis in high-to-moderate risk patients are as follows:
Invasive manipulation of the respiratory tract (eg, tonsillectomies, rigid bronchoscopy)
Gastrointestinal surgery, biliary tract surgery, sclerotherapy of esophageal varices, dilatation of esophageal
strictures, and endoscopic retrograde cholangiopancreatography in the presence of biliary obstruction
Prostate surgery, cystoscopy, and urethral dilatation
Generally, hysterectomies, vaginal delivery, cesarean delivery, urethral catheterizations, dilation and curettage,
therapeutic abortions, sterilization procedures, insertion or removal of intrauterine devices, cardiac catheterizations,
angioplasties, or endoscopies with or without biopsies do not require prophylaxis.
Adult antimicrobial IE preventive regimens for dental, oral, respiratory tract, or esophageal procedures recommended by the
American Heart Association to prevent streptococcal IE from oral-dental sources are as follows:
Administer amoxicillin at 2 g orally 1 hour before the procedure or ampicillin at 2 g IM or IV 30 minutes before the
procedure.
If the individual is allergic to penicillin, clindamycin at 600 mg, cephalexin at 2 g, or azithromycin at 500 mg orally 1
hour before the procedure are alternatives.
If the individual is allergic to penicillin and is unable to take oral medication, clindamycin at 600 mg IV or cefazolin at 1
g IM or IV should be given 30 minutes before the procedure.
Adult IE prophylactic regimens for individuals undergoing lower gastrointestinal tract surgery or instrumentation of genitourinary
tract procedures are for preventing enterococcal endocarditis. They are as follows:
High Risk regimens recommended by the American Heart Association are ampicillin at 2 g IM or IV plus gentamicin at
1.5 mg/kg (not to exceed 120 mg) within 30 minutes of the procedure, followed by ampicillin at 1 g IM, IV, or orally 6
hours later.
High-risk individuals who are allergic to penicillins should receive vancomycin at 1 g IV over 1-2 hours plus gentamicin
at 1.5 mg/kg IV or IM (not to exceed 120 mg) within 30 minutes of starting the procedure.
For moderate-risk patients, amoxicillin at 2 g orally 1 hour before the procedure or ampicillin at 2 g IM or IV within 30
minutes of starting the procedure is recommended.
The alternative for patients who are allergic to penicillin who are at moderate risk is vancomycin at 1 g IV over 1-2
hours, completed 30 minutes before the procedure.
What is the prognosis?
Depends on underlying cause
If IHD or dilated CMP, Px dependent on the underlying disease
If due to MVP
Asymptomatic regurgitation is a serious disease with a 5-yr death rate of 22-33% form cardiac adverse events

2. Mitral Valve Prolapse (Floppy MV, Barlows syndrome, Click-murmur syndrome)


Presentation
Sir, this patient has got a MR that is severe and secondary to a mitral valve prolapse.
I say this because there is a presence of a mid-systolic click associated with a late systolic crescendo-decrescendo murmur heard
best at the apex.
This murmur radiates towards the axilla and is a grade..(present as for MR)
I would like to complete my examination by asking the patient to perform the valsalva manoeuvre as well as to stand to accentuate
the murmur; take BP and temperature chart as well as a neurological examination for signs of stroke.
Questions
What causes a mid-systolic click?
o Inability of the papillary muscles or the chordae tendinea to tether the mitral valves in the late stages of systole
o The prolapsing of the valve leaflet into the LA and sudden tensing of the mitral valve apparatus causes the mid-systolic
click
What are the differential diagnoses for a systolic murmur?
o AS, PS, MVP, HOCM
What are the causes and associations of a MVP?
o Myxomatous degeneration of the mitral valve tissue
o Associated with
o Heart conditions
ASD (Secundum type)
Cardiomyopathy
Myocarditis
o Systemic conditions
Marfans syndrome
Ehlers danlos syndrome
Osteogenesis Imperfecta
Polycystic Kidney disease
SLE
What manoeuvres can accentuate the findings of an MVP and why?
o Valsalva manoeuvre and standing
o Decrease preload
o Reduction in the cardiac volume
o Further imparing the papillary muscles or chordae tendinea from maintaining tension on the leaflets and preventing the
leaflets from prolapsing into the LA
o Hence the systolic click occurs earlier with a longer duration of the systolic murmur
How do patients with MVP present?
o Asymptomatic
o Symptomatic
o Palpitations
o Anxiety
o Atypical chest pain
o Light-headedness
o Complications of MR
o CCF fatigue and dyspnea
o IE
o Arrhythmias
o Embolic phenomenon
o Sudden death
How would you Ix?
o Echocardiogram: Confirm Dx, Complications of MR
How would you manage?
o Education and reassurance
o Medical
o Antibiotic prophylaxis
Only if associated with MR
Otherwise not necessary if just MVP
o Symptomatic
Palpitations Rx with beta blockers (benign ventricular ectopy)
o Rx underlying cause or associations
o Rx complications such as MR with CCF, IE or AF or TIA
o Surgical
o As for MR

3. Mitral Stenosis
Presentation
Sir, this patient has mitral stenosis which is severe in nature with complications of infective endocarditis and congestive cardiac
failure.
There is presence of a mid diastolic murmur heard best at the apex and is accentuated in the left lateral position. It is a grade 3/6
murmur and is not associated with any diastolic thrill. It is severe as it is associated with an early opening snap and a long mid
diastolic murmur. The first heart sound is also loud. The apex beat is tapping in nature and is not displaced, located just medial to
th
the mid-clavicular line in the 5 intercostal space.
There is associated pulmonary hypertension with a palpable pulmonary component of the second heart sound with left parasternal
heave. There is a loud pulmonary component of the second heart with a presence of a functional TR as evidence of PSM at the
LLSE that is louder with inspiration associated with a giant V wave which is elevated at 5cm. There is no associated Graham Steel
murmur (PR).
This is associated with congestive cardiac failure with bilateral basal crepitations and bilateral pedal edema.
Examination of the peripheries reveals evidence of stigmata of infective endocarditis. Patient is clubbed with Janeway lesions on
the palms and Oslers nodes noted on the pulp of the fingers. There are also splinter haemorrhages with presence of conjunctival
pallor. There is presence of a peripherally inserted central catheter suggesting use of long term antibiotics. The patient is on IV
cloxacillin suggesting that the infective organism is MSSA. There is complication of atrial fibrillation. The HR is irregularly irregular
with a rate of 84 bpm. There is a characteristic pulsus parvus pulse. There are no bruises to suggest overanticoagulation.
I did not notice any mitral facies. The patients voice is also not hoarse which may suggest Ortners syndrome.
(mention the lateral thoracotomy scar with possible mitral valvotomy as intervention)
I would like to complete the examination by looking at the patient temperature chart as well as taking the patients BP.
In summary, this patient has MS that is severe in nature with complications of atrial fibrillation, pulmonary hypertension congestive
cardiac failure and infective endocarditis secondary to MSSA. The possible causes for MS include rheumatic heart disease or
congenital parachute valves.
Questions
How do you grade the severity of MS clinically?
Mild no PHT
Moderate PHT
Severe - CCF
What are your differentials for a mid-diastolic murmur?
MS
Atrial myxoma
Ball-valve thrombosis
Flow across the TV in ASD
MR with increased flow through the mitral valve during diastole
Austin Flint murmur (in severe AR)
What are the causes of mitral stenosis?
Common
Rheumatic heart disease
Congenital parachute valve
Rare
Calcification of mitral annulus and leaflets
CTDs: SLE, RA
Carcinoid (malignant)
What causes a tapping apex beat?
An accentuated first heart sound
What causes an opening snap?
Opening of a stenosed mitral valve and indicates that leaflets are pliable
Why is the first heart sound loud?
The mitral valve is held open during diastole by the transmitral gradient. The valve is suddenly slammed shut during ventricular
contraction
What causes presystolic accentuation of the murmur?
Occurs in sinus rhythm only during atrial systole which increases flow from the LA to the LV through the stenotic valve
How do patients present?
Asymptomatic
Women, may have h/o RH heart disease
Symptoms ppt especially during pregnancy or development of AF
Usually left-sided heart failure: exertional dyspnea, PND, orthopnea
Less frequently with right sided heart failure, hemoptysis and hoarseness of voice, although this symptoms are more specific

How do you diagnose rheumatic fever?


Duckett-Jones criteria
2 major or 1 major + 2 minor
Major
Carditis, Sydenhams chorea, SC nodules, erythema marginatum, arthritis
Minor
Past H/o RHD, fever, arthralgia, prolonged PR, ESR, CRP
+ a h/o streptococcal infection (ASOT raised, recent scarlet fever, +GpA
Strep throat c/s or kits for GpA Strep with high specificity but low sensitivity.
How do you manage Rheumatic fever?
Prevention
o
Primary prevention: Rx with IM Benzathine Pen G 1 dose or 10 days of Pen V
o
Secondary prevention: all patient with history of Rh fever should receive prophylaxis; IM Pen G once/month or PO Pen
V daily bd
What are the signs of severity for MS?
Early opening snap
Long MDM
Pulmonary hypertension
CCF
Pulsus parvus
What is Ortners syndrome?
Hoarseness of voice from compression of the recurrent laryngeal nerve from an enlarged left atrium
What is Lutembachers syndrome?
Association of MS with ASD
How would you investigate?
ECG
P mitrale
P pulmonale, RVH, RAD,
AF
CXR
Calcified mitral valve
Enlarged LA (double silhoutte sign, straightening of the left heart border, filling of the pulmonary bay by enlarged LA,
horizontalization of the left bronchus)
Prominent pulmonary trunk
Pulmonary congestion (Upper lobe diversion, Kerly B lines)
Echocardiogram
Dx
Assess severity (Valve area calculation and transmitral gradient)
Look for complications (eg IE)
Assessment suitability for balloon valvotomy
What is the normal cross-sectional area of the valve? 4-6 cm2
What is a significantly stenosed mitral valve? <1 cm2 and >10 mmHg gradient across the valve
How would you manage?
Education
Medical treatment
Antibiotic prophylaxis
Treat complications
AF anticoagulation and rate control
CCF symptomatic treatment vs improvement of mortality
Surgery
Indications
Symptomatic (limit activity) with significant stenosis (<1 cm2 and >10mmHg)
Pulmonary hypertension
Hemoptysis
Recurrent thromboembolic events despite adequate anticoagulation
Type
Valvotomy
Closed
Closed mitral valvotomy
Balloon valvuloplasty (procedure of choice) but need to satisfy
Good mobility of valves
Minimal calcification
No or mild MR
Minimal subvalvular disease
Open valvotomy
Mitral replacement
In which trimester does pregnancy results in symptomatic MS?
Second trimester due to increase in blood volume

4. Aortic Regurgitation
Examination
Proceed as per cardiovascular examination
On detecting AR, to examine eyes for Argyll-Robertson pupil and auscultate the femorals for pistols shots (Traube sign) and
Duroziez sign.
Presentation
Sir, this patient has aortic regurgitation that is severe.
My findings are:
Presence of a high-pitched early diastolic murmur heard best at the left lower sternal edge and is loudest at end expiration with the
patient sitting forwards. It is a grade 4/6 murmur and is associated with a diastolic thrill. It is severe as the murmur is of a long
duration associated a soft second heart sound with a S3 present. There is also an Austin Flint murmur with presence of a mid
diastolic murmur heard at the apex not associated with an opening snap.
th
The apex beat is displaced at the 6 IC anterior axillary line and is thrusting in nature.
This is associated with evidence of CHF with bibasal crepitations, raised JVP at 4 cm with a prominent V wave as well as bilateral
peal edema.
Peripheral examination showed no evidence of IE. The pulse is bounding and collapsing in nature at a rate of 90 bpm in SR. There
is no RR or RF delay to suggest coarctation of the aorta. In addition, quinkes sign was negative.
There was no conjunctival pallor but Corrigans sign and brachial dance were present. Mullers sign, Duroziez and Traubes signs
were not detected.
In terms of etiology, there is no evidence of symmetrical deforming polyarthropathy to suggest RA and patient does not have a
Marfanoid habitus or a high arched palate. There is no Argyll-Robertson pupil to suggest lewitic disease.
To complete my examination, I would like to take patients BP looking for wide pulse pressure and severe hypertension. I would
also want to look at the patients temperature chart.
In summary, this patient has got AR that is severe with complication of CCF. Possible causes for this patients AR are Rh heart
disease, infective endocarditis or congenital bicuspid valve.
Questions
What are the signs of severity of AR?
S3
Austin Flint murmur (functional mdm at the apex due to regurgitant jet striking the anterior leaflet of the MV, therefore
obstructing flow from the LA into the LV)
Soft S2
Duration of the decrescendo murmur and loudness of murmur (cf with AS)
Apex beat displaced and thrusting
CCF
Wide pulse pressure
Hills sign
What are the characteristic signs of AR?
Collapsing pulse
Brachial dance
Quinkes sign (visible capillary pulsation in the nail bed)
Corrigans sign (Visible Carotid pulsation in the neck)
De Mussets sign (head nodding in time with the heart beat
Mullers sign (pulsation of the uvula)
Traubes sign(pistol shots) and Duroziez sign(to and fro murmur on sl compression of the femoral artery)
What are the causes of a collapsing pulse?
AR
PDA
An aortopulmonary window
A ruptured aneurysm of the aortic sinus
Active Pagets
High fever
Severe Anaemia
Pregnancy
What would you expect to find on taking this patients blood pressure?
Wide pulse pressure
Severe hypertension (with functional AR)
UL and LL discrepancy with systolic in LL>UL = Hills sign
How do you differentiate an Austin Flint murmur from mitral stenosis?
Opening snap
Loud S1
Tapping apex beat, which is not displaced
10

What are the causes of AR?


Valvular
Rh, IE and congenital biscupid valve (associated with CoA)
Aortic root dilatation
Syphilis, RA, AS, Marfan, severe hypertension
Acute causes
IE, trauma, Aortic dissection, rupture of sinus of valsalva
How would you investigate?
ECG LVH with diastolic overload pattern deep but narrow Q, isoelectric ST, and tall T waves in left praecordial leads
CXR valvular calcification, cardiomegaly, pulmonary congestion, widened aorta
2D echo
Confirm Dx
Assess cause
Severity
Complications
How would you manage this patient?
Education
Medical
Antibiotic prophylaxis
Treat underlying cause
Treat complications such as CCF, IE
Vasodilators ACE and CCB
Surgical
Indications
Symptomatic CCF, angina and severe AR
LV ESD >55mm
Aortic root >55mm
Reduction of EF >5% on exercise
Types of surgery
What is the prognosis?
4% develop symptoms, CCF or both annually

11

5. Aortic Stenosis
Presentation
Sir, this patient has Aortic stenosis that is severe in nature.
My findings include:
Presence of an ejection systolic murmur heard best at the aortic area and radiates to the carotids. It is a grade 4/6 systolic murmur
a/w with a systolic thrill. It is severe as there is an early ejection click a/w a long systolic murmur with delayed peaking of the
murmur. I could not detect an S4 and the second heart sound is soft. There was also no paradoxical splitting of the second heart
sound.
th
The apex beat is heaving in nature and is displaced, located at the 6 IC space at the just lateral to the mid-clavicular line.
This is associated with signs of congestive cardiac failure as evidenced by presence of bibasal crepitations, raised JVP at 3 cm with
prominent V wave and bilateral pedal edema but she does not require supplemental oxygen.
Peripheral examination does not reveal any stigmata of IE. The pulse is regular at 84bpm and is anacrotic/pulsus parvus et tardus
in nature. There are no features suggestive of haemolytic anaemia with no conjunctival pallor and patient is not jaundice.
I would like to complete my examination by taking the patients blood pressure to look for a narrow pulse pressure as well as his
temperature chart. I would also like to enquire on patients symptoms of angina, syncope and dyspnea as these are important
prognostic markers.
In summary, this patient has got aortic stenosis that is severe in nature with complication of congestive cardiac failure. There is no
evidence of infective endocarditis or haemolytic anaemia. The most likely causes include Rh heart disease, calcified biscupid aortic
valve or degenerative calcified aortic valves.
Questions
What are the differential diagnoses for an ejection systolic murmur?
AS
PS
HOCM
MVP/MR
Coarctation
How do you differentiate between them?
AS and PS expiration and inspiration
AS and HOCM Valsalva, squatting
AS and MVP location and clicks
AS and Coarctation differential pulse
What are the types of pulses associated with aortic stenosis?
Pulsus parvus et tardus means low volume pulse with delayed upstroke due to a reduction in systolic pressure and a gradual
decline in diastolic pressure
Anacrotic pulse small volume pulse with a notch on the upstroke
What does a normal pulse volume in AS mean?
The travsvalvular gradient is <50 mmHg
What does a palpable systolic thrill implies?
It means that the transvalvular gradient is > 40mmHg
What does the second heart sound indicate about the aortic stenosis?
Soft second heart sound means poorly mobile and stenotic valve
Reversed splitting means mechanical or electrical prolongation of ventricular systole; S2 is normally created by the closure of the
aortic valve followed by the pulmonary valve, if the closure of the aortic valve is delayed enough, it may close after the pulmonary,
creating an abnormal paradoxical splitting of S2.
Single second heart sound implies fibrosis and fusion of the leaflets
Normal second heart sound implies insignificant stenosis
What is Gallavardin phenomenon?
Systolic murmur may radiate towards the apex, which may be confused with a MR murmur
How can haemolytic anaemia result from aortic stenosis? MAHA from severely calcified aortic valve
What are the causes of aortic stenosis?
Rheumatic heart disease (<60), Degenerative calcification (>75), Calcified biscupid (60-75, males)
What are the severity markers?
Early ejection click
Long Systolic murmur
Late peaking of the murmur
4th heart sound
Paradoxical splitting of S2
Heaving apex beat which is displaced
Systolic thrill

Pulsus parvus et tardus


Narrow pulse pressure
Symptoms (ASD)
Angina
Syncope
Dyspnea (Most impt)

Px
5 years
3 years
2

years

12

How do you differentiate AS from aortic sclerosis?


No severity signs as above
ESM which is localised to aortic area with a normal S2 in elderly person
How do patients present?
Asymptomatic and incidental finding
Angina
o
Increase oxygen requirement for hypertrophied LV with hypoperfusion of the subendocardial myocardium
Syncope
o
Cardiac arrythmias
o
Peripheral vasodilatation eg post exercise without concomitant increase in CO
o
Transient elctromechanical dissociation
Dyspnea
o
Implies LV dysfunction and heart failure
How would you investigate?
ECG LVH with strain, 1st degree heart block, LBBB
CXR Calcified aortic valve, cardiomegaly, pulmonary congestion
2D echo
o
Dx
o
Severity
LVH, EF

Severity
Mild
Moderate
Severe
Critical
Complications eg IE

Area
>1.5
1-1.5
<1
<0.7

Transvalvular gradient
<25
25-50
50-80
>80

How would you manage?


Education
Medical
o
Antibiotic prophylaxis
o
Rx complications such as arrythmias and CCF (caution with antihypt to avoid reducing preload)
o
Statins may have a role in reducing calcification of the aortic valve
Surgical treatment
o
Indications
Symptomatic and severe
Asymptomatic but has
Area<0.6
LV systolic dysfunction
Hypotension on exercise
VT
LVH>15mm
Moderate AS but going for Sx for CABG, MVR or aortic root surgery
o
Options
Valve replacement (Sx of choice)
Valvuloplasty (for moribund patients)
What are your thoughts on a young person with AS murmur but a normal aortic valve?
Supravalvular stenosis
o
Can be isolated or associated with Williams syndrome
o
It is an inherited disorder, autosomal dominant, Ch 7
o
Features of elfin facies, hypertension and mental retardation with other cardiac lesions such as PS
Subvalvular stenosis
What abdominal condition is associated with AS?
Angiodysplasia of the colon (PR bleed)
What is pulse pressure?
Difference between systolic and diastolic pressure
Normal 40mmHg
Wide - >60 mmHg
Narrow - <25mmHg
Note: There is no official definition but studies usually measures the pulse pressure as a continuum

13

6. Mixed Mitral Valve disease


Sir, this patient has mixed mitral valve disease and the predominant lesion is
1. Mitral stenosis
Early opening snap with long diastolic murmur
Loud S1
No S3
Tapping apex beat that is not displaced
Pulsus parvus pulse
2. Mitral Regurgitation
Displaced apex beat that is thrusting in nature
Soft S1
S3
Jerky pulse
Mdm is a short diastolic murmur with no opening snap
My findings are:
o Presence of a MDM heard best at the apex and accentuated at the left lateral position. It is a grade 4/6 murmur as it is
associated with a diastolic thrill. It is severe as it is associated with an early opening snap and a long mdm. There is a loud
first heart sound. There is also a MR murmur as evidenced by a PSM heard best at the apex. It is a grade 3/6 murmur and
is not associated with any systolic thrill. There is also no third heart sound.
o Apex beat is not displaced
o Pulmonary hypertension, CCF
o IE, AF, over-anticoagulation
o Mitral facies and Ortners
o No lateral thoracotomy scars to suggest previous operation for MS
o Requests
o In summary, this patient has mixed mitral valves disease with the predominant lesion being MS. This is complicated by
pulmonary hypertension and AF. He is not in heart failure and signs of IE. The most likely cause in this patient is
rheumatic heart disease.
o
o
o
o
o
o
o
o

Presence of a PSM heard best at the apex; 4/6 murmur and is associated with a systolic thrill. There is a soft first hearts
sound and a presence of a third heart sound. There is also a MS as evidenced by MDM heard best at the apex at the left
lateral position. It is associated with a late opening snap and short mdm.
Apex
CCF, Pulmonary hypt
IE, AF
Lateral thoracotomy scar
Marfans and SLE
Requests
In summary

Questions
What is the significance of a third heart sound in mixed mitral valve disease?
o Presence of S3 implies no significant MS
What are the causes?
o Rheumatic heart disease
o MS with valvotomy done that has been complicated by MR
What are the frequencies of valvular involvement in rheumatic heart disease?
o MV 80%
o AV 50%
o Mixed MV and AV 20%
o TV 10%
o PV -1%

14

7. Mixed Aortic Valve Disease


Sir, this patient has got mixed aortic valve disease and the predominant lesion is
Aortic Stenosis
1. small volume pulse
2. heaving and undisplaced apex beat
3. Loud and harsh systolic murmur
4. associated with systolic thrill
Aortic regurgitation
1. Collapsing pulse
2. Displaced and thrusting apex beat
3. Soft systolic murmur
4. No systolic thrill
My findings are:
o Presence of an ESM heard best at he aortic area that radiates towards the carotids. It is a grade 4/6/ murmur and it is
associated with a systolic thrill. It is severe as it is associated with an early ejection click with a long systolic murmur with
late peaking. There is no S4 detected and the second heart sound is soft; I could not detect a paradoxial splitting of the
second heart sound.
o There is also an EDM heard bset at the LLSE and is loudest in expiration with the patient sitting forwards. It is a grade 3/6
murmur and is not associated with any diastolic thrill. (skip the severity markers for AR)
o Apex
o CCF
o IE, SR and small volume, haemolytic anaemia
o Request BP especially for narrow pulse pressure, Temperature chart and enquire symptoms of angina, syncope and
dyspnea.
o In summary
o
o
o
o
o
o
o
o
o

Presence of an EDM heard best at the LLSE and is loudest in expiration with the patient sitting forwards. It is a grade 3/6
murmur as it is not asssociatd with ay diastolic thrill. The second heart sound is soft and there is no third hear sound.
There is also no mdm at the paex to suugest an Austin Flint murmur.
There is also an ESM heard best at the aortic area that radiates towards the carotids. It is a grade 2/6/ murmur and is not
associated with any systolic thrill. (skip the severity markers for AS)
Apex
CCF
IE, SR, collapsing, brachial dance and Corrigans
No quinke, mullers de Mussets, Duroziez and Traubes
No Marfans, AS or RA
Requests for BP especially for a wide pulse pressure, temperature chart.
In summary

Question
What are the causes of a mixed aortic valvular lesion?
o Rheumatic heart disease
o Biscupid aortic valve

15

8. Prosthetic Heart Valves


Sir, this patient has got mechanical mitral/aortic valve which has been done for an underlying mitral/aortic stenosis/regurgitation.
I say this because there presence of a mid-line sternotomy scar associated with audible metallic clicks to the unaided ear.
There is presence of a mitral valve replacement with a metallic first heart sound and a normal second heart sound. There is no pansystolic murmur to suggest a valve leakage.
(There is presence of an aortic valve replacement as evidenced by a normal first heart sound followed by a metallic click and a
metallic second heart sound. There is no early diastolic murmur or a collapsing pulse to suggest a valve leakage.)
(There are both mitral and aortic valve replacement as evidenced by dual metallic heart sounds. There is no pan-systolic murmur to
suggest a mitral valvular leakage or an early diastolic murmur which indicates an aortic valve leakage.)
Ther metallic sounds are crisps (no valvular thrombosis) and there is no conjunctival pallor or jaundice to suggest hemolytic
th
anaemia. The apex beat is displaced at the 6 IC at the ant axillary line. (Displaced and MVR = MR; undisplaced and MVR = MS;
Displaced and AVR = AR). There is no evidence of pulmonary hypt(MVR). Patient is in CCF as evidenced by presence of bibasal
crepitations, raised JVP of 3 cm and bipedal edema.
Patient is not in AF(MVR) and pulse is not collapsing in nature (mention this if AVR for leakage). There is no peripheral stigmata of
IE such as clubbing, Janeways lesion, Oslers nodes or splinter haemorrhages. This is associated with bruises which suggest
overanticoagulation.
There is no evidence of any Marfans, RA, AS or Syphilis (mention this if AVR for AR or MVR for MR)
I would like to complete my examination by taking the BP of the patient and looking at his temperature chart and neurological
examination for strokes.
In summary, this patient has got MVR/AVR or both which is most likely done for MR/MS/AR/AS (which is due to underlying
Marfans syndrome). There is no clinical evidence of valvular leakage, thrombosis or haemolytic anaemia. There is also no pulm
hypt but pt is in heart failure and in AF. There are no signs of IE or overanticoagulation.

16

Questions
What are the indications of a mitral/aortic valve replacement?
o See respective MS/MR/AS/AR
What are the types of prosthetic valves?
Mechanical valves
Ball and cage valve (Starr-Edwards)
Single tilting disc (Bjork-Shiley)
Double tilting disc (St Jude)
Bioprosthetic Homograft or heterograft
What are their differences?
Duration
Mechanical valves last 20-30yrs
Bioprosthetic may fail within 10-15 years
Thrombogenecity
Mechanical require lifelong anticoagulation (Starr-Edwards>single disc>double disc)
Bioprosthetic does not require lifelong anticoagulation
Therefore in the young and those who already require long term anticoagulation, mechanical valves preferred
And in the elderly(lifespan <10-15 years) or those that cannot tolerate anticoagulation, bioprosthetic valve preferred
What are the complications?
Complications of prosthesis
Valve leakage (mild- hemolytic anaemia, severe CHF)
Valve thrombosis
Valve strut failure (rare, acute presentation with high mortality, Bjork-Shiley)
Hemolytic anaemia (from valvular leakage due to partial dehiscence; Rx with Fe, folate, transfusions, B blockers or if fit for
op, repair of valve replacement)
Complications of valvular heart disease
Infective endocarditis
Congestive cardiac failure
Thromboembolism (rule out IE and thrombosis)
Complications of management
Overanticoagulation
Bleeding
What are the causes of anaemia in such patients?
Bleeding from anticoagulant
Hemolytic anaemia
Infective endocarditis
How do you tell clinically that the valve has malfunction?
New murmur
Change in characteristic of a preexisting murmur
Change in intensity or characteristic of an audible sound
How would you investigate a patient suspected of having valve dysfunction?
Cinefluoroscopy rapid, fast ad inexpensive for structural integrity
TTE often difficulty study due to reverberations from the metal
TEE useful for assessing MV prosthesis but limited in AV prosthesis
Can MRI be done for a patient with mechanical heart valves?
Yes it is safe except those with pre 6000 Starr-Edwards prosthesis (1960-64)
Valve thrombosis
Up to 5% per patient-year
Factors inadequate anticoagulation and mitral location
Manisfest as
pulmonary congestion, poor peripheral perfusion or systemic embolisation, acute deterioration
Change in audible sounds or murmur
Ix shows reduced movement of the disc or poppet, reduced orifice area, increased regurgitation or transvulvular pressure
Mx
<5mm IV heparin
>5mm Fibrinolysis (if high operative mortality) or valve replacement

17

9. VSD
(Clue: Young patient; look for associated conditions of Down and Fallots Tetrology)
Presentation
Sir, this patient has got a Ventricular Septal Defect that is hemodynamically significant as evidenced by:
Presence of a pan-systolic murmur heard best at the left lower sternal edge with radiation towards the right side of the sternum.
This murmur can be heard at the apex but there is no radiation to the axilla. It is louder on expiration. It is a grade of 5/6 murmur
and is associated with a systolic thrill. The first heart sound is not soft. I did not detect any third heart sound.
I did not detect any early diastolic murmur at the left lower sternal edge to suggest an associated AR. This is also no associated
mid-diastolic apical rumble at the apex to suggest a flow murmur at mitral valve which can be a/w VSD.
th

Apex beat is displaced and is located at the anterior axillary line at the 6 IC. It is thrusting in nature.
No evidence of Eisenmengers syndrome such as central cyanosis, clubbing. There is evidence of pulmonary hypertension such as
palpable or loud P2, no parasternal heave.
There are no signs of CCF such as bilateral pedal edema, no basal crepitations or raised JVP; she is comfortable at rest with a RR
of 14 bpm and does not require any supplemental oxygen.
There is no peripheral stigmata of IE. The pulse is regular at 70 bpm and character of the pulse is normal. There are no feat ures to
suggest Down syndrome or (Turner syndrome - if female).
I would like to complete my examination by looking at the patients temperature chart and taking his blood pressure.
In summary, this young man has got a VSD that is severe with a displaced apex beat and is complicated by pulmonary
hypertension. Clinically, there is no heart failure or Eisenmengers syndrome or IE. The most likely cause is congenital VSD.
Dy/Dx MR, TR, VSD For VSD, murmur radiates to the right of the sternum, young patient and a palpable thrill
Questions
What are your differential diagnoses for a PSM?
o MR PSM at apex radiates towards the axilla, soft S1
o TR PSM heard at the triscupid area, louder on inspiration; usually secondary to pulmonary hypertension or seen in
IVDAs; Giant V wave, pulsatile liver
o VSD PSM heard at the LLSE which is louder on expiration
What are the causes of a VSD?
o Congenital
o Acquired: MI
How common is VSD?
o The most common congenital heart condition
o 2 per 1000
o Usually in the membranous portion (can also be found in the muscular)
o Small defects close spontaneously in early childhood in about 50%
What are the types of VSD?
o Supracristal (above the crista supraventricularis)
o Infracristal
o Upper membranous
o Lower muscular (<5%)
o Different morphology
o Maladie de Roger
o Swiss cheese
o Large
o Gerbode defects (opens into the RA)
What are the conditions in which VSD is part of?
o Fallots tetralogy
o Truncus arteriosus
o AV canal defects
o DORV (double outlet RV)
What are the complications of VSD?
o AR
o Pulmonary Hypt
o Eisenmengers complex
o CCF
o IE
18

Does the loudness of the murmur correlate with severity?


o No; in fact, a small VSD results in a louud murmur and the converse is true.
What is Maladie de Roger?
o A term used to describe a small VSD that is hemodynamically insignificant with normal heart size, ECG and CXR; it is a
loud murmur on ausculatation
How do you differentiate an isolated VSD with one that is associated with Fallots tetralogy?
o Pulmonary thrill, PS murmur
o Clubbed and central cyanosis (but could be VSD with Eisenmengers)
How do you differentiate VSD from HOCM?
o ESM rather than PSM
o Apex is not displaced, double apical impulse
o Jerky pulse
How would you Ix?
o ECG
o Normal in small defects
o LVH, RVH, p mitrale
o Pulmonary hypertension P pulmonale, RAD
o CXR
o Normal in small defects
o Cardiomegaly, LA and LVH
o Pulmonary plethora initially
o Pulmonary hypertension later with prominent pulmonary trunks, rapid tapering of the peripheral pulmonary
arteries and oligaemic lung fields
o CCF
o Echocardiogram
o Diagnostic
o Determine severity and direction of shunt via color doppler
How would you manage?
o Counsel
o Medical
o Antibiotic prophylaxis
o Rx complications of CCF
o Surgical
o Small, asymptomatic and normal pulmonary pressure do not need surgery
o Indications
Evidence of pulmonary hypertension or CCF
Right ventricular pressure >50 mmHg
Right to left flow ratio or pulmonary to systemic resistance ratio >1.5
Recurrent IE
Cx by AR
Acquired cause eg rupture of septum form MI
o Contraindication
Development of Eisenmenger
o Types
Surgery
Percutaneous transcatheter

19

10. ASD
Presentation
Sir, this patient has atrial septal defect as evidenced by presence of a wide and fixed splitting of the second heart sound.
There is presence of an ejection systolic murmur over the pulmonary area which is louder on inspiration, implying presence of a
pulmonary systolic murmur. This is a grade 3/6 murmur and there is no associated systolic thrill.
There is no associated mid-diastolic flow murmur to suggest relative tricuspid stenosis or Lutembachers syndrome (Acquired MS
and ASD). There was also no associated PSM to suggest an ostium primum defect (TR, MR, VSD).
th

The apex beat is not displaced and is located in the 5 IC space just medial to the mid-clavicular line.
There is no complication of Eisenmengers syndrome; there is no evidence of pulmonary hypertension; is not clubbed and no
central cyanosis. There is also no evidence of congestive cardiac failure.
There are no stigmata of infective endocarditis. Patient is in atrial fibrillation with an irregularly irregular pulse and is rate controlled
at a rate of 84 bpm; there are also no bruises to suggest over-anticoagulation.
There is no evidence of any thumb defects to suggest Holt-Oram syndrome. The patient also does not features of Downs
syndrome.
I would like to complete my examination by examining patients chest for pneumonia as patients are prone to recurrent chest
infections as well as a neurological examination to look for evidence of stroke due to paradoxical embolus.
In summary, this patient has got an ASD with complications of AF. There are no complications of pulmonary hypertension, heart
failure or Eisenmengers syndrome. There is also no infective endocarditis. This patient has ASD is most likely due to an ostium
secundum atrial septal defect which is a congenital heart condition.
Questions
What are the types of ASDs?
o Ostium secundum type
o 90%
o common congenital heart condition
o Most remain asymptomatic
o If small <2 cm, normal life expectancy with no symptoms
o Larger defects may present in the second or third decades with dyspnea or fatigue
o defect in the fossa ovalis with no involvement of the AV valves
o Ostium primum type
o 10%
o Failure of fusion of the septum primum with the endocardial cushions
o AV valves affected MR, TR and VSD
o Sinus venosus type
o Defect in the septum just below the entrance of the SVC (inverted P waves in the inferior leads)
How do patients present?
o Secundum
o Asymptomtic
o Symptomatic
Fatigue, dyspnea
Right heart failure
AF
Recurrent pulmonary infections
Paradoxical emboli
o Primum
o In addition to the above
Failure to thrive, poor development
IE
Syncope (heart block)
What are the complications of ASD?
o Pulmonary hypertension, heart failure, Eisenmengers
o AF, IE (primum defects)
o Recurrent chest infection, paradoxical emboli
What are the various types of murmurs that can be associated with ASD and what do they mean?
o Pulmonary ejection systolic murmur and mid-diastolic murmur at the triscuspid area implies increased flow of blood
through the pulmonary and triscupid valve respectively due to left to right shunting of blood via the ASD
o MS murmur means acquired Rh heart disease affecting the mitral valve in Lutembachers syndrome
o MR, TR or VSD murmur implies that ASD is of the ostium primum type
What is the mechanism of a split second heart sound?
o A split S2 is caused physiologically during inspiration because the increase in venous return overloads the right ventricle
and delays the closure of the pulmonary valve
20

Why is there wide and fixed splitting of the second heart sound in ASD?
o With an atrial septal defect, the right ventricle can be thought of as continuously overloaded because of the left to right
shunt, producing a widely split S2, with the pulmonary valve closing much later cf to the aortic valve
o It is fixed because the atria are linked via the defect, inspiration produces no net pressure change between them, and has
no effect on the splitting of S2
How do you differentiate between a flow mumur through the pulmonary valve vs a PS murmur?
o PS murmur is a/w P2 that is soft, delayed and varies with respiration
What are the conditions that can cause a wide splitting of the second heart sound?
o Increase RV volume ASD, VSD, PR
o Increase RV pressure PS
o RV conduction delay RBBB
o Increase LV emptying MR, VSD
What is Eisenmengers syndrome?
o It implies a reversal of a left to right shunt as a result of the development of pulmonary hypertension
o This occurs in conditions such as ASD, VSD or PDA
o Patients are markedly clubbed and deeply cyanosed
o The defect must not be repaired once this complication occur due to high mortality risk
What is Lutembachers syndrome?
o Acquired Rh MS
o ASD
What is Fallots trilogy?
o ASD
o RVH
o PS
What is Holt-Oram syndrome?
o ASD secundum type
o Hypoplastic thumb with accessory phalanx
o Autosomal dominant
How would you investigate?
o ECG
o Secundum Partial RBBB, RAD
o Primum LBBB, LAD, low atrial rhythm
o Sinus venosus inverted P in inferior leads
o Pulmonary hypertension p pulmonale, RVH
o CXR
o Cardiomegaly
o Pulmonary hypertension
Prominent pulmonary trunk
Enlarged RA and RV
o Shunt vascularity/pulmonary plethora (well visualised pulmonary arteries in the periphery of the lung
o Small aortic knob
o Echocardiogram
o Diagnosis - demonstrate the defect
o Cardiac catheterisation
o Determine the severity and direction of shunt
How would you manage?
o Counsel
o Medical
o No antibiotic prophylaxis required, repaired or unrepaired
o Treatment of complications such as heart failure and AF
o May consider anticoagulation if there is evidence of bidirectional shunting to prevent strokes from paradoxical
emboli
o Surgery
o Early childhood closure is recommended at 5-10 years of age to prevent complications
o Small ASDs can be left alone(5 mm or less)
o Large ASDs or pulmonary to systemic flow ratio>1.5
o Closed surgically or transcatheter button or clam-shell devices
o Closure prevents pulmonary hypertension and RHF but does not alter incidence of AF
How would you counsel a patient with ASD who intends to get pregnant?
o Pregnancy is well tolerated in patients with small and hemodynamically insignificant ASD
o For large defects with pulmonary hypertension, Eisenmengers syndrome, avoid pregnancy as there is increase morbidity
and mortality both to fetus and mother
o Routine closure before pregnancy as complications of progressive pulmonary vascular disease may develop
21

11. HOCM
Presentation
Sir, this patient has got Hypertrophic obstructive cardiomyopathy.
There is presence of a ESM heard best at the LLSE. It is a grade 3/6 murmur as it is not associated with any systolic thrill. In
addition, there is presence of a MR mumur with a PSM heard bst at the ap3ex beat and radiates to the axilaa. It is a grade 4/6/
murm,ru as it is associated with a systolic thrill. The first heart sound is soft and there is no associated third or fourth heart sounds.
th

The apex beat is not displaced located at the 5 IC space just medial to the midclavicular line. It has a double apical impulse (say
this only if not if AF).
There are no complications of congestive cardiac failure. However the JVP is raised at 3 cm with a prominent a wave.
Examination of the peripheries did not show any stigmata of infective endocarditis. He is is SR at a pulse rate of 84 bpm and has a
characteristic bifid pulse (only if not in AF; if in AF, say shrap, rising and jerky pulse)
I did not notice any clinical features to suggest Friederichs ataxia.
I would like to complete my examination by taking performing the valsalva manoeuvre or standing to accentuate the murmurs as
well as take the patients blood pressure and look for fever from the temperature chart. A neurological examination would be useful
to screen for any signs of stroke.
In summary, this patient has got a HOCM with an ESM and MR murmur associated with a double apical impulse, bifid pulse and a
raised JVP with prominent awave. There are no complications of heart failure, AF or IE. This is a genetic condition.
Questions
What is HOCM?
o Hypertrophic Cardiomyopathy
o Genetic cardiac disorder caused by missense mutation in the genes that encode proteins of the cardiac sarcomere;
autosomal dominant
o Resulting in hypertrophy of the ventricular septum with LV outflow tract obstruction
o 1 in 500, male:female 1:1
o Variable penetrance
o Variable expression
o Asymptomatic (majority)
o Symptomatic
o Angina, syncope, dyspnea, palpitations
o Sudden death (Ventricular fibrillation) (overall annual mortality in 1%)
o Complications of CCF, AF, IE and thromboembolic stroke
Why is there a double apical impulse?
o Presence of a LV heave with a prominent presystolic pulse caused by atrial contraction
o A differential diagnosis is LV aneurysm
Why is there a prominent a wave?
o Due to forceful atrial contraction against a non-compliant right ventricle
What is Brockenbrough-Braunwauld-Morrow sign?
o Reduced pulse pressure in the post-extrasystolic beat
o Occurs in HOCM and AS
What are the causes of HOCM?
o Familial
o Friederichs Ataxia
o Idiopathic

22

How would you investigate?


o ECG
o Normal in 25%
o Tall QRS in precordial leads with ST-T changes, Q in inf and lateral leads
o LAD
o AF
o CXR
o Normal
o LA enlargement, LVH
o Echocardiogram
o Diagnostic
Asymmetrical septal hypertrophy
Systolic anterior motion of the anterior mitral valve leaflet
Diastolic dysfunction
o Severity
Septal thickness >18mm
Outflow tract gradient > 40mmHg as rest
o Complications
MR
IE
o TMX for those with angina
o Holter monitoring looking for arrythmias especially presence of VT
How would you predict poor outcome?
o Family history of sudden death
o History of syncope, cardiac arrest
o Poor BP response to exercise
o Holter monitoring with ventricual arrythmias detected, esp spontaneous VT
o Echo findings
How would you treat?
o Education and counselling with screening of first degree relatives
o 50% chance of being affected
o Screen with ECG and 2D echo
Annually for adolescent (12-18)
And 5 yearly
o Treatment is directed at symptom relief and prevention of sudden cardiac death
o Relief symptoms
Beta blockers
If cannot tolerate, verapamil but caution I patients with sever symptomatic obstruction because of
increase death especially after first few doses
Beta blockers and disopyramide
o Rx complications
Rx CCF
Rx AF
Rx and prevention of IE
Prevention of sudden death
Amiodarone
Pacing (dual-chamber pacing)
o Septal ablation with alcohol or surgery
o Surgical septal myomectomy (Gold standard)

23

12. Approach to Central cyanosis and Clubbing


Examination
o On detecting this, concentrate on
o Differential cyanosis and clubbing (ULs vs LLs or right LL vs others where the LLs are cyanosed and clubbed)
o Look for weak L radial pulse (BT shunt)
o Shunt scar (BT shunt)
o On auscultation determine if
o Eisenmenger
ASD, VSD, PDA
No PS
Has pulmonary hypertension (loud and palpable P2) and RVH
Check single (VSD) or fixed splitting (ASD)
o Fallots tetralogy
PS murmur (No VSD murmur as this is non restrictive)
No pulmonary hypertension but has RVH
Presentation
o Sir this patient has got VSD/ASD/PDA complicated by Eisenmengers complex.
o Eisenmengers because - clubbed and central cyanosis and Pulmonary Hypertension
o The underlying cause is ASD/VSD/PDA because - second heart sound is crucial
o ASD fixed spitting second heart sound
o VSD Single second heart sound
o PDA reversed splitting second heart sound (split on expiration)
o No PS murmur to suggest ToF
o Apex beat
o CCF
o IE stigmata
o Pulse
o Peripheral presentation for ASD/VSD/PDA
o (For Eisenmengers syndrome, will have pulmonary hypertension and therefore look for TR and PR murmur)
o Also state complications of polycythaemia, venesection marks
o Requests
o Summary
o
o
o

o
o
o
o
o
o
o
o
o
o

Sir this patient has ToF with a BT shunt done previously


Clubbed and centrally cyanosis
Presence of PS murmur and shunt murmur
o PS murmur ESM heard best at the pulmonary area 4/6 and systolic thrill
o Shunt murmur (continuous murmur)
o (No VSD murmur as it is large and non-restrictive)
RVH with left parasternal heave
Apex beat
No pulmonary hypertension (no loud P2)
Cor pulmonale raised JVP, pedal oedema, no lung crepitation
No IE
Pulse
Presence of a thoracotomy scar with a weak left radial pulse suggesting BT shunt
Venesection marks, polycythemia
Requests
In summary

Questions
How do you differentiate ToF vs Eisenmengers syndrome?
o ToF has PS murmur with systolic thrill and a soft P2
o ToF no pulmonary hypertension (CXR ToF has small pulmonary aretrial trunks)
What are the characteristic findings of a PDA?
o Collapsing pulse
o Continuos murmur heard best just below the left clavicle and radiates to the back
What are the differential diagnoses for a continuous murmur?
o Collapsing pulse
o PDA
o MR with AR
o VSD with AR
o No collapsing pulse
o BT shunt
o Venous hum (right of the sternum, children, disappears when lie flat or right JVP occluded)
What is ToF?
o Congenital heart condition comprising of
o VSD, RVH, Overriding aorta, PS

24

13. Dextrocardia
Examination
After the routine examination
Request to examine the abdomen for a liver on the left side of the abdomen for situs inversus
Presentation
Sir this patient has dextrocardia as evidenced by:
o Right apex beat
o Heart sounds that are better heard on the right than on the left
The heart sounds are normal and there are no murmurs detected.
th
Apex is not displaced located at the right 5 IC just medial to the midclavicular line and has a normal characteristic.
She is in SR with a rate of 84bpm
On examination of his lungs posteriorly, there was no evidence of coarse late inspiratory crepitations to suggest bronchiectasis and
patient does not have a nasal voice to suggest sinusitis. (Katargeners syndrome)
There is no evidence of Turners syndrome. (mention this only if female!)
I would like to complete my examination examining the abdomen for a left sided liver for situs inversus.
In summary this patient has got dextrocardia and is well clinically and is of congenital etiology.
Questions
What is the significance of situs inversus in patients with dextrocardia?
o It usually implies that there is no significant cardiac malformation
What conditions is dextrocardia associated with?
o Kartageners syndrome a type of immotile ciliary syndrome
o Triad of
Bronchiectasis
Sinusitis, otitis media and dysplasia of the frontal sinuses
Infertility
o Turners syndrome
o Asplenia PBF may show Heinz bodies and Howell-Juoly bodies
What is situs inversus?
o Right sided apex and right descending aorta
o Left lung having 3 lobes and right lung with 2 lobes
o Left sided liver and right sided stomach
o Right descending colon
What is dextroversion?
o Right sided apex and left sided descending aorta
o Left sided stomach
What is levoversion?
o Left sided apex and right sided descending aorta
o Right sided stomach

25

RESPI!
14. Bronchiectasis
Presentation
Sir, this patient has got bronchiectasis affecting both lower lobes as evidenced by late, coarse inspiratory crepitations heard best
posteriorly in the lower one third bilaterally. Patient has a productive cough with large volume of purulent sputum with hemoptysis
associated with clubbing.
Chest excursion was reduced bilaterally with a normal percussion note and vocal resonance. Trachea is central and the apex beat
is not displaced.
There are no signs to suggest presence of COPD.
(There is concomitant COPD with a reduced chest excursion bilaterally, hyperinflation of the chest associated with hyperresonance
on percussion with loss of liver and cardiac dullness. There is presence of ronchi and a prolonged expiratory phase. Vocal
resonance is normal. Trachea is central and apex beat is not displaced.)
There is complication of pulmonary hypertension with a loud and palpable component of the second heart sound associated with a
left parasternal heave. There is also cor pulmonale with a raised JVP of 3 cm with prominent a wave associated with bilateral pedal
oedema. Clinically there are no signs of polycythemia such as plethoric facies or conjunctival suffusion.
He is not in respiratory distress (with a RR of 14 bpm without use of accessory muscles of respiration). There are no signs of
respiratory failure (he does not require any supplemental oxygen and there is no central cyanosis; there is also no flapping trem or
of the hands and no bounding pulse). There is also no nicotine staining of the fingers, patient is not cachexic looking and no
enlarged Cx LNs.
With regards to aetiology, there is no dextrocardia or a nasal voice to suggest possible Kartageners syndrome. In addition, there is
no symmetrical deforming polyarthropathy to suggest RA or any cutaneous signs of SLE. There is no kyphoscoliosis.
With regards to treatment, patient has a steroid metered-dose inhaler, salbutamol and ipratropium metered-dose inhalers by the
bed side.
I would like to complete the examination by looking at the temperature chart for fever as well as an abdominal examination to look
for splenomegaly from amyloidosis which can result from bronchiectasis. A neurological examination is useful to screen for deficit
as patients are prone to brain abscesses.
In summary, this patient has bronchiectasis affecting both lower lobes with complications of pulmonary hypertension and cor
pulmonale. There is no concomitant COPD and no polycythemia. He is clinically not in respiratory failure. The possible causes for
this patients bronchiectasis are post infective causes such as post viral, bacterial, TB or ABPA, connective tissue disease such as
RA or SLE, congenital conditions such as cystic fibrosis, Kartageners syndrome or hypogammaglobulinemia.

26

Questions
What are your differential diagnoses for a patient that is clubbed and has crepitations?
o
Bronchiectasis
o
Pulmonary fibrosis
o
Mitotic lung lesion
o
Abscess
What is bronchiectasis?
o
Definition: permanent dilatation of the bronchi
o
Pathology: Retained secretions and chronic inflammation
o
Clinical course: Chronic, progressive with recurrent infective exacerbations
o
Clinical: Symptoms - productive purulent cough, dyspnea and hemoptysis and Signs: coarse late inspiratory crepitations with a 3
layered purulent sputum
What are the causes of bronchiectasis?
o
Focal
o
Luminal blockage FB, broncholith
o
Arising from the wall mitotic lesion of the lung
o
Extrinsic enlarged LNs esp middle lobe from TB/fungi; displacement of airways post lobar resection
o
Diffuse
o
Post infectious conditions
Bacteria Pseudomonas, Hemophilus, Pertussis
TB
Aspergillus (for upper lobe or proximal bronchiectasis) as in allergic bronchopulmonary aspergillosis from type
III immune complex reactions.
Virus adenovirus, measles, influenza
o
Congenital conditions
Cystic fibrosis
Alpha 1 Antitrypsin deficiency
Kartageners syndrome of immotile ciliary syndrome
Hypogammaglobulinemia
o
NB: Immunodeficiency form secondary causes such as cancer, chemotherapy or immune
modulation post transplant
o
Rheumatic conditions
RA (1-3% of patients)
SLE
Sjogrens
o
Others
Yellow nail syndrome (yellow nails, bronchiectasis, pl effusion and lymphedema)
Youngs syndrome(secondary ciliary dyskinesia from mercury intoxication)
Inflammatory bowel disease (UC or Crohn)
Congenital kyphoscoliosis
Idiopathic (50%)
What is bronchiectasis sicca?
o
dry bronchiectasis
o
Presents with recurrent hemoptysis and dry cough
o
Affects the upper lobes therefore good drainage
o
Usually from past history of granulomatous infection eg TB
What is Kartageners syndrome?
o
It is a type of immotile ciliary syndrome
o
Comprising of
o
dextrocardia, situs inversus
o
bronchiectasis, sinusitis, frontal sinus dysplasia, otitis media
o
infertility
o
Resulting in poor ciliary function with retained secretions and recurrent infections and thus bronchiectasis
What is cystic fibrosis?
o
Most commonly due to mutations to CFTR (CF transmembrane conductance regulator) with F508
o
Recurrent respiratory infections with pancreatic exocrine deficiency and short stature
o
Upper lobe involvement
o
Staph aureus, Ps aeuroginosa
o
Elevated sweat Na and Cl concentrations
What are the differences in bronchiectasis vs COPD?
o
They may both occur concomitantly
COPD
Cause
Cigarette
Infection
Secondary
Organism
S. pneumoniae, Haem
Symptoms
Dyspnea, chronic cough
Sputum
Mucoid clear
CXR
Hyperlucency, hyperinflated

Bronchiectasis
Infection, genetic
Primary
Haem, Pseudomonas
Dyspnea, hemoptysis, productive
3 layered, purulent
Airway thickening, dilated

27

What are the complications of bronchiectasis?


o
Pneumonia, collapse, pleural effusion, lung abscess, pneumothorax, hemoptysis
o
Brain abscess
o
Sinusitis
o
Amyloidosis
How would you investigate?
The diagnostic investigation of choice is a HRCT but simple Ix such as CXR and LFT are also useful:
o
CXR Diagnosis, extent and complications
o
90% abnormal
o
Diagnosis
specific
dilated and thickened airways
Ring shadows (seen on end)
Tram lines
Non-specific
Linear or plate-like atelectasis
Scattered irregular opacities
Focal pneumonitis
o
Extent and distribution
o
Complications
Pneumonia, abscesses, pleural effusion
o
Lung function test
o
Obstructive pattern with FEV1/FVC <70%
o
Severity of obstruction based on FEV1
o
Reversibility with beta agonist
40% of patients have >15% improvement
o
High-resolution computer tomography scan of the thorax
o
Non-contrast study with 1 mm cuts every 1 cm with acquisition time of one second during full inspiration (requires
patient cooperation); 90% sensitivity
o
Diagnostic
Dilatation of airway lumen >1.5X cf to a nearby vessel
Signet ring sign (dilated bronchus with its pulmonary artery)
Lack of tapering of an airway toward the periphery with presence of bronchi within 1 cm from the pleura
Reids classifications
Cylindrical or tubular
Varicose
Saccular or cystic
Useful also in elucidation cause of focal bronchiectasis
o
Assess distribution
Usually lower lobes
If upper lobes suspect Cystic fibrosis or ABPA
If proximal bronchiectis, ABPA
If ML or lingula M. avium complex
o
Complications

28

How would you manage?


o
Non-Pharmacological
o
Education and counselling
o
Stop smoking, vaccinations (yearly influenza and 3-yearly pneumococcal)
o
Chest percussion and postural drainage (no evidence actually)
o
Rx underlying cause
o
Pharmacological
o
Rx acute exacerbations
o
ODonnells 4/9 symptoms of exacerbations
Increased dyspnea
Increase cough
Increase sputum production
Increased wheezing
Fever
Lethargy, malaise
Changes in chest sounds
Reduced pulmonary function
Radiographic changes consistent with a new pulmonary process
o
Antibiotics targeting
Haem, Ps and Strep and Moraxella
Fluoroquinolones
Others
MAC Rifampiciin, ethambutol and Azithro till c/s negative for 1 year
ABPA augmentation of corticosteroids and use of itraconazole 200mg bd for 4 weeks then
200mg om for 4 more weeks
o
Bronchodilator therapy such as beta agonists and anticholinergics with inhaled corticosteroids
o
Improve lung function (FEV1) and reduce sputum volume
o
No effect on mortality
o
Aerosolised recombinant human DNAse for cystic fibrosis (not for other causes of bronchiectasis)
o
Surgery
o
Focal
o
Removal of obstructing tumour or FB
o
Diffuse
o
Segments that are most damaged and contributing to recurrent acute exacerbations
o
Segments involved with uncontrolled haemorrhage
o
Removal of segments suspected of harbouring drug resistant organism such as MDR MTB or MAC
o
Lung transplant
How do you manage complication of hemoptysis?
o
Quantify
o
If >600mls /day = massive
o
Lie on the affected side
o
Protect airway
o
Bronchoscope or CT to determine site of bleed
o
Interventional radiology or surgical removal

29

15. Interstitial Lung Disease


Presentation
Sir, this patient has interstitial lung disease affecting both lower lobes (upper lobes) as evidenced by fine velcro-like late inspiratory
crepitations heard best posteriorly(anteriorly) in the lower one third bilaterally. This is associated with clubbing(50%) and a nonproductive cough.
Chest excursion was reduced bilaterally with a normal percussion note and vocal resonance. Trachea is central and apex beat is
not displaced.
There are no signs of pulmonary hypertension or cor pulmonale. There are also no features of polycythemia.
Patient respiratory rate is 14 breaths per minute and there are no signs of respiratory distress. There are also no signs of
respiratory failure. There is also no nicotine staining of the fingers and I note that the patient is cachexic looking with wasting of the
temporalis muscles.
In terms of aetiology, there is no symmetrical deforming polyarthropathy of the hands to suggest RA, or cutaneous signs to suggest
presence of SLE, dermatomyositis or scleroderma as these conditions may be complicated by pulmonary fibrosis.
With regards to treatment, patient is not Cushingoid and does not have papery thin skin or steroid purpura to suggest chronic
steroid usage. On inspection there are no surgical scars to suggest open lung biopsy.
I would like to complete the examination by asking for a detailed drug history as well as an occupational history.
In summary, this patient has got pulmonary fibrosis affecting bilateral lower lobes. There are no complications of pulmonary
hypertension, cor pulmonale and polycythemia. He is clinically not in respiratory failure and has no features of chronic steroid
usage. The differential diagnoses include collagen vascular disease, drugs, occupational causes and idiopathic pulmonary fibrosis.
Questions
What are the differential diagnoses for clubbing and crepitations?
Pulmonary fibrosis
Bronchiectasis
Lung abscess
Mitotic lung conditions
What are the characteristic auscultatory findings?
Late, fine inspiratory crepitations
Velcro-like
Disappears or quietens with the patient leaning forwards
What are the causes of fibrosis?
Upper Lobes
S Silicosis, sarcoidosis
C- coal worker pnemoconiosis
H- histiocytosis
A- Ankylosing spondylitis, ABPA
R radiation
T TB
Lower lobes
R- RA
A-Asbestosis
S- Scleroderma
I Idiopathic pulmonary fibrosis
O- others ie drugs
Cytotoxics MTX, Aza, bleomycin, bulsulphan, cyclo, chlorambucil
CNS - Amitryptyline, phenytoin and carbamazepine
CVS - Amiodarone, hydralazine, procainamide
Antibiotics - Nitrofurantoin, isoniazid
Antirheumatics Gold, sulphasalazine
Both
N Neurofibromatosis, Tuberous sclerosis
E Extrinsic allergic alveolitis (acute symptoms within 6 hrs of inhaled allergens eg farmers lungs)
P pulmonary haemorrhage syndromes
A alveolar proteinosis
Primary
Secondary Inhaled organic dusts(Silica, Al), chronic infection, malignancy
Lymphangiomyomatosis

30

How would you classify interstitial lung disease? (ATS/ERS 2001)


Diffuse parenchymal lung disease(DPLD) of known cause
Collagen Vascular disease
RA, SLE, Dermatomyositis, Systemic sclerosis
Occupational/Environmental
Asbestosis, silicosis, extrinsic allergic alveolitis
Drug related
Cytotoxic, CNS, CVS, Antibiotics and antirheumatic
Idiopathic
IPF
Other idiopathic interstitial pneumonias
DIP
AIP
LIP
NSIP
Cryptogenic organising pneumonia
Respiratory bronchiolitis
Granulomatous
Sarcoidosis
Others - LAMs, histiocytosis
How would you diagnose idiopathic pulmonary fibrosis?
Clinical-radiological-pathological diagnosis
Clinical
o
Exclusion of other causes of ILD
o
>50 yrs, insidious onset of dyspnea, > 3months, non-productive cough
o
Typical physical findings
Radiological (see below)
Pathological (see below)
How would you investigate?
The diagnostic Ix of choice is a HRCT of the thorax but simple IX such as CXR and LFT are useful:
CXR
Diagnostic
bilateral basal reticulonodular shadows, peripheries, which advances upwards
honeycombing in advanced cases (gps of closely set ring shadows)
loss of lung volume
Extent and distribution
Complications
Lung function
Restrictive pattern (reduced TLC or VC with increased FEV1/FVC ratio)
Severity of restriction based on TLC
Reduced transfer factor (impaired gas exchange)
HRCT scan
Dx patchy reticular abnormalities, focal ground glass, architectural distortion, volume loss, subpleural cyst, honeycombing (no
consolidation or nodules)
Extent and severity basal, peripheral, subpleural
Complications
NB: Similar to that of collagen vascular disease and asbestosis
Others
Bronchoscopy lavage
Predominantly lymphocyte responds to steroids and better Px= not UIP
Predominantly neutrophils and eosinophils means poor Px= UIP (if >20% of eosinophils to consider eosinophilic lung
disease)
Lung biopsy
IPF Usual interstitial pneumonia
Bloods
ABGs
To rule out causes
How would you manage?
Education and counselling
Stop smoking
Regular follow up and vaccinations
Treat underlying cause
Pharmacological
Trial of steroids
If responding continue steroids
If not responding, cyclophosphamide or azathioprine
Antifibrotic agents
Eg penicillamine which has not been proven to be useful
Surgical: Lung transplant (single lung transplantation)
Manage complications
Cor pulmonale - diuresis for heart failure
Polycythemia - venesection if Hct >55%
Respiratory failure Oxygen therapy
Monitor for lung cancer

31

What are the good prognosticating factors?


Young age
Female
Short duration
Ground glass appearance on the CXR
Minimal fibrosis on lung biopsy
What is the clinical course of patients with IPF?
Gradual onset
Progressive
Median survival from time of dx about 3 years
What are the causes of death?
Cor pulmonale
Respiratory failure
Pneumonia
Lung carcinoma
What is Hamman-Rich syndrome?
Rapidly progressive and fatal variant of interstitial lung disease

32

16. COPD
Presentation
Sir, this patient has severe COPD that is complicated by pulmonary hypertension, cor pulmonale and polycythemia. He is
tachypneic at rest and requires use of intranasal oxygen supplementation.
Patient has got hyperinflated chest with reduced chest expansion bilaterally at 2cm. The percussion note is resonant with loss of
liver and cardiac dullness. There is prolonged expiratory phase with expiratory ronchi. Vocal resonance is normal. Trachea is
central and apex beat is not displaced.
There is complication of pulmonary hypertension as evidenced by loud and palpable P2 associated with a left parasternal heave.
There is also cor pulmonale with raised JVP of 4cm with giant V waves associated with bilateral pedal oedema. There are also
features of polycythemia with plethoric facies and conjunctival suffusion.
The patient is in respiratory distress. He is tachypneic at rest with a RR of 20 bpm and uses his accessory muscles of respiration at
rest. He is also in respiratory failure with presence of central cyanosis and is oxygen dependent. However, he does not have a
flapping tremor or a bounding pulse to suggest CO2 retention clinically.
In terms of aetiology, the presence of nicotine staining of his fingers implies significant history of smoking. He is not clubbed. The
Cx LNs are not enlarged and he is not cachexic looking.
There is presence of steroid MDI as well as bronchodilators by his side. There is no evidence of a hoarse voice or oral thrush or
other features of chronic systemic steroid usage.
I would like to complete the examination by testing patients forced expiratory time, checking his temperature and examining his
sputum.
In summary, this patient has got severe COPD with complications of pulmonary hypertension, cor pulmonale and polycythemia. He
is in respiratory failure and respiratory distress. The most likely aetiology is smoking.
Questions
How do you dx COPD?
Clinical (>35 years, smoking, wheeze, SOB, cough with sputum, winter bronchitis)
Airflow obstruction FEV1/FVC<70 and FEV1<80
o Mild 50-80%
o Mod 30-50%
o Severe - <30%
Exclude differential diagnoses
o Asthma (>400mls to dilators or PO pred 30mg OM 2 weeks or variation in PEFR >20%)
o Cancer
o Bronchiectasis
o ILD
How do you grade the severity of dyspnea?
MRC scale
o 1 SOB on strenuous exercise
o 2 on hurrying or up hill
o 3 walks slower than contemporaries and stops for breaths
o 4 stops for breath after walking 100m
o 5 SOB on ADLs
How would you investigate admitted with an acute exacerbation?
FBC (anaemia, polycythemia), biochemical, theophylline levels, ABG
Blood C/S if febrile
CXR
Spirometry
ECG, 2D echo

33

How would you manage?


Non-pharmacological
o Stop smoking
o Regular follow up (if >500 mls decline over 5 years implies accelerated decline)
o Pneumococcal and influenza vaccination
o Pulmonary rehabilitation for MRC 3 or above(PT/OT)
o MSW, Nurse
o Assessment of inhaler technique (should not clean space more than once a month due to increased static)
Pharmacological
o Bronchodilators
Beta agonist and anticholinergics
Short acting and long acting
Improves symptoms and exercise capacity
o Theophylline
o Steroids
Reduce exacerbations and decline in health status
Used if
FEV1<50%
2 or more exacerbations a year requiring antibiotics or steroids
Prophylaxis against osteoporosis (once >65)
o Mucolytics (not anti-tussive)
o Management of exacerbation
Bronchodilator - Nebs or via spacer
Systemic steroids
IV aminophylline
Antibiotics such as macrolide (increase volume or purulence)
Oxygen therapy
Intranasal if hypoxic
NIPPV if hypercapneic and pH 7.25-2.35
Intubation
Manage complications
o Hypoxemia assessment for need of LTOT
PaO2 <55 mmHg
PaO2 <60 mmHg and presence of
Polycythemia
Pulmonary hypertension
Cor Pulmonale
Nocturnal hypoxemia
For at least 15 hrs/day if not 20 hrs
o Cor pulmonale
Diuretics
o Polycythemia
>55% consider venesection
Surgical
o Bullectomy
Single large bulla
FEV1<50%
o LVRS
Upper lobe bullous involvement
FEV1>20%
TLCO >20%
PO2 <45
o Transplant
When would you order a AAT levels?
COPD with
o Young, Family H/O, No smoking history
Not recommended for AAT replacement if present; treat the COPD
How would you advice on air travel?
FiO2 at 15% - hypoxemia
Pressurised at 8000 ft pneumothorax
Assessment
o H/O and PE and H/O of problems encountered during previous flight
o Spirometry
o SpO2 <95% on RA
50m walk test
Hypoxic challenge test
Bring inhalers in the hand luggage
Inform the airline
See Link to Air Travel Advice
34

17. Pleural Effusion


Presentation
Sir, this patient has a right sided moderate pleural effusion affecting the lower two thirds of the right hemithorax.
There is reduced chest excursion of the right hemithorax associated with stony dullness with reduced/absent vesicular breath
sounds and vocal resonance affecting the lower two thirds of the right hemithorax. Apex beat was not displaced and trachea was
central in position. There are no scars on the chest wall to suggest a previous chest tube or a thoracotomy.
Patient respiratory rate is 14 bpm with no signs of respiratory failure or distress.
With regards to aetiology: (State the positives first and rearrange accordingly)
1. Patient does not have any raised JVP, S3or pedal oedema to suggest CCF. There are also no stigmata of chronic liver disease
or generalised oedema or renal biopsy scar to suggest nephrotic syndrome. Patient does not have features of hypothyroidism.
2. There is no nicotine staining of the fingers, no clubbing, no palpable cervical lymph nodes and he is not cachexic. I did not detect
any signs of SVCO, Horners syndrome and patient does not have a hoarse voice to suggest a malignant effusion.
I could not detect any bronchial breathing above the effusion. The patient is not toxic looking and did not detect any Mantoux
testing with respect to TB pleural effusion.
There is also no deforming polyarthropathy of RA or cutaneous signs of SLE.
There is no calf swelling or tenderness noted to suggest DVT.
With regards to treatment, I did not notice (if there are features of malignancy) any radiation marks on the right hemithorax and
there are also no features of side effects of chemotherapy such as alopecia or oral ulcers.
I would like to complete the examination by looking at the patients temperature chart as well as examining his sputum and examine
the patients breasts (if female).
In summary, this patient has got a moderate size right sided pleural effusion. He is not in respiratory distress or failure. In view:
1. Patient is cachexic, the likely underlying aetiology includes tuberculous pleural effusion or a malignant pleural effusion.
2. Fever, (young patient, short history) the most likely aetiology for this patient includes a parapneumonic effusion. Other
diagnosis includes tuberculous effusion, malignant effusion or autoimmune cause.
3. Patient has Cx lymphadenopathy, I would like to offer the diagnosis of tuberculous effusion. Another possibility is a malignant
effusion.
4. Patient has complications of SVCO/Horners syndrome/Clubbing with HPOA/Nicotine staining of the nails/tender ribs/chest
wall, he has a malignant effusion.
5. Patient has vasculitic lesions of the hands, joint deformities/tenderness, malar rash. The aetiology of the effusion is most likely
due to collagen vascular disease/SLE/RA.
6. Young female, aetiologies include CTD, hypothyroidism and TB
Questions
What are your differential diagnoses for dullness on percussion of the right lower zone?
1. Pleural thickening :Old TB, old empyema, mesothelioma, asbestosis, PHx of hemothorax
2. Basal consolidation
3. Lower lobe collapse
4. Raised hemidiaphragm:
a. phrenic nerve palsy from Ca or phrenic nerve crush for old TB treatment with supraclavicular fossa scar
b. hepatomegaly
5. Mitotic mass
What are the causes of a pleural effusion?
o Transudative
o CCF, constrictive pericarditis
o Nephrotic syndrome, hypoalbuminemia, peritoneal dialysis
o Chronic liver disease (hepatic hydrothorax)
o Myxoedema
o Atelectasis
o Exudative
o Malignancy
Primary - bronchial or pleural
Secondaries Breast, pancreas, kidneys, ovaries, lymphomas
o Infective parapneumonic, TB
o CTD SLE, RA
o PE (can also be transudative but less common)
o Pancreatitis (left sided)
o Drug induced nitrofurantoin, bromocriptine
o Meigs syndrome (ovarian fibroma with ascites and pl effusion)
o Yellow nail syndrome (triad of yellow nails/onycholysis, pl effusion/bronchiectasis and lymphedema)
What are the causes of hemothorax?
o Trauma
o Rupture of pleural adhesion containing blood vessel, carcinoma
35

What are the causes of a chylothorax?


o Trauma or surgery to the thoracic duct
o Carcinoma or lymphoma affecting the thoracic duct
What are the causes of an empyema?
o Pneumonia
o Abscess
o Bronchiectasis
o TB
How do you confirm the diagnosis of a pleural effusion?
o Perform a lateral decubitus film to look for layering
o USS
How do you differentiate between an exudative and transudative effusion?
o Lights criteria
o Sensitive but not specific for exudates
o One out of 3 criteria
Pl fluid protein: serum protein > 0.5
Pl fluid LDH:serum LDH>0.6
Pl fluid LDH > 2/3 upper limit of serum LDH
o If suspect transudate still, can do the serum to pleural albumin gradient (difference). If > 1.2 g/dL, transudate. (less
sensitive for exudates)
How would you investigate?
o CXR
o Sputum for gram stain, c/s and AFB smear, cytology
o Diagnostic pleural tap and pleural biopsy
o Indications for a diagnostic thoracocentesis
>10mm thick on a lateral decubitus film or USS
o Appearance
Bloody appearance
<1% insignificant
1-20% = malignancy, PE or trauma
>50% cf to peripheral Hct = hemothorax
Turbid (parapnemonic, chylothorax)
Putrid odour (anaerobic)
o Haemotological Ix
Total cell > 1500 cell/ml
>50% neutrophils (parapneumonic)
Lymphocyte predominant (cancer, TB, lymphoma, CTDs)
Mononuclear cells chronic
Eosinophils
Blood or air in the pleural space
Drugs nitrofurantoin, bromocriptine, dantrolene
Churg-Strauss
Paragonimiasis
Asbestosis
o Biochemical
Lights criteria and serum pleural albumin gradient
pH
Glucose (<0.5 cf to peripheries = TB, malignancy, RA)
Amylase level (pancreatitis)
o Microbiological
Smear and C/S
AFB smears
PCR, ADA(adenosine deaminase) or gamma interferon
o Cytology
Fluid
Pleural biopsy
o Others
o Blood Ix, mantoux
o Bronchoscope, CT scan
When must you order a CXR post Dx tap?
o When you suspect complications of pneumothorax
o Air is aspirated
o Patient develops cough, chest pain or dyspnea
o Loss of tactile fremitus over the superior part of the aspirated hemithorax

36

How would you manage?


o Treat the underlying cause
o Treat symptoms
o Fever, pain
o SOB therapeutic thoracentesis (up to 1500mls)
o Chest tube insertion (tube thoracostomy)
o Complicated parapneumonic effusion
Gross pus or empyema
pH <7.2
Glucose <3.0
G/S positive for organism
LDH >1000
o Hemothorax
o Pleurodesis for malignant effusions
What are the surface markings for a respiratory examination?
Anteriorly
th
o 4 rib and above = upper lobe on right
th
th
o between 4 to 6 rib = middle lobe on the right
th
o 6 rib and below on the left = lower lobe
th
th
Horizontal fissure extends on the right extends from 4 rib to midaxillary line where it bisects the oblique fissure at the 5 rib
th
Oblique fissure extends from the 6 rib anteriorly to the back at T2
Lower lobe extends posteriorly from T2 to T11
Upper lobe posteriorly extends from T2 and above
Prominent Cx spine = C7
Inferior angle of the scapula = T7

37

18. Collapse
Presentation
Sir, this patient has a right upper lobe collapse as evidenced by reduced chest excursion of the right hemithorax associated with
dullness on percussion and reduced vesicular breath sounds and reduced vocal resonance affecting the upper one third of the right
hemithorax. This is associated with tracheal deviation to the right. There was no displacement of the apex beat.
Patients respiratory rate is 14 bpm and is not in respiratory distress or failure.
With regards to aetiology:
There are signs to suggest underlying malignancy. Patient is cachexic looking and is clubbed. I did not detect any tenderness in the
wrists to suggest HPOA. There were no enlarged palpable Cx LNs. There is also no conjunctival pallor or jaundice noted. I did not
detect any associated pleural effusion or raised hemidiaphragm. There were no signs of SVCO, patients does not have a right
Horners syndrome and there is no wasting of the intrinsic muscles of the right hand (hoarseness of voice for left sided lesion).
In considering other etiologies:
There are also no ronchi on auscultation to suggest asthma or allergic bronchopulmonary aspergillosis.
There are no Mantoux testing detected on the upper limbs but endobronchial TB is a possible differential diagnosis.
I did not find any signs of treatment such as radiotherapy hyperpigmentation or side effects of chemotherapy such as alopecia,
phlebitic veins or oral ulcers.
I would like to complete my examination by looking at the patients temperature chart as well as examining his sputum.
In summary, this patient has a right upper lobe collapsed and is clinically comfortable. In view that patient has:
1.
2.
3.

A history of weight loss and is cachexic looking, the possible diagnoses include endobronchial mitotic lesion or an
endobronchial tuberculous infection.
Fever/cough/hemoptysis/Cx lymphadenopathy, the possible diagnoses includes an endobronchial tuberculous infection,
endobronchial mitotic lesion or a collapse consolidation from a pneumonia.
Complications of Right Horners syndrome, signs of SVCO, clubbed with HPOA, the most likely cause is an endobronchial
mitotic lesion affecting the right upper lobe.

Questions
What are the causes of a lung collapse?
Intraluminal Mucus plugging from asthma or ABPA, FB
Endobronchial tumor, TB
Extrinsic compression enlarged LNs for mitotic lesion (pri or sec), lymphomas or TB
What is Brocks syndrome?
Collapse of the right middle lobe from enlarged LNs
How would you investigate?
Simple investigations
o
CXR
o
ABG, FBC and biochemical profile
o
Sputum AFB smear and c/s and cytology
Diagnostic - Bronchoscopy and Bx
Staging CT thorax and abdomen with adrenal cuts, bone scan
Physiological staging: Lung function test
o
FEV1 >1.5
o
Transfer factor>50%

38

How would you treat the patient?


Depending on the underlying cause
For mitotic lesion
o
Multidisciplinary approach
o
Education and counselling, support groups and stop smoking
o
Symptomatic treatment
o
For non-small cell
Assessment for surgical resectability
Staging (up to stage IIIA); ie once T4, N3 or M1 not a candidate
Physiological staging
Chemotherapy
Neoadjuvant
Adjuvant
Radiotherapy
Adjuvant
Palliative
Palliative
Radiotherapy
o
Pain, bone mets
o
Dyspnea from bronchial obstruction, dysphagia
o
SVCO, pancoast syndrome
Chemotherapy
o
For small cell: Chemotherapy
How does patient with bronchogenic carcinoma present?
Primary tumor
o
Cough, dyspnea, hemoptysis, pneumonia
Mediastinal spread
o
SVCO, Horners, pleural effusion, phrenic nerve palsy, hoarseness of voice, T1 wasting, pericardial effusion
Metastasis
o
Liver, bone, brain, skin, adrenal glands
Paraneoplastic symptoms
Systemic effects
o
LOA, LOW, fatigue
What are the paraneoplastic syndromes?
Endocrine
o
PTH-related peptide (hypercalcemia) SCC
o
SIADH Small cell (usually asymptomatic)
o
ACTH Cushings (usually hypokalemic metabolic alkalosis)
o
Gynaecomastia
Neurological
o
Subacute cerebellar degeneration
o
Peripheral neuropathies
o
Lambert-Eaton syndrome
Cardiovascular
o
Non-thrombotic endocarditis
Renal
o
Nephrotic syndrome, GN (membranous)
Skin
o
Migratory venous thrombopleblitis (Trosseaus sign)
o
Acanthosis nigricans
o
Dermatomyositis
o
Zoster
MSK
o
Clubbing, HPOA
Haematological
o
DIC
o
Anaemia
What is SVCO?
Tumour with obstruction of the SVC
Plethoric facies
Facial and UL oedema
Conjunctival suffusion
Undersurface of the tongue with multiple venous angiomata
Fixed engorgement of the neck veins
Stridor
Upper chest telangiectasia
Radiation marks
(NB think of polycythemia which also have plethoric facies)
Causes
o
Lung carcinoma, especially small cell
o
Lymphoma
o
Others mediastinal goiter

39

19. Consolidation
Presentation
Sir, this patient has a right upper lobe consolidation as evidenced by reduced chest excursion of the right hemithorax associated
with a dull percussion note, bronchial breath sounds and crepitations and increased vocal resonance. These signs were best heard
in the upper one third anteriorly in the right hemithorax. The trachea is central and apex beat is not displaced.
There are no signs to suggest that the patient is in respiratory distress or in failure.
With regards to aetiology, an underlying malignancy is considered as there is associated complication of SVCO (hoarseness of
voice if left sided), with plethoric facies, ruddy complexion a/w oedema of the face and upper limbs associated with suffusion of the
eyes, fixed engorgement of the neck veins, dilatation of the superficial veins of the neck, and venous angiomata detected on the
undersurface of his tongue. There is no Horners syndrome, wasting of the intrinsic muscle of the hands and no soft heart sounds to
suggest pericardial effusion. There was also no associated pleural effusion or a raised right hemidiaphragm.
He is also clubbed with HPOA and has nicotine staining of his fingers. He is cachexic looking with enlarged palpable cervical LNs.
There is also thrombophiblitis of the forearms which may suggest Trosseaus sign.
(if there are no signs of cancer, proceed to mention TB/pneumonia ie mantoux testing, toxic looking, productive cough with purulent
sputum; DVT ie swelling and tender calves)
There is presence of radiation therapy marks on the right chest wall as well as side effects of chemotherapy such as alopecia and
oral ulcers.
In summary, patient has a right upper lobe consolidation complicated by SVCO. He is not in respiratory distress. The underlying
cause is most likely a mitotic lesion of the lung.
Questions
What are the causes of a consolidation?
Infection
o Pneumonia
o Abscess
o TB
o Aspergilloma, cryptococcoma, hydatid cyst
Neoplastic or mass
o Carcinoma
o Lymphoma
Pulmonary infarction
How would you investigate?
Simple
o CXR
o ABGs, blood tests FBC and biochemical profile, blood c/s
o Sputum
Directed tests
o Infection
o Cancer Bronchoscopy and Bx, CT staging, bone scan, physio staging
o Infarction
What are the causes of an unresolved pneumonia?
FB
Tumor
Abscess
Inappropriate antibiotics, resistant organism
Bronchopulmonary sequestration
o Rare, congenital
o Non-functioning lung tissue with anomalous arterial supply with no connection to the bronchopulmonary tree
What are the extrapulmonary manifestations of Mycoplasma?
CNS meningitis, encephalitis
CVS percarditis, myocarditis
Hepatitis, GN
DIC, AIHA
EM, SJS
Arthralgia, arthritis

40

What are the complications of pneumonia?


Local
o Abscess
o Empyema
o Respiratory failure
Sepsis
o Septic shock
o ARDS
o MOF
o DIC
What are the pulmonary eosinophilic disorders?
9
Defined as radiographic infiltrates with hypereosinophilia (>1.5 x10 L)
Includes
o Churg-Strauss Asthma with vasculitis and hypereosinophilia
o Tropical pulmonary eosinophilia high anti-filarial Ab
o Chronic pulmonary eosinophilia cough, progressive SOB, weight loss with photographic negative pulmonary
oedema ie diffuse peripheral pulmonary infiltrates
How do you manage pneumonia? BTS 2004
Depending on the type of pneumonia
o CAP
o HCAP
o HAP
o note that HCAP is Rx the same way as CAP in BTS guidelines
Risk Stratify - CAP
o CURB-65 score 6 point score from 0 to 5
Confusion
Urea >7 mmol/L
RR >30 bpm
BP <90/60
Age 65 and above
o 0 to 1 low risk of death and home Rx
o 2 increased risk of death and short hospitalization or supervised home treatment
o 3 and above high risk of death and urgent hospitalization
Ix FBC, CRP, Bld c/s (for those with severe indicators or co-morbidities), sputum g/s and c/s, Lg and Pneumococcal
Urine Ag only for severe pneumonia
Mx
o Use of SpO2 monitoring advocated in GP setting
o NIV not for use in severe pneumonia unless in ICU setting
o Antibiotics
CAP- Penicillins or macrolides; fluoroquinolones if intolerant or selected inpatient treatment with PO
moxifloxacin preferred over levofloxacin
o Discharge planning
Should not be discharged within 24Hrs if have >1 of
T >37.8
HR>100
RR>24
SpO2<90%
BP sys <90
Inability to maintain oral intake
Abnormal mental status

41

How do you mange pulmonary infarction? BTS 2003


All patients with suspected PE should have clinical probability assessed
Hence evaluate patients for
o SOB or tachypnea, chest pain or hemoptysis
o Absence of an alternative explanation (a)
o Presence of a major risk factor (b)
Therefore if
o (a) and (b) present = high probability
o Only (a) or (b) present = intermediate probability
o None present = low probability
D-dimer
o To use only if low or intermediate probability
o Highly specific = if negative, no need to do imaging
o Not to order if high probability; image immediately
Imaging
o Done within 1 hr for massive and 24 hrs for non massive
o Imaging options
CTPA
Isotope lung scan
Facilities available
CXR normal
No cardiopulmonary disorder
Standardised reporting criteria
Non-diagnostic results followed by other imaging
USS DVT
Can be used if presence of clinical DVT and if positive, is sufficient to confirm VTE
If negative, cannot be used to confirm absence of VTE
o Management
Massive (BP compromised or cardiopulmonary collapse)
Thrombolysis with alteplase (No lowering BP effect cf with streptokinase)
Thrombus fragmentation and IVC filters if expertise present and available
Non-massive PE
NO thrombolysis
Use of heparin before imaging in high or intermediate clinical probability
Use LMWH as first choice
Use unfractionated heparin if
o First dose bolus
o Massive PE
o Acute reversal desired
If VTE confirmed
o Commenced oral anticoagulation
o INR 2-3
o Duration is 4-6 wks for temporary risk factors. 3 months for first idiopathic episode and
6 months for other clinical situations
Special populations
o Pregnancy
LMWH during pregnancy
UFH approaching delivery
UFH stopped or reduced 4-6hrs prior delivery
Oral anticoagulation commenced after delivery and continued for 6 weeks or
3 months after PE whichever is longer
o Cancer patients
Oral anticoagulation as per above
Duration unknown
Higher risk of thrombosis and bleeding
If recurrent thrombosis
INR 3-3.5 or
LMWH + anticoagulation or
IVC

42

43

20. Lobectomy/Pneumonectomy
1. Sir, this patient has left lobectomy as evidenced by a left sided thoracotomy scar associated with asymmetrical deformity of the
chest. There is reduced chest excursion of the left and the percussion note is dull in the lower third of the left hemithorax with
decreased breath sounds and vocal resonance in this area. The tracheal is not deviated and the apex beat is not displaced.
2. Sir, this patient has a left pneumonectomy as evidenced by a left thoracotomy scar associated with asymmetrical deformity of the
chest. There is reduced chest wall excursion on the left with a dull percussion note and absent breath sounds and vocal resonance
over the entire left hemithorax. Trachea is deviated towards the left with the apex beat displaced in the same direction. This is
associated with hyperinflation of the right chest with hyper-resonant percussion note and loss of liver dullness.
He is comfortable at rest with a RR of 12 bpm with no evidence of respiratory failure or distress.
With regards to aetiology
1. I did not detect any coarse inspiratory crepitations to suggest bronchiectasis nor was there any ronchi or prolonged expiratory
phase to suggest COPD or ABPA.
2. Patient is not clubbed and there is nicotine staining of the fingers. There is no palpable enlarged cervical lymph nodes and he is
not cachexic looking to suggest mitotic lesion of tuberculosis.
I would like to look at the patients temperature chart as well as his sputum.
In summary, this patient has a left lobectomy/pneumonectomy and the possible causes include:
1. surgical resection for early stage mitotic lesion of the lung or a SPN of uncertain cause.
2. Resistant lung abscess
3. Mycetoma
4. Treatment modality for pulmonary tuberculosis in the past (1950s, pt should be late 60s)
5. lung volume reduction surgery in COPD (lobectomy)
6. Localised bronchiectasis or its complications
7. Trauma
Questions
What are the indications for a lobectomy/pnemonectomy? (7)
Causes of lobectomy and pneumonectomy
o Mitotic lesion or SPN of uncertain nature
o Abscess
o Bronchiectasis localised or complications
o Mycetoma, ABPA (remaining lungs may have ronchi)
o TB treatment
o LVRS for COPD
o Trauma
Lobectomy and splenectomy
o TB
o Sarcoidosis
What are the contraindications to a lobectomy/pneumonectomy in a lung cancer patient?
Resectability
o T4 (mediastinal structures), N3 (contralateral mediastinal or hilar or ipsilateral supraclavicular LNs or scalene) or M1
o Tumor within 2 cm of carina (potentially curable by radiotherapy)
o Bilateral endobronchial tumor (potentially curable by radiotherapy)
Physiologic staging
o Severe pulmonary hypertension
o CO2 retention
o FEV1< 1L
o Transfer factor <40%
o Concomitant disease that would shorten life expectancy
o Recent MI in the past 3 months
o Borderline function with cardiopulmonary exercise testing with a maximal oxygen consumption <15ml/kg/min
What are the indications for lung volume reduction surgery in COPD patients?
Emphysema
Predominantly upper lobes
No or mild pulmonary hypertension (PASP > 45 mmHg)
No concomitant disabling disease
FEV1>20%
DLCO >20%

44

21. Approach to Lateral Thoracotomy Scar


Lateral thoracotomy scar
o Asymmetrical chest wall with absent ribs
Thoracoplasty for TB treatment in the past (can get respiratory failure and BiPAP useful)
o Symmetrical or no chest wall deformity
Normal underlying lung on examination
Lung transplant
Pleurectomy (eg for recurrent pntx in Ehlers Danlos)
Bullectomy
Abnormal underlying lung
Reduced breath sounds
o Lobectomy (still as breath sounds in the axilla)
o Pneumonectomy (no breath sounds)
COPD with lung volume reduction surgery
Lung transplant with complications

45

22. Lung Transplant


Presentation 1
Sir, this patient has a bilateral lung transplant as evidenced by presence of a median sternotomy scar associated with normal
underlying respiratory findings. The patient has a wrist tag indicating that he is a lung transplant patient. Chest excursion is normal
with a normal percussion note, vesicular breath sounds and vocal resonance. Trachea is central and apex beat is not displaced.
Hence there are no signs to suggest underlying rejection of the transplanted lung.
Patients respiration rate is 14 bpm and there are no sign of respiratory distress or failure.
Peripheral examination was normal. There was no evidence of nicotine staining, clubbing or short stature.
Patient is not Cushingoid with no steroid purpura or thin skin to suggest chronic steroid use. There is also no hypertrichosis or gum
hypertrophy to suggest cyclosporine adverse effects.
In summary, this patient has bilateral lung transplant and is clinically well currently. The likely underlying causes
a. Young patient Cystic fibrosis, AAT, primary pulmonary hypertension, Eisenmenger (heart-lung transplant)
b. Old patient COPD, IPF and bronchiectasis (always bilateral)
Presentation 2
Sir, this patient has a left lung transplant as evidenced by a left lateral thoracotomy scar with a tag indicating that he is a lung
transplant patient. There is reduced chest excursion bilaterally with normal precussion note. Auscultatory findings include fine late
inspiratory crepitations of the left hemithorax a/w normal vocal resonance, suggesting bronchiolitis obliterans syndrome post
transplant.
Trachea is central and apex beat is not displaced.
There are complications of pulmonary hypertension, cor pulmonale ad polycythaemia.
He is also in respiratory distress with a respiratory rate of 24 bpm with use of accessory muscles of respiration. Clinically he is in
repsiratory failure and is oxygen dependent on INO2 2L/min; there is no central cyanosis and no signs of CO2 retention such as
flapping tremors of the hands or a bounding pulse.
In terms of aetiology:
Examination of the right hemithorax showed presence of ILD/COPD with features of
a. fine late inspiratory crepitations which are Velcro-like, clubbing
b. prolonged expiratory phase with ronch and hyperresonance on percussion of the right hemithorax and loss of liver
dullness, nicotine staining
Steroid toxicity and cyclosporin toxicity.
In summary, this patient has a left lung transplant for an underlying ILD. There are complications of pulmonary hypertension and
cor pulmonale and is clinically in respiratory failure. There also signs to suggest bronchiolitis obliterans syndrome with signs of
chronic steroid usage and cyclosporine usage.

46

Questions
What are the indications for lung transplant?
Cardiopulmonary
o Primary pulmonary hypertension
o Eisenmengers (heart-lung transplant)
Chronic Lung Conditions
o Restrictive pulmonary disease - ILD
o Obstructive pulmonary disease COPD, AAT
o Suppurative Bronchiectasis (must be bilateral tplt), Cystic fibrosis
o Sarcoidosis
What are the contraindications for lung transplant?
Disease specific guidelines
Includes
o Age
o Comorbidities: Absence of concomitant disease that shortens life expectancy (HIV, Hep B)
o Contraindications to surgery/GA (recent MI)
o Smoking, alcoholic, poor social support
What are the complications?
Graft dysfunction (reperfusion edema in the first week)
Airway complications dehiscence, stenosis or bronchomalacia
Rejection
o Acute lymphocytic inflammation, adjust steroid and immune suppression
o Chronic alloimmune inflammatory and non-alloimmune fibroproliferative; bronchiolitis obliterans syndrome
Infection
o CMV ganciclovir
o Aspergillus
o Bacterial pneumonia (Ps aeuroginosa)
How would you manage?
Multidisciplinary, regular follow up, transplant coordinator
3 drug maintenance
o Calcineurin inhibitors (Cyclosporin or tacrolimus)
o Purine synthesis antagonist ( AZA or MMF)
o Steroid
CMV and PCP prophylaxis

47

ABDO!
23. Chronic Liver Disease
Prensentation
Sir, this patient has decompensated chronic liver disease with portal hypertension, splenomegaly and ascites.
My findings include:
Presence of an enlarged spleen that is palpable 3cm from the left costal margin. It is non-tender, firm in consistency, smooth
surface, regular edge, notch border with no splenic rub. I am unable to get above this mass. The liver is not enlarged with a span of
12 cm in the right mid-clavicular line. The kidneys are not ballotable. There is presence of ascites with shifting dullness and this is
not associated with tenderness.
He is deeply jaundice and bruising noted on the ULs and LLs with presence of stigmata of CLD including leukonychia, clubbing,
palmar erythema, spider naevi and gynaecomastia with loss of axillary hair. There is also presence of bilateral edema.
Complications:
He is cooperative with the examination with no flapping tremor to suggest hepatic encephalopathy.
There are no enlarged Cx LNs and patient is not cachexic looking. There is also no conjunctival pallor noted.
Aetiology:
I did not find any parotidomegaly, dupytrens contracture, tattoos, surgical scars or thrombosed veins.
Treatment:
I did not notice any abdominal tap marks but patient has sinus bradycardia, indicating use of beta-blockers.
I would like to complete my examination by looking at the patients temperature chart for fever and a rectal examination for hard
impacted stools or malena.
In summary, this patient has decompensated chronic liver disease with portal hypertension, splenomegaly and ascites. There is
presence of bruising, leukonychia, jaundice with no evidence of hepatic encephalopathy.
A. The most likely etiology for this gentleman is
1. Chronic ethanol ingestion due to presence of parotidomegaly but no dupytrens contracture; presence of hepatomegaly
2. Chronic hepatitis B infection as patient has tattoos.
3. Chronic hepatitis C infection as I note an abdominal surgical scar with possibility of transfusion in the past
B. In the local context, the most likely underlying etiology is chronic ethanol ingestion, chronic hepatitis B and C.
C. The most likely aetiology is chronic ethanol ingestion as I notice that this patient has presence of parotidomegaly. In view that
there is also a hard irregular liver that is palpable, it raises the possibility of an underlying mitotic lesion of the liver.
D. The most likely aetiology is
1. Primary biliary cirrhosis as she is a middle-aged lady with evidence of CLD with pruritus, xanthelasma and generalised
pigmentation.
2. Hemochromatosis as he is a middle-aged gentleman with slate-grey appearance with presence of diabetic dermopathy. I would
like to complete the examination by examining the CVS for CMP, urine dipstick for glycosuria and for small testes secondary to
pituitary dysfunction.
3. Wilsons disease as the patient has a short stature associated with Kayser Fleisher rings of the eyes and tremor and chorea of
the affecting the left upper limb.
4. Haemolytic anaemia (Thalassemia major/intermedius, Hereditary spherocytosis) as the patient has a short stature associated
with hyperpigmentation and thalassemic facies with frontal bossing, flat nasal bridge and maxillary hyperplasia. I would like to
complete the examination by examining the CVS for CMP, urine dipstick for glycosuria and for small testes secondary to pituitary
dysfunction.

48

Questions
What is cirrhosis of the liver?
Defined pathologically
Diffuse liver abnormality
Fibrosis and abnormal regenerating nodules
What are the causes of liver cirrhosis?
Chronic ethanol ingestion
Viral hepatitis B and C
In UK, the risk for hep C is blood transfusion before Sept 1991 or blood products before 1986
Cardiac failure
Others
Autoimmune chronic active hepatitis (female)
Primary biliary cirrhosis (female)
Primary sclerosing cholangitis
Haemochromatosis (male)
Hemolytic disease
Wilsons disease
Alpha 1 AT deficiency
Galactosemia
Type 4 glycogen storage disease
Budd-Chiari (in malignancy- PRV or intraabdominal, AI, OCPs, IBD and PNH)
Drugs MTX (Look for RA or Psoriasis; Bx before starting MTX and bx every 1.5g accumulated dose), amiodarone, isoniazid,
methyldopa (MAMI)
Cryptogenic
What are the complications of cirrhosis? (5)
Portal hypertension
Ascites Tense ascites, SBP
Splenomegaly thrombocytopenia
Varices
Hepatorenal syndrome
Dx
Cr Clr <40
Absence of other causes for renal impairment
Absence of Cr improvement, proteinuria (0.5g/d), hematuria (<50/hpf) and urinary Na <10
Type 1 = rapidly deterioration in renal fn ie doubling of serum Cr in < 2wks to >221 umol/l
Type 2 = stable or slowly progressive that does not mean criteria for type 1
Hepatic encephalopathy
Stages
1 depression, euphoria, sleep disturbance, slurred speech; may have asterixis, normal EEG
2 lethargy, moderate confusion; asterixis present; abnormal EEG
3 marked confusion, arousable; asterixis present; abnormal EEG
4 coma; abnormal EEG
Coagulopathy low platelets and reduced clotting factors
HCC
How do you stage cirrhosis of the liver?
Child-Pugh staging
o Consists of 5 parameters with score ranging from 5 to 15
o Prognosticate
o 5 parameters ( 2 clinical and 3 Ix)
o
Bilirubin (<34, 34-50, >50 umol/l)
o
INR (<1.7, 1.7-2.3, >2.3)
o
Albumin (>35, 28-35, <28)
o
Ascites (mild, moderate, severe)
o
Encephalopathy (absent, I and II, III and IV)
o A 5-6 pts (1 year 100%, 2 year 85%)
o B 7-9 (1 year 80%, 2 year 60%)
o C 10-15 (1 year 45%, 2 year 35%)

49

How would you investigate? (Note the STEM STATEMENT) (5)


Confirming the dx
o
Abdominal USS or CT
Establishing the aetiology
o
Hep markers, CAGE questionnaire, liver Bx in selected cases
Prognosticate
o
LFT Albumin, bilirubin
o
INR
Complications
o Endoscopy of the upper GIT
o Mitotic change USS and AFP
o Evaluation of renal function urea, electrolytes and Cr
o Evaluation of ascitic fluid
Cell count
Ascites albumin (SAAG)
Gram stain and C/S
Others AFB smear and c/s, cytology
Evaluation for liver transplant
o 5 year survival rate for cirrhosis with ascites is 30-40% vs 70-80% for post liver transplant
o MELD score (Model for End Stage liver disease which has bilirubin, creatinine and INR)
o Consider for those with refractory ascites, SBP or HRS
When should an abdominal paracentesis be done for a patient with cirrhosis and ascites?
Newly diagnosed to r/o SBP
Symptomatic fever, abdominal pain, encephalopathy, GI bleed

50

How would you manage? (4)


Education and counselling
o
Stop drinking alcohol, regular follow up
Manage the underlying disease
o
Hepatitis B
General measures (stop alcohol, hep A vaccination)
Lifelong surveillance for HCC with USS and AFP
Antiviral for
Immune clearance phase( HBeAg +, ALT raised)
Reactivation phase ( HBe Ag -, ALT raised, HBV DNA raised)
IFN alpha (SE : influenza-like; neutropenia and thrombocytopenia; neuropsychiatric and unmasking AI
disease)
Lamivudine (well tolerated but YMDD mutant)
o
Hepatitis C
At risk are IVDAs and transfusion pre 1989 (Singapore) or pretransfusion Sept 1991 or blood pdts before 1986
(UK)
General measures
Surveillance (HCC and screen for HIV)
Indications
HCV RNA levels (>50 IU/ml)
Raised ALT
Bx showing fibrosis and inflammation
Treatment
Peg interferon
Ribavirin
o
Alcoholic liver disease
>21u/wk in males and >14u/wk in females
100% of normal liver develops fatty liver
35% develop alcoholic hepatitis
20% develop cirrhosis
40% of alcoholic hepatitis develop cirrhosis
Maddreys discrimination function
PT x Bil x 4.6
>32 = severe
Treat with corticosteroids or total enteral nutrition (20-30 kcal/kg/day)
o
Others (see notes below)
Manage the complications
o
Hepatic encepholpathy
Treat precipitants (see below)
Prevent
Low protein diet
Lactulose
o
Hepatorenal syndrome
Treatment with
Noradrenaline infusion, telipressin or midodrine with octreotide plus
Albumin infusion (1g/kg on D1 then 20-40g/day)
For 5-15 days
Prevention (in patient with cirrhosis and ascites)
IV albumin
NB that hemodialysis does not help in this condition
o
Ascites (see ascites)
o
Upper GI bleed
Secure VS
Urgent endoscopy
Operative
Prevention
Propanolol to reduce HR by 25% or to 55-60 bpm
Variceal banding
o
HCC
Definitive treatment
o
Liver transplant
o
MARS (Molecular adsorbent Recirculating system)dialysis as an interim measure before liver transplant
What are the factors precipitating decompensation?
Infection SBP, pneumonia, UTI
GI bleed
Constipation
Diuretics and electrolyte imbalance
Diarrhea and vomiting
Sedatives
Surgery

51

What are the nail changes of hypoalbuminaemia?


Leukonychia, ie nail bed opacify indicating an albumin level <30g/dL; affecting the thumb and index nails bilaterally initailly
Muehrckes lines transverse white lines
What are the causes of palmar erythema?
CLD
RA, thyrotoxicosis and polycythaemia
Pregnancy, normal finding
What are the causes of anaemia in cirrhotic patients?
Anaemia of chronic disease
Fe deficiency from GI bleed
Hemolysis from hypersplenism
Folate and B12 from poor nutrition
How many spider naevi should be present to be considered as significant?
More than 5

52

When examining a patient with signs of chronic liver disease, think of:
Primary biliary cirrhosis
Clinical
Female middle age
CLD with pruritus, xanthelesma, generalised pigmentation, hepatosplenomegaly
Stages
Asymmptomtic with normal LFTs (positive Abs)
Asymptomatic with abnormal LFTs
Symptomatic lethary and pruritus
Decompensated
Commonly associated with sicca syndrome, arthralgia, Raynauds, Sclerodactyly and Thyroid disease
Ix
Raised ALP, Anti-Mitochondrial Ab M2 Ab, IgM
Lipids
Other tests for CLD
Histology Granulomatous cholangitis
Mx
Symptomatic
Urosdeoxycholic acid
Cholestyramine
Fat soluble vitamins
Immunosuppression Cyclosporin, steroids, AZA, MTX, tacrolimus, colchicines
Liver transplant
Hemochromatosis
Clinical
Male
Slate-grey appearance, hepatomegaly
Affects
Liver cirrhosis and cancer
Pancrease DM
Heart failure (CMP)
Pituitary dysfunction
Pseudogout
Therefore requests: Urine dipstick, CVS examination and testicular examination
Autosomal recessive, HLA-A3, Ch 6 HFE gene, increased Fe absorption with tissue deposition,
Ix
Raised ferrritin, transferrin saturation and liver Bx
Mx
Non-pharmological
Avoid alcohol
Avoid shellfish as they are susceptible to Vibrio vulnificus
Venesection
Dy/Dx of generalised pigmentation
Liver hemochromatosis in males and PBC in females
Addisons
Uremia
Chronic debilitating conditions eg malignancy
Chronic haemolytic anaemia
Wilsons disease
Clinical
Short stature
Eyes
KF rings - greenish yellow to golden brown pigmentation of the limbus of the cornea due to deposition of Cu in Descemets
membrane at 12 and 6 oclock position. Also occurs in PBC and cryptogenic cirrhosis
Sunflower cataract
Extrapyrimidal
Tremor and chorea
Presents as difficulty writing and speaking in school
Pseudogout
Penicillamine complications
Myasthenic ptosis
Lupus malar rash, small hand arthritis
Urinalysis for glycosuria from proximal RTA
Autosomal recessive, Ch 13, increased Cu absorption and tissue deposition
Ix
Low serum ceruloplasmin, increased 24H urinary Cu
Liver Bx increased Cu deposition
Mx
Penicillamine
Ulcerative Colitis
Clinical
o
Skin erythema nodosum, pyoderma gangrenosum
o
Joint arthropathy LL arthritis, AS, sacroilitis
o
Aphthous ulcers
o
Ocular iritis, uveitis and episcleritis
o
CLD Cirrhosis, chronic active hepatitis, fatty liver PSC, Cholangiocarcinoma, metastatic colorectal cancer, amyloid

53

24. Hepatomegaly
Presentation
Sir, this patient has an enlarged liver without any signs of liver cirrhosis. (The mass in the RUQ is a liver mass as I am unable to get
above the mass and am able to trace the edge of the liver past the midline of the abdomen.) It is massively/moderately enlarged
with a
Size cm from the right costal margin with a span of cm
Edge regular or irregular
Surface smooth or nodular
Consistency soft, firm, hard
Tender
Pulsatile
Bruit
The spleen is not palpable or percussible. The kidneys are not enlarged. There is no associated ascites.
Examination of the peripheries:
Jaundice, bruises, stigmata of CLD and edema of LL
Hepatic flap
Causes
o Cachexia, Cx LNs, conjunctival pallor
o Dupytrens contracture, parotidomegaly
o Toxic looking, rashes or injected throat or enlarged tonsils
o CCF presence of raised JVP
I would like to complete the examination by checking patients temperature chart for fever (if infective cause is a differential) and a
rectal examination for masses (if secondaries are a differential).
In summary, this patient has an enlarged liver that is (state the important Cs). Hence my differential diagnosis includes:
Massive
HCC/Secondaries/myeloprolif
RVF
Alcoholic liver disease
Mild-moderate
As above plus
Infection
Viruses EBV, CMV, hepatitis A & B
Bacteria Weils disease (leptospirosis), meliodosis, abscesses, TB, brucellosis, syphilitic gumma
Protozoal hydatid cysts, amoebic abscess
Malignancy lymphoproliferative, myeloproliferative, primary, secondary, adenoma from OCP
Infiltrative sarcoid (erythema nodosum, lupus pernio), amyloid, fatty liver
Endocrine acromegaly, hyperthyroid
Collagen Vascular disease
Chronic hemolytic anaemia( AI, thalassemia, HS)
Reidels lobe
Possibility of minimal CLD signs with just hepatomegaly
PBC
Hemochromatosis
Tender
Liver abscess/infective (viral/bacterial/parasitic)
HCC/Secondaries
Right Heart Failure/Budd chiari
Pulsatile
TR
HCC
AVM
Hard/Irregular
Mitotic (primary/Secondary)
Macronodular cirrhosis ie >3mm (post hepatitis B, C, Wilsons and AAT) (Micronodular cirrhosis implies alcoholic liver cirrhosis)
Amyloidosis/Hydatid cyst/granulomatous disease/gummatous disease/APCKD

54

Questions
What are the causes?
See above
How would you investigate?
According to the most likely etiologies
Think of
o Blood Ix Dx and PX
o Imaging
o Liver Bx
How would you manage?
According to the most likely etiologies
(Dont forget that 40% of CLD has no peripheral stigmata of CLD therefore think of Alcoholic liver cirrhosis, PBC and
Hemochromatosis in the right setting)

55

25. Splenomegaly
Presentation
Sir, this patient has a moderately enlarged spleen without evidence of liver cirrhosis. There is associated with
tenderness/pallor/lymphadenopathy.
The spleen is moderately enlarged at 4 cm from the left costal margin. There is a palpable notch with a regular edge and smooth
surface, firm consistency and is non tender. I did not detect a splenic rub. This is not associated with hepatomegaly or ascites. The
kidneys are also not ballotable.
Peripheral examination showed that there is no evidence of any stigmata of CLD and patient is not jaundiced with no bruises or
petechiae.
Aetiology:
1. Patient is not cachexic looking with no conjunctival pallor or enlarged Cx LNs. There is also no evidence of polycythemia such as
plethoric facies or conjunctival suffusion or bone marrow biopsy scar.
2. Patient is not toxic looking and no rashes or enlarged tonsils are noted.
3. There are no splinter haemorrhages or stigmata of IE.
4. There are no features of SLE or RA or chronic haemolytic anaemia.
I would like to complete the examination by looking at the patients temperature chart and take a history of night sweats, LOW and
travel history
My differential diagnoses for this young patient with moderately enlarged spleen with anaemia are
Massive Splenomegaly (>8 cm)
CML
Myelofibrosis
PRV
Chronic malaria
Kala-azar (visceral leshmaniasis)
Others(Gauchers, rapidly progressive lymphoma)
Moderately Enlarged (4 to 8 cm/ 2-4 FB)
Myeloproliferative
Lymphoproliferative
Haematological AI, ITP, Thalassemia and HS
Chronic malaria
Cirrhosis
Mildly Enlarged(4cm</1-2FB)
Myeloproliferative, Lymphoproliferative
Infections
Viral CMV,EBV
SBE, splenic abscesses, leptospirosis, Meliodosis, TB, Typhoid, Brucellosis(farmer)
Acute malaria
Infiltrative Amyloidosis, Sarcoidosis
Endocrine Acromegaly, thyrotoxicosis
Collagen vascular SLE, Feltys
Chronic haemolytic Thalassemia, AI, HS, ITP
Tender
Infective causes
Acute myeloproliferative and lymphoproliferative
Pallor
Myeloproliferative
Lymphoproliferative
Malaria
Hemolytic anaemia(Thalassemia and AIHA)
AI Feltys, SLE
Cirrhosis of the liver with portal hypertension
Lymph Nodes
Lymphoproliferative(CLL/lymphoma)
Infective(IMS, Meliodosis, CMV, TB, HIV)

56

Questions
What are the causes?
See above
What are the features of an enlarged spleen in contrast to an enlarged left kidney?
Palpation
o Unable to get above it
o Notch border
o Not bimanually palpable or ballotable
th
Percussion note over the mass is dull from the left 9 rib in the mid-axillary line extending inferior-medially in the axis of the
th
10 rib
Inspection shows that the mass moves inferior-medially with inspiration rather than inferiorly
Auscultation may reveal a splenic rub
Is a normal spleen palpable or percussible?
A palpable spleen implies that is at least twice enlarged
th
th
th
A normal spleen can be percussed along the 9 , 10 and 11 rib but is not percussible beyond the anterior axillary line
How would you Ix?
How would you manage?
Notes on conditions (See History Section)

57

26. Hepatosplenomegaly
Presentation
Sir this patient has hepatosplenomegaly without evidence of cirrhosis of the liver.
(I say this because enlarged masses in the right and left hypochondrial regions of which I am unable to get above theses mass es
and is not bimanually palpable or ballotable. Hence, these are unlikely to be due to kidney masses.)
The liver is enlarged
Size, edge, surface, consistency, tender, bruit or pulsatile
The spleen is enlarged
Size, edge, surface, consistency, tender
Kidneys are not enlarged and no associated ascites
Peripheral examination
CLD stigmata, jaundice, bruises
Hepatic encephalopathy
Causes
o Pallor, cachexia, Cx LNs, PRV
o Toxic, rashes, tonsils
o Chronic ethanol ingestion
o CCF
o SBE, SLE, RA, Hemolytic anaemia
I would like to complete the examination
In summary, this patient has hepatosplenomegaly that is associated with. The differential diagnoses are:
(Determine which is the predominantly enlarged organ eg massive liver with small spleen or massively spleen with small liver;
determine if there is any Cs liver findings such as pulsatile liver; if both are mildy enlarged then combine the causes)

58

Massive Splenomegaly (>8 cm)


CML
Myelofibrosis
PRV
Chronic malaria
Kala-azar (visceral leshmaniasis)
Others(Gauchers, rapidly progressive lymphoma)
Moderately Enlarged (4 to 8 cm/ 2-4 FB)
Myeloproliferative
Lymphoproliferative
Haemotological AI, ITP, Thalassemia and HS
Chronic malaria
Cirrhosis
Mildly Enlarged(4cm</1-2FB)
Myeloproliferative, Lymphoproliferative
Infections
Viral CMV,EBV
SBE, splenic abscesses, leptospirosis, Meliodosis, TB, Typhoid, Brucellosis(farmer)
Acute malaria
Infiltrative Amylodosis, Sacoidosis
Endocrine Acromegaly, thyrotoxicosis
Collagen vascular SLE, Feltys
Chronic haemolytic Thalassemia, AI, HS, ITP
Tender spleen:
Infective causes
Acute myeloproliferative and lymphoproliferative
Pallor(same as moderately enlarged spleen)
Myeloproliferative
Lymphoproliferative
Malaria
Hemolytic anaemia(Thal and AIHA)
AI Feltys, SLE
Cirrhosis of the liver with portal

hypertension

59

Lymph Nodes
Lymphoproliferative(CLL/lymphoma)
Infective(IMS, Meliodosis, CMV, TB, HIV)
Massive Liver
HCC/Secondaries/myeloprolif
RVF
Alcoholic liver disease
Mild-moderate Liver
As above plus
Infection
Viruses EBV, CMV, hepatitis A & B
Bacteria Weils disease (leptospirosis), meliodosis, abscesses, TB, brucellosis, syphilitic gumma
Protozoal hydatid cysts, amoebic abscess
Malignancy lymphoproliferative, myeloproliferative, primary, secondary, adenoma from OCP
Infiltrative sarcoid (erythema nodosum, lupus pernio), amyloid, fatty liver
Endocrine acromegaly, hyperthyroid
Collagen Vascular disease
Chronic hemolytic anaemia( AI, thalassemia, HS)
Reidels lobe
Possibility of minimal CLD signs with just hepatomegaly
PBC
Hemochromatosis
Tender Liver
Liver abscess/infective (viral/bacterial/parasitic)
HCC/Secondaries
Right Heart Failure/Budd chiari
Pulsatile Liver: TR, HCC, AVM
Hard/Irregular Liver
Mitotic (primary/Secondary)
Macronodular cirrhosis (post hepatitis B/C, Wilsons and AAT)
Amyloidosis/Hydatid cyst/granulomatous disease/gummatous disease/APCKD

Questions
What are the causes? How would you investigate? How would you manage?

60

27. Ascites
(Think of CLD, Budd-Chiari, renal failure or heart failure, hypothyroidism, also malignancy, TB)
Presentation
Sir, this patient has gross ascites.
There is presence of abdominal distension with an everted umbilicus. There is a positive fluid thrill as well as shifting dullness. This
is not associated with any abdominal tenderness and patient is able to lie flat for the examination. There is also abdominal scar
marks suggesting abdominal tap has been done.
I am unable to palpate the liver and it has a span of 12 cm in the right mid-clavicular line. The spleen is not palpable or percussible.
The kidneys are not ballotable. There are no other masses palpable in the abdomen.
There are no stigmata of chronic liver disease such as leukochynia, clubbing, palmar erythema, spider naevi, gynaecomastia or
loss of axillary hair. There is also no hepatic fetor or a hepatic flap. Patient is not jaundice and there is no conjunctival pallor.
There is associated pedal edema up to the knee level with sacral edema but no periorbital edema. There are no signs of renal
failure such as a sallow appearance or uremic fetor.
Patient also does not have any features to suggest hypothyroidism such as a cream and peaches complexion, macroglossia,
hoarseness of voice or bradycardia.
He is not cachexic looking and there are no palpable cervical LNs. He is not toxic looking.
I would like to complete my examination by
CVS looking at the JVP with the patient seated 45 degrees to look for raised JVP with steep x and y descent, early S3
suggestive of constrictive pericarditis
Urine dipstick for proteinuria
Temperature chart for fever (TB)
Rectal examination for a rectal mass
In summary, this patient has got gross ascites that is not associated with any intra-abdominal organomegaly or masses of which no
apparent cause is found clinically. The possible differential diagnoses include cirrhosis of the liver, Budd-chiari syndrome, nephrotic
syndrome or protein-losing enteropathy, congestive cardiac failure or intra-abdominal malignancy or TB.
Questions
What are the causes of abdominal distention?
Fat, fluid, flatus, faeces, fetus and organ enlargement
What is ascites?
Pathologically accumulation of fluid in the peritoneal cavity
How much fluid must be present before there is flank dullness?
1.5 L of ascitic fluid
How would you approach a patient with ascites clinically?
Abdominal examination
o Liver
Look for jaundice, spleen and stigmata of CLD cirrhosis of liver
Liver palpable and smooth think of Budd-chiari
Liver palpable and hard and nodular think of malignancy
o Kidneys
Look for evidence of kidney failure and anasarca
o Look for congestive cardiac failure or constrictive pericarditis
o Look for features of hypothyroidism
o If all above absent, think of
TB peritonitis
Intra-abdominal malignancy
Carcinomatosis peritonei
Secondaries
o Liver
o Colon
o Ovaries
o Pancreas

61

What are the causes of ascites?


Serum ascites albumin gradient >1.1g/dl = portal hypertension (97% accuracy)
o Cirrhosis of the liver
o Budd-Chiari
o CCF
o Constrictive pericarditis
o Malabsorption
o Meigs syndrome
o Hypothyroidism
Serum ascites albumin gradient< 1.1g/dl
o Intra-abdominal malignancy
o TB
o Nephrotic syndrome
o Protein losing enteropathy
What is the pathophysiology of ascites in cirrhosis of the liver?
The chief factor is splanchnic vasodilatation
Cirrhosis leads to increased resistance to portal flow
Leading to portal hypertension
Portal hypertension results in local production of vasodilators, with splanchnic arterial vasodilatation
(1) Arterial underfilling
o Early stage minimal effect on effective arterial volume as can be compensated by increase in plasma volume and
cardiac output
o Later stage
splanchnic vasodilation so marked that effectve arterial pressure falls and results in activation of vasoconstrictors
and atrial natriuretic factors
Sodium and fluid retention and expansion of plasma volume contributing to ascites
Impaired free water execretion leading to dilutional hyponatraemia
Renal vasoconstriction with hepatorenal syndrome
(2) Increase in splanchnic capillary pressure with lymph formation exceeding return therefore ascites
How would you investigate to determine the cause of the ascites?
(Liver, renal, heart, thyroid, TB)
Ascitic tap
o Cell count, albumin, and total protein concentration if cirrhosis and dx
See attached
o Others
Infection c/s and g/s AFB
Malignancy - cytology
o <0.1% of Cx such as hemperitoneum or bowel perforation
o 1% of abdominal wall hematoma
o 2FB cephalad and medial to the ASIS in the left lower quandrant
Imaging
o USS/CT
Liver cirrhosis, budd-chiari
Renal
o Echo and ECG
o CXR (TB, Pl effusion)
Bloods: LFT, Renal, TFT, FBC
How would you manage a patient with ascites secondary to cirrhosis of the liver?
Treat the underlying cause
Avoid alcohol or medications that are toxic to liver
Management of ascites
o General measures
Salt restriction <2 g/day
Fluid restriction <1l/day (for ascites, edema with Na <130)
o Specific measures
Diuretics (Spironolactone, frusemide initially)
Aim to 0.5kg/day if no peripheral edema
Aim 1kg/day if presence of peripheral edema also
Increase diuretics with spironolactone up to 400mg/d or frusemide 160mg/d
Paracentesis
If >5L then requires albumin administration (8g per L of fluid removed)
TIPSS (Transjugular Intrahepatic portosystemic shunt)
High rate of shunt stenosis; up to 75% at 1 year
o Liver transplant
5 year survival rate for cirrhosis with ascites is 30-40% vs 70-80% for post liver transplant
MELD score (Model for End Stage liver disease which has bilirubin, creatinine and INR)
Consider for those with refractory ascites, SBP or HRS
Manage other complications of cirrhosis
62

How do treat and prevent spontaneous bacterial peritonitis?


Defined as >250 polymorphs per ml of ascitic fluid
Commonly E coli, Klebsiella and pneumococci
Translocation of bacteria from intestinal lumen to LNs then bacteremia
Rule out secondary peritonitis
o Loculated infection or perforated viscus
o Fluid
>1000 polymorphs
LDH > upper limit of serum
Low glucose
High protein >1 g/L
CEA > 5ng/ml
ALP >240u/L
Treatment
rd
o 3 generation cephalosporin
o IV albumin to prevent HRS
Prevention
o Indications
After 1 episode of SBP as recurrence as high as 70%/year
In patients with acute variceal bleed
Ascitic fluid protein concentration<1g/dl (controversial)
o Prophylaxis with ciprofloxacin or norfloxacin
What does development of ascites in a patient with cirrhosis of the liver means?
Decompensation
Occurs in 50% of patients within 10 years of diagnosing compensated cirrhosis
Poor Px
o only 50% survive beyond 2 years
o poor quality of life
o increased risk of infection and renal failure

63

28. Unilateral Enlarged Kidney


1. Sir, this patient has an enlarged left kidney and is on hemodialysis. There is presence of an enlarged left kidney as evidence by a
left flank mass that is bimanually palpable and ballotable with a nodular surface. I am able to get above the mass and there was no
palpable notch. Percussion note is resonant above the mass and it moves inferiorly with respiration. It is non tender and not
associated renal bruit.
2. Sir, this patient has an enlarged right kidney and is on hemodialysis. There is presence of a right flank mass that is bimanually
palpable and ballotable with a nodular surface. I am able to get above this mass and the mass does not cross the midline.
Percussion note is resonant over the mass and it moves inferiorly with respiration. It is non-tender and there is no associated renal
bruit.
There is no ascites. The liver is not enlarged with a span of 12 cm in the right midclavicular line. The spleen is also not enlarged. I
did not notice any scars.
The patient has evidenced of chronic renal failure of which he is receiving hemodialysis. There is presence of a sallow appearance.
I did not notice any pruritic scratch marks or bruises on the ULs or LLs and he is not cachexic looking. There are no signs o f
leukonychia or Terrys nails. There is no conjunctival pallor to suggest anaemia and he does not have features of polycythemia.
Patient is not in fluid overload, has no uremic fetor or flapping tremor of the hands and no Kussmauls breathing.
Patient is undergoing hemodialysis via a left arterio-venous fistula. There is a strong thrill felt over the fistula with recent needle
injection marks. There are no complications of aneurysm of the fistula. There are no abdominal scars to suggest previous TK
insertion or renal transplant.
I would like to complete my examination by
Temperature chart for fever
Blood pressure for hypertension
Fundoscopy for hypertensive changes
(dont mention urine analysis if ESRF on RRT!)
Cardiovascular examination for MVP and AR
Neurological examination for III nerve palsy of stroke
FHx of stroke or aneurysm
In summary, this middle age gentleman has an enlarged left sided kidney with complications of ESRF of which he is undergoing
hemodialysis. The most likely aetiology is asymmetrically enlarged Adult Polycystic Kidney disease.
Question
What are the causes of a unilaterally enlarged kidney?
Causes of bilateral asymmetrical enlargement
o APCKD, Acromegaly, DM, bilateral HN
o Tuberous sclerosis, VHL, Amyloidosis
Unilateral disease
o RCC
o Acute renal vein thrombosis
o Pyonephrosis
o Hypertrophy of a single functioning kidney
See Bilateral enlarged Kidneys for other questions.

64

29. Bilateral Enlarged Kidneys


Presentation
Sir, this patient has bilateral enlarged kidneys. There are bilateral masses in the flanks which are bimanually palpable and
ballotable with a nodular surface. Of note, I am able to get above both masses. Percussion note was resonant over both kidneys
and they move inferiorly with respiration. They are not tender in nature and there was no renal bruit.
There is no associated hepatomegaly and the liver span is 12 cm at the right mid-clavicualar line. The spleen is not enlarged. There
is no ascites detected clinically and the bladder is not palpable or percussible.
The patient does not have a sallow appearance and not cachexic looking. There are no pruritic scratch marks or bruising. There is
also no leukonychia or Terrys nails. There is no conjunctival pallor to suggest anaemia and no features of polycythemia such as a
plethoric facies or conjunctival suffusion. Patient is not in fluid overload as there is no pedal oedema, he is able to lie flat and is not
oxygen dependent. There is no Kussmauls breathing pattern and also no flapping tremor or uremic fetor.
He does not have any acromegalic features, no DM dermopathy and no adenoma sebaceum to suggest tuberous sclerosis.
There is no evidence of renal replacement therapy such as AVF, TK cathether or a transplanted kidney.
I would like to complete my examination by
checking the patients temperature chart for fever,
blood pressure for hypertension
fundoscopy for hypertensive changes
urine disptick for hematuria, proteinura and pyuria
Cardiovascular examination for signs of MVP or AR
Neurological examination for a third nerve palsy secondary to berry aneurysm or any evidence of a stroke
FHx of aneurysm or SAH (5% risk overall but 20% if positive FHx)
In summary, this middle age gentleman has got bilateral enlarged kidneys with no complications of chronic renal failure detected
clinically. There is also no evidence that the patient is undergoing renal replacement therapy. The most underlying etiology is Adult
Polycystic kidney disease.
Questions
What are the causes of bilateral enlarged kidneys?
APCK
Commoner
o Acromegaly (hepatosplenomegaly)
o Early diabetic nephropathy
o Bilateral hydronephrosis
Rare
o Tuberous sclerosis
o Amyloidosis
o Von-Hippel Lindau disease
Autosomal dominant
Multiple angiomata in the retina, CNS
Cysts in liver, kidneys pancreas
RCC, phaeochromocytoma
What are the conditions that can result in bilateral renal cysts?
o Polycystic kidneys
Dominant and recessive
Simple cyst
Von Hippel Lindau
Tuberous sclerosis

65

What are the complications of APCK? (Renal and Extra-renal Cx)


Fever
o UTI, pyelonephritis, pyocyst
Hypertension (75%)
o Activation of RAA from intra-renal ischaemia from architectural distortion
o Malignant hypertension
Renal artery stenosis from compression
Renin producing cyst
Pain
o Chronic pain
o Acute pain
UTI
Nephrolithiasis
Cyst rupture
Haemorrhage into cyst
Upper tract obstruction
Massively enlarged cyst
Clot
Stone
Anaemia
o CRF
o Persistent gross hematuria
Polycythaemia
o Increased erythropoietin production
Malnutrition
o CRF
o Bilateral renal enlargement with early satiety
Acute renal failure
o Malignant hypertension
o UTI
o Nephrolithiasis (Uric acid)
Chronic renal failure
Renal cell carcinoma (rare)
Extra-renal
o Abdominal cysts in liver, spleen, pancreas, ovaries; colonic diverticular disease
o Cardiac MVP(25%), AR, TR
o Intracranial aneurysm (III nerve palsy), SAH (3%)
What are the complications of CRF?
Fluid
Electrolytes Hyperkalaemia
Acid-base Metabolic acidosis
Uremia and its complications
Hypertension
Anaemia (NCNC)
Secondary and tertiary hyperparathyroidism
Renal bone disease
Why are patients with CRF sallow?
Impaired execretion of urinary pigments combined with anaemia
What are the types of signs in the nails that you can detect in patients with CRF?
Hypoalbuminaemia
o Leukonychia
o Muehrckes nails (paired white transverse line near the distal end of nails)
Renal failure
o Terrys nails (distal brown arc 1mm or >)
o Mees line (single white line; also in arsenic poisoning)
o Beaus line (non-pigmented indented band = catabolic state)
What are the causes of anaemia in patients with CRF?
Erythropoeitin deficiency
Anaemia of chronic disease
Fe deficiency anaemia blood loss, nutrition
Folate deficiency nutrition

66

What is Adults Polycystic Kidney disease?


Multisystemic, progressive disease, 1 in 400 to 1 in 1000 people
Characterised by cysts formation and enlargement in the kidneys and other organs
Autosomal dominant with almost 100% penetrance
Focal cystic dilatation of the renal tubules
2 predominant type
o 85% - APCKD 1 on Ch 16
o 15% - APCKD 2 on Ch 4
rd
o 3 type of which loci is not fully known
rd
th
Presents clinically in the 3 or 4 decades with
o Hematuria, hypertension, recurrent UTI, pain and uremia
o Stroke
By age 60 years, 50% will require RRT
Poorer Px males, PCK 1 and early onset of clinical features
Mortality
o ESRF (1/3)
o Stroke and other hypertensive Cx (1/3/)
o Others
How do you investigate?
Blood Tests
o FBC
o Biochemical
o CRF Ca, PO4, iPTH, Uric acid, urinalysis
USS (useful >20 years old); Ravines criteria
o At risk patients, 20-30yrs: 2 cysts in 1 kidney or 1 cyst in each kidney
o At risk patients, 30-60 yrs: 2 cysts in each kidney
o At risk patients, >60 yrs: 4 cysts in each kidney
Other imaging(CT and MRI)
MRA for patients with high risk of an aneurysm, Ba enema and Echocardiogram
Genetic testing
o For young people with no cysts on USS who are potential organ donors
How would you manage?
Education and counselling, regular follow up, screening of first degree relatives
Avoidance of medications that can precipitate renal impairment such as NSAIDs or tetracycline antibiotics
Medical treatment
o Hypertension with ACE inhibitors or ATII RA
o UTI, cysts infection usually GN bacteria therefore use Bactrim or fluroquinolones with good renal tissue penetration
o Pain treatment
o Renal failure medical treatment and RRT for those with ESRF
o Antibiotic prophylaxis
Surgical treatment
o Pyocyst drainage
o Cystectomy
o Nephrectomy
o Alcohol sclerosant
o RRT
o Aneurysm clipping, MVP with MR

67

30. Transplanted Kidney


Presentation
Sir, this patient has a transplanted kidney in the right iliac fossa associated with bilateral enlarged kidneys with a functioning AVF
with features of cyclosporine and chronic steroid use.
There is presence of a rounded palpable mass in the right iliac fossa with an overlying scar. It is non-tender. In addition there are
bilateral masses in the flanks which are bimanually palpable and ballotable with a nodular surface. I am able to get above these
masses and they are not tender. They move inferiorly with respiration and percussion note is resonant over them. There is no
associated ascites and no renal bruit.
The liver and spleen are both not enlarged.
The patient does have features of renal impairment with a sallow appearance and is thin looking. He does not have any bruises or
pruritic scratch marks and no leukonychia or Terrys nails were detected. There is also no conjuctival pallor to suggest anaemia and
there are also no features of polycythemia such as a plethoric facies or conjunctival effusion. He is also not in fluid overload with no
pedal edema and is able to lie flat and is not oxygen dependent. There is no Kussmauls breathing with no uremic fetor or flapping
tremor of the hands.
There is presence of an arterio-venous fistula in the right upper limb. It is functioning with a good thrill. There are no recent needle
puncture marks and no aneurysm was noted.
There is presence of diabetic dermopathy noted on the lower limbs.
There is no evidence of transplant related hepatitis B or C with no jaundice or stigmata of chronic liver disease.
Patient has hypertrichosis and gum hypertrophy which are complications of cyclosporine usage. Moreover, he has a Cushingnoid
habitus with steroid purpura and thin skin, suggesting chronic steroid usage.
I would like to complete my examination:
Temperature chart for fever
BP for hypertension
Fundoscopy for hypertensive changes
Urine analysis for hematuria, proteinura or pyuria
CVM MVP or AR
Neurological III nerve palsy or PHx of stroke
In summary, this middle age gentleman has a transplanted kidney for underlying Adult Polycystic kidney disease with previous
dialysis. The graft is functioning well as he is not uremic and is well with features of cyclosporin and steroid use.
Questions
What are the differential diagnoses for a right iliac fossa mass?
Transplanted kidneys
Carcinoma of the caecum (hard mass, LNs)
Abscess appendicular, ileocecal
Crohns disease (mouth ulcers, PR for fistulas)
Ovarian tumors (in females)
Others
o Amoebiasis, TB lymphadenitis, actinomycosis
o Carcinoid
o Ectopic kidney
What are your differential diagnoses for a left iliac fossa mass?
Transplanted kidney
Colonic carcinoma (hard mass, hepatomegaly LNs)
Diverticular abscess
Fecal mass
Ovarian tumors
Others lymphadenitis
What are the common kidney diseases leading to transplant?
DM
Hypertension
GN
How does renal transplant compare with dialysis?
Higher patient survival rates
Better quality of life with lower hospitalisation rates

68

What are the causes of transplant loss?


Patient death
Allograft failure
o Immunological
Acute rejection
Single most important event determining graft survival
Can result in rapid loss of graft or progression to chronic rejection or chronic allograft nephropathy
Treated with pulse steroid or anti-lymphocyte antibody therapy
Chronic rejection
o Non-immunological
Renovascular thrombosis
Ischaemia reperfusion injury
Nephrotoxicity from calcineurin inhibitors
CMV, polyoma virus
DM, hypertension, hyperlipdaemia
o Others
Recurrence of primary disease (GN and DM)
Chronic allograft nephropathy
What is delayed graft function?
Defined as requirement of dialysis in the first week post transplant
o Immunological acute rejection
o Non-immunological ischaemia reperfusion injury, donor hypertension
What are the strategies one can use to reduce graft loss?
Immunological
o Live donor better than cadaveric
o HLA matched at A, B and DR loci
o Absence of pre-sensitisation
Previous transplant
Pregnancies
Transfusions
Idiopathic
o Immunosuppresive therapy to reduce acute rejection
Traditionally use of steroid and cyclosporin
Others
Calcineurin inhibitors eg Cyclosporin and tacrolimus
Mycophenolate mofetil
Sirolimus
Non-immunological
o Pre-transplant
Donor factors old age, CVA, hypertension
Recepient factors older, male, obese, diabetic, hypertension
o Technical factors
increase cold ischemia time LD transplant, renoprotective preservative solutions
hyperfiltration from inadequate nephron dose match size and better if male to female; use of ACE inhibitors
o Post-transplant
Calcineurin inhibitors induced nephrotoxicity
Monitor levels
Use others such as sirolimus or MMF
CMV infections and polyoma virus
Prophylaxis with ganciclovir for CMV
No Rx for polyoma virus
Treat BP (<130/80) and hyperlipidaemia and DM
What are the complications of cyclosporine?
Hirsutism/hypertrichosis
Hypertrophy of the gums
Hypertension
Hyperkalaemia, hyperuricaemia, hypercholesterolaemia, hypomagnesemia
Hepatotoxicity
Hemolytic uremic syndrome
Hiccuping (gastroparesis)
Hole-in-bones (osteoporosis)
Nephrotoxicity
Neoplasia (lymphoproliferative)
Neurological (tremors, headaches, seizures and strokes)

69

What are the complications of chronic steroid use?


Skin thin skin, telangiectasia, steroid purpura
Cushingoid habitus
Osteoporosis, AVN femoral head
Peptic ulcer disease
Hypertension
Diabetes mellitus
Cataracts
Steroid psychosis
How do you manage?
Education and counselling, regular follow up, compliant
Treat underlying cause
Require preparation prior to transplant
Post transplant management to reduce graft loss (See above)

70

NEURO!
31. Approach to Examination of the Face
(I) Assessment of Higher Cortical Function
(II) Approach to the examination of the Eyes
(III) Cranial Nerves
Isolated CN abnormalities
Multiple CN abnormalities
Conforming
Cavernous sinus syndrome
Superior Orbital syndrome
Cerebellar Pontine Angle
Syringobulbia
Lateral Medullary syndrome
Medial Medullary syndrome
Bulbar Palsy
Pseudobulbar Palsy
Jugular Foramen syndrome
Non-conforming
Brainstem Stroke
Multiple sclerosis
Base of skull metastasis/leptomeningeal metastasis/NPC
Basal meningitis
Pagets disease
Myasthenia Gravis
Miller Fisher Syndrome
Guillain Barre Syndrome
Mononeuritis multiple
Migraines (Paralytic)
(IV) Speech
Dysarthria
Cerebellar Speech
Bulbar Palsy
Pseudobulbar Palsy
Slurred Speech (VII )
Parkinsons Syndrome
Dysphasia
Expressive
Receptive
Nominal
Conductive
Global
Dysphonia
Recurrent Laryngeal Nerve Abnormality
VC abnormalities

71

32. Cranial Nerve Syndromes


Conforming
Superior Orbital syndrome (see VI nerve palsy)
Cavernous sinus syndrome (see VI nerve palsy)
Cerebellar Pontine Angle syndrome
o Involvement
V1-3 (tinnitus and deafness earliest symptom then vertigo; loss of corneal reflex is the earliest sign)
VI
VII
VIII
IX
Cerebellar
o Causes
Tumor
Primary
o Acoustic neuromas schwannomas of the vestibular
o Meningiomas, haemangioblastomsa, medulloblastomas
o Choleastoma
Secondaries
o NPC ( loss of corneal reflex and V2 early)
o Lymphoma
Aneurysm
For bilateral lesions
Bilateral acoustic neuromas in NF type 2
o Examination
Examine CNs
ULs for cerebellar signs
Proceed to check neck for LNs
Look for NF features (caf au lait spots, neurofibroma, freckling and Lisch nodules)
o Presentation
Sir, this patient has righ/left CPA lesion as evidenced by
There is no enlarged Cx LNs to suggests secondaries
There is also no evidence of NF
Possible etiologies includes
o Questions
What is the CPA?
Shallow trangular fossa lying between the cerebellum, lateral pons and the petrous temporal bone
Histology?
Schawannoma
Ix?
Imaging CT/MRI/Angio
Audiography
ENT to exclude NPC
Mx
Microsurgical resection
Stereotaxic radiosurgery (Cx rate same as surgery)

72

Lateral medullary syndrome


o 5 vessels involved (wedge shaped infarction of the lateral aspect of the medulla and the inferior surface of the
cerebellum)
PICA
Vertebral artery (most common artery that is involved)
Lateral medullary artery (superior, middle, inferior)
o Areas affected
Descending sympathetic fibres
Ipsilateral Horners syndrome
Ptosis, meiosis and anhidrosis
Spinothalamic tract
Contralateral hemi-sensory loss of pain and temperature
Descending tract and nucleus of V
Ipsilateral loss of pain and temperature of the face
Nucleus ambiguus(X) and IX
Hoarsenss of voice, dysphagia, hiccups
Vestibular nuclei
Nystagmus, vertigo, nausea
Cerebellar (restiform body of the inferior cerebellar peduncle)
Ipsilateral ataxia and gait ataxia
o Examination
CN examination
Go to ULs for loss of pain and temperature and cerebellar
Check for AF and DM dermopathy
Visual Fields for homonymous hemianopia (posterior circulation)
o Sir this patient has right/left LMS as evidenced by
State the findings
Mention NG
Aetiology infarction affecting the vertebral artery or the PICA, LMA
Did not find any xanthelesma or DM dermopathy, or AF
Request for BP and asking patient on symptoms of dysphagia
Medial medullary syndrome
o Triad of XII, medial lemniscus and pyrimidal tract
o Ipsilateral wasted tongue, contralateral loss of vibration and propioception and contralateral hemiparesis respectively
o Either vertebral artery or lower basilar
Bulbar palsy
o Bilateral involvement of LMN IX, X, XI and XII
o Examination
Proceed with CN
Do Jaw jerk
Requests to examine speech, and gag reflex
Requests to examine ULs for fasciculations and dissociated sensory loss
o Presentation
Patient has bulbar palsy as evidenced by weakness of the soft palate, wasted tongue with fasciculations a/w
a nasal voice and a normal or absent jaw jerk
o Causes (MGS, NNNP)
MND
GBS
Syringomyelia
Poliomyelitis, NPC, neurosyphilis and neurosarcoid
Pseudobulbar palsy
o Bilateral UMN lesions of the IX, X and XII, V and VII (III/IV and VI are spared)
o Examination
Proceed with CN
Do jaw jerk
Request for speech, gag reflex and enquire emotional lability
Request for AF, DM dermopathy and xanthelesma
Requests for ULs to look for UMNs
o Presentation
Patient has PBP as evidenced by presence of sluggish palatal movement, small, stiff and spastic tongue a/w
brisk jaw jerk with Donald duck speech (slow, thick and indistinct)
No AF, DM or xanthelasma
No evidence of mix UMN and LMN signs to suggest MND
No RAPD or INO to suggest multiple sclerosis
Possible causes (BMM)
Bilateral stroke
MND
Multiple sclerosis
73

Syringobulbia
o See syringomyelia
o Extension of syrinx to involve the brainstem
o V(descending tract of V), VII, IX, X, XI, XII and Horners syndrome
o Usually unilateral
Jugular foramen syndrome
o Involvement of the IX, X, XI (XII maybe affected due to proximity)
o Unilateral
o Examination
CN exams
Proceed to check for enlarged Cx LNs
And request to assess speech for husky voice and bovine cough
o Presentation
Sir, this patient has right/left JFS as evidenced by
Notice that this patient is on NG
No enlarged Cx LNs
Possible etiologies includes
o Questions
Causes
Ca of the pharynx (commonest cause), tumor, neurofibroma
Basal meningitis
Pagets disease, trauma
Thrombosis of the jugular vein
IX, X and XI leaves the skull via jugular foramen (between the lateral part of occiput and the petrous part of
the temporal bone)
XII leaves via the anterior condylar foramen
Isolated XI implies injury to XI in the neck eg stab wounds
Non-Conforming
Myasthenia Gravis (see Myasthenia Gravis)
Miller Fisher Syndrome
o Variant of Guillain Barre syndrome
o Characterise by triad of ophthalmoplegia, ataxia and areflexia
o Cs by anti G1Qb antibodies
o Rare
o Good prognosis with recovery beginning within 1 month of onset and complete recovery within 6 months
o Some maybe left with residual weakness and 3% will have relapses
Guillain-Barre syndrome
Mononeuritis multiplex
Migraine (paralytic)
Pagets
Base of skull (trauma)
Basal meningitis
Brainstem strokes or multiple sclerosis

74

33. Isolated Third Nerve Palsy


Examination
Complete the examination routine for eyes or CN as instructed
Proceed to look for intortion of the affected orbit by tilting the head towards the involved site or looking for intortion when asking
patient to look down and medially of the affected eye; patient maybe tilting his head voluntary away from the side of the lesion
th
(implies 4 nerve palsy)
Rule out
Thyroid, MG
Superior orbital syndrome and Cavenous sinus syndrome
Proceed with
Neck for LNs
Examine the upper limbs for Cerebellar, hemiplegia, EPSE and areflexia
Look dor DM dermopathy
Request
Corneal reflex (reduced or absent)
Visual fields (bitemporal hemianopia)
Fundoscopy for optic atrophy (MS), DM or hypertensive changes
Visual acuity
Blood pressure
Urine dipstick
Temperature chart
Headache or pain
Presentation
Sir, this patient has an isolated right third nerve palsy as evidenced by presence of
Divergent strabismus involving the right orbit which is in a down and out position
Complete ptosis/partial ptosis of the right eye
Dilated pupil which is not reactive to direct light and to accommodation
There is no ptosis or superior rectus palsy of the left eye to suggest a III nerve nuclear lesion.
There are no associated CN palsies to suggest superior orbital fissure syndrome or cavernous sinus syndrome. I did not find any
th
th
associated 4 CN palsy with presence of intortion on asking the patient to adduct the right eye and look downwards. The 6 CN is
th
also intact. There is also no paraesthesia of the ophthalmic division of the 5 CN. Gross VA is also intact.
There are no signs of Graves ophthalmopathy (no conjunctival suffusion and proptosis or lid edema of the right eye)
There is no evidence of fatiguiability to suggest myasthenia gravis.
On examination of the neck, I did not find any enlarged cervical LNs. There is also no evidence of hemparesis, cerebellar signs,
areflexia or tremors or chorea on examination of the upper limbs. I also did not notice any diabetic dermopathy.
I would like to complete the examination by:
Corneal reflex (reduced or absent)
Visual fields (bitemporal hemianopia)
Fundoscopy for optic atrophy (MS), DM or hypertensive changes
Visual acuity
Blood pressure
Urine dipstick
Temperature chart
Headache or pain
In summary, this patient has an isolated right third nerve palsy. The possible causes include
Questions
rd
What is the course and anatomy of the 3 CN?
Nuclear portion at the midbrain
Fascicular intraparenchymal portion close to the red nucleus, emerges from cerebral peduncle
Fascicular subarachnoid portion meninges, PCA aneurysm(between the PCA and internal carotid)
Fascicular cavernous sinus portion sella turcica between the petroclinoid ligament below and interclinoid above
Fascicular orbital portion superior orbital fissure
rd

Axons run ipsilateral except those to the (1)superior rectus which is innervated from the contralateral 3 nucleus and (2) the levator
palpebrae which has innervations from both nuclei.
rd

Hence, right sided 3 nerve palsy can have contralateral ptosis which is often milder than the ipsilateral ptosis; also the ipsilateral
superior rectus can still be affected due to involvement of the contralateral fascicular intraparenchymal midbrain portion of the left
rd
3 nerve.
For pupillary reflex and accommodation, it is served by the Edinger-Westphal nucleus and all axons are ipsilateral.
75

rd

What are the causes of an isolated 3 nerve palsy?


Brainstem
Infarct, haemorrhage, tumour, abscess, multiple sclerosis
For nuclear lesions
Will also have contralateral ptosis and elevation palsy
rd
May have bilateral 3 nerve palsies (+/- INO)
For fascicular midbrain lesions
Webers (+ contralateral hemiplegia) base of midbrain
Northnagel (+ contralateral cerebellar) tectum of midbrain
Benedikts (+ contralateral hemiplegia, contralateral cerebellar and contralateral tremor, athetosis and chorea)
tegmentum of midbrain, red nucleus
Peripheral
Subarachnoid portion- PCA aneurysm, meningitis, infiltrative, others eg sarcoidosis
Cavernous sinus lesions- Tumour(pituitary adenoma, meningioma, cranipharyngioma), cavernous sinus thrombosis,
inflammatory (Tolosa-Hunt syndrome which is a non-caseating granulomatous or non-granulomatous inflammation within
cavernous sinus or superior orbital fissure that is treated with steroids) and ischaemia from microvascular disease
affecting the vasa nervosa, mononeuritis multiplex
Orbital- tumor (meningioma, hemangioma), endocrine (thyroid) and inflammatory(orbital inflammatory pseudotumor ie
Tolosa Hunt)
Mononeuritis multiplex, Miller Fischer and MG
Dont forget migraines and myasthenia! (emergency Coning, Giant cell Arteritis and aneurysm)
How would patient present?
Diplopia
Ptosis
Symptomatic glare from failure of constriction of pupil
Blurring of vision on attempt to focus of near objects due to loss of accomodation
Pain in certain etiologies
Diabetes mellitus
Tolosa-Hunt syndrome
PCA aneurysm
Migraine
What are the causes of a dilated pupil?
III nerve palsy
Optic atrophy (direct light and accommodation absent with intact consensual reflex)
Holmes Adie Pupil (Myotonic pupil)
o Unilateral
o Slow reaction to bright light and incomplete constriction to convergence
o Young women
o Reduced or absent reflexes
Mydiatric eye drops
Sympathetic overactivity
Why does a PCA aneurysm result in pupillary involvement whereas conditions such as DM or HYPT spare the pupil?
The pupillary fibres are situated superficially and prone to compression whereas ischaemic lesions tends to affect the core of
the nerve thus sparing the pupillary fibres
How would you investigate?
Imaging
o CT, MRI
o Angiogram
Blood test
Fasting blood glucose, ESR
TFT and edrophonium
LP
How would you manage?
rd
Medical 3 nerve palsy
Education watchful waiting and avoid driving, heavy machinery and climbing high places
Treat underlying conditions such as DM and hypertension
Watchful waiting
Spontaneously recover within 8 weeks
Symptomatic treatment
NSAIDs for pain
If complete ptosis, no need to treat diplopia
Use eye patch for severe diplopia and a prism Fresnel paste on for mild diplopia
rd
Surgical 3 nerve palsy surgery

76

34. Isolated VI Nerve Palsy


Examination (Examine patients eyes, CNs)
Complete the eye examination
Rule out MG, thyroid
Rule out Cavernous Sinus syndrome, Superior orbital fissure syndrome
Proceed to check cranial nerves
V, VII and VIII for CPA tumor
XI, X, XI, XII for any base of skull lesions
Examine upper limbs
Hemiplegia (long tract signs suggesting brainstem)
Cerebellar signs (CPA lesion, Miller Fisher usually truncal and gait)
Reflexes for areflexia (Miller Fisher)
Examine the neck for LNs and mastoid for tenderness (Gradenigos syndrome)
Request
Fundoscopy (papilloedema from raised ICP, optic atrophy for MS)
Field testing (bitemporal hemianopia)
Acuity (reduced in orbital lesions)
Corneal testing for reduced sensation form V1 involvement
BP
Urine dipstick
Temperature chart for fever(meningitis)
Ask for retrobulbar pain
Presentation
Sir, this patient has an isolated right sided VI nerve palsy as evidenced by
Convergent strabismus at primary gaze
Failure of abduction of the right eye
With diplopia where the image is side by side and furthest apart on rightward gaze, with disappearance of the outer image on
covering the right eye. This suggests a right lateral rectus muscle weakness and hence a right VI nerve palsy.
There is no evidence of a III or IV or V1 palsy which may suggest cavernous sinus or superior orbital syndrome.
There is also no involvement of the VII or VIII nerve palsy and no cerebellar signs to suggest A CPA lesion
There are no associated CN lesions of IX to XII and no enlarged Cx LNs.
There is no fatiguibilty to suggest MG and no thyroid eye signs.
There are no enlarged LNs or tender mastoid. There is also no hemiparesis, no cerebellar signs and reflexes are present.
I would like to complete my examination by:
Fundoscopy (papilloedema from raised ICP, optic atrophy for MS, DM or hypt changes)
Field testing (bitemporal hemianopia)
Acuity (reduced in orbital lesions)
Corneal testing for reduced sensation form V1 involvement
BP
Urine dipstick
Temperature chart for fever(meningitis)
Ask for retrobular pain
In summary, this patient has an isolated right sided VI nerve palsy. The aetiology includes.
Questions
What is the course of the VI nerve?
Nucleus of CN VI located in the pons, sending motor neurons supplying the lateral rectus muscle of the ipsilateral eye;
also internuclear neurons that project (via MLF) to medial rectus subdivision of contralateral oculomotor (CN III) nucleus (ie
lesion of nucleus causes ipsilateral horizontal gaze palsy)
nerve fascicles exit the pons anteriorly and course through subarachnoid space where it runs a vertical (upward) course, along
ventral surface of pons (confined by adjacent AICA)
pierces dura overlying clivus, entering into Dorello's canal, contacting tip of petrous pyramid (part of temporal bone; adjacent to
mastoid air cells)
enters the cavernous sinus after passing through the petroclinoid (Gruber's) ligament
runs in body of sinus (rather than lateral wall where CN III, IV, V located) along foramen lacerum (near internal carotid artery)
enters orbit via superior orbital fissure and shortly thereafter pierces lateral rectus muscle

77

What are the causes of a unilateral VI nerve palsy?


Brainstem (Pons) infarct, haemorrhage, abscess, demyelinating (look for VI and VII palsies due to close proximity)
Aneurysm (ectatic basilar artery)
Meningitis
Infective TB, Fungal, HIV, Syphilis, Lyme
Mitotic leptomeningeal carcinaomatosis, secondaries(NPC), lymphoma, radiotherapy
Sarcoidosis
Trauma
CPA lesions
Petrous temporal bone (Gradenigos syndrome)
Cavernous sinus syndrome
Superior orbital syndrome
Miller Fisher syndrome
Mononeuritis multiplex
DM, hypertension
MG
Raised ICP
NB: - most common aetiology of isolated abducens lesion is post-viral in children and ischemia / infarction (eg diabetic) in adults
What are the causes of bilateral VI nerve palsy?
Leptomeningeal causes (see above)
Miller Fisher syndrome
Mononeuritis multiplex
MG
Raised ICP
Wernickes encephalopathy (Ophthalmoplegia, confusion and ataxia a/w Korsakoffs psychosis from thiamine deficiency)
What are the syndromes associated with VI nerve palsy?
Central
Raymonds syndrome ipsilateral VI with contralateral hemiparesis
Millard-Gubler syndrome Ipsilateral VI and VII with contralateral hemiparesis
Peripheral
Gradenigo syndrome inflammation of the tip of the petrous bone from mastoiditis; VI and V(gasserian ganglion therefore
ipsilateral pain) and VII
CPA
Cavernous sinus
Superior orbital
Congenital
Mobius syndrome (VI with facial diplegia)
Duanes syndrome (Congenital absence of VI nuclei with III nuclei innervating the lateral recti; orbit retraction on adduction and
protrusion on abducting
How do you evaluate diplopia?
Evaluation of diplopia follows 3 rules
o The double vision is maximal in the direction of gaze in the affected muscle
o The false image is the outer image
o The false image arises form the affected eye
What is Cavernous sinus syndrome?
Lesion in the cavernous sinus leading to
Ophthalmoplegia, V1, Horners, proptosis, chemosis and pain
Causes include carotid aneurysm, carotid-cavernous fistula, tumour and thrombosis, Tolosa Hunt syndrome
What is superior orbital fissure syndrome?
Lesions occurring at the superior orbital fissure leading to
Ophthalmoplegia, V1, Horners, proptosis, chemosis, pain and Optic Nerve
Causes includes meningiomas, hemangiomas and thyroid eye disease, Tolosa Hunt syndrome
What is Miller Fisher syndrome?
Triad of ophthalmoplegia, ataxia and areflexia
Cs by present of anti GQ1B antibodies
Variant of GBS
What are the causes of mononeuritis multiplex?
Endocrine DM, hypt
AI Churg Strauss, Wegeners, PAN, Sjogrens, RA, SLE
Infective Lyme, leprosy
Infiltrative Amyloid, sarcoid

78

How would you investigate?


Blood tests
fasting glucose, ESR, ANA, VDRL
Imaging
CT brain or MRI brain
Vascular imaging if proptosis / chemosis looking for dural carotid-cavernous fistula or cavernous sinus thrombosis
Examine CSF (also if systemic illness, immunocompr, bilateral, other CN deficits)
8-30% will remain cryptogenic after work-up
How would you manage?
Education on Px if ischaemic (see below) and Ix and avoid driving, climbing high places and operating heavy machinery
Treat underlying cause
Example control DM and BP
Symptomatic treatment
Patch
Prism
If chronic, chemodenervation with botox or strabismus surgery
What is the prognosis?
most either spontaneously improve or have underlying lesion found
ischemic palsies almost always recover completely in 2-4 months while some recovery seen in half of traumatic cases (but
take over one year)
if no recovery over 3-6 months then suspect underlying lesion such as tumour
few may have chronic isolated abducens palsy of unknown cause
follow regularly looking for emergency of new localizing signs and ensure adequate neuroimaging and ENT evaluation

79

35. VII Nerve Palsy


Examination
Upon noticing facial asymmetry, proceed to tests VII nerve functions
Look up (frontalis) and attempt to push the folds down, close your eyes (orbicularis oculi) and attempt to force them open,
and frown (corrugator superficialis)
Look for exposure keratitis, tarsorraphy
Nasolabial fold, show your teeth and and blow against closed lips
Look for drooling of saliva
Determine UMN or LMN, unilateral or bilateral
UMN unilateral
Examine UL and look for hemiparesis on the same side of the facial weakness
Check for xanthelesma, DM signs and BP
LMN unilateral
Examine other CN
VI nerve and contralateral weakness in brainstem lesions
CPA lesion (V, VI, VII and VIII with cerebellar)
Other CN nerves involvement non-conforming type
Basal meningitis lesions
Mononeuritis multiplex, MG
Therefore proceed to examine the neck
Look at the Palate for vesicles
Examine the parotids and for surgical scars
Mastoid tenderness
Examine the neck for cervical LNs
Upper limbs
Contralateral hemiparesis
Ipsilateral cerebellar
Ask to examine
Otoscopy for vesicles in EC and otitis media
For hyperacusis (sensitive to high-pitched or loud sounds)
For loss of taste in the anterior two-thirds of the tongue
Urine dipstick for glucose and BP
Upon noticing facial diplegia, proceed with
Rule out MG (Bilateral ptosis)
Rule out Dystrophia myotonica or fascio-scapular-humeral dystrophy
Bilateral LMN VII
Test for frontalis, corrugator and orbicularis oculi
Ask patient to show teeth and blow against closed lips
Look for V, VI, VIII
Examine parotids (Sarcoidosis, amyloidosis)
Examine tongue (scrotal tongue for MR syndrome)
Examine ULs for GBS, MND, leprosy, Lymes (radiculopathy) & bilat cerebellar signs if suggestive of bilateral CPA tumors
Think of rare: Melkersson-Rosenthal syndrome, Mobius syndrome
Presentation
Sir, this patient has got a right sided lower motor neurone facial nerve palsy as evidenced by:
Paralysis of both the upper and lower facial muscles on the right
Loss of wrinkling of the right side of the forehead, inability to fully close his right eye shut with Bells phenemenon
Associated with loss of the right nasolabial fold and drooping of the right angle of the mouth
There are no cxs of exposure keratitis, no drooling of saliva. I also did not notice any evidence of a right sided tarsorraphy
There is no associated VI nerve palsy to suggest a brainstem lesion.
There are also no features of Cerebellopontine angle lesion with no involvement of the V, VI, VIII or cerebellar signs on the right.
I did not find any evidence of a parotid swelling or a surgical scar and there are no vesicles on the palate. There was no ri ght sided
facial oedema or plication of the tongue to suggest the rare syndrome of MR syndrome.
There is no mastoid tenderness and no enlarged cervical LNs. I did not detect any contralateral hemiparesis or cerebellar signs.
I would like to complete my examination by
Otoscopy for vesicles in EC and otitis media
Ask about hyperacusis (sensitive to high-pitched or loud sounds)
Ask for loss of taste in the anterior two-thirds of the tongue
Urine dipstick for glucose and BP (mononeuritis multiplex)
The most likely cause for this patient would be a right sided Bells palsy.
80

Questions
What is the course of the facial nerve?
VII nerve nucleus lies in the pons in close proximity with VI nerve nuclei
VII leave the pons with VIII via the cerebellopontine angle
It enters the facial canal and enlarges to become the geniculate ganglion
A branch is given off to the stapedius muscle and the greater superficial petrosal branch goes to the lacrimal glands
The chorda tympani which supplies taste sensation to the anterior two thirds of the tongue joins the VII nerve in the facial canal
VII nerve exits the skull via the stylomastoid foramen, through the parotids with the following branches
Temporalis
Zygomatic
Buccal
Mandibular
Cervical
What are the causes of a unilateral LMN VII nerve palsy?
Brainstem (Infarct/haemorrhage, MS, abscess and tumour, syringobulbia)
Base of skull lesions (infective, tumour, infiltrative)
CPA lesions (acoustic neuroma, meningioma, neurofibroma)
Petrous temporal bone (Bells palsy, Ramsay Hunt, OM)
Parotid (tumour, sarcoidosis, surgery)
Mononeuritis multiplex
NB: Most common is Bells palsy
What are the causes of bilateral LMN VII nerve palsies?
After ruling out MG and myopathies
Bilateral CPA tumor as in NF type 2
Bilateral Bells palsy
Bilateral Parotid enlargement (Sarcoidosis Uvoeoparotid fever or Heerfordts fever)
GBS, MND and leprosy, Lyme disease
Rare: Rosenthal Melkersson syndrome (triad of VII palsy with facial edema and plication of the tongue, Mobius syndrome
(congenital facial diplegia, oculoparalysis from III and VI and infantile nuclear hypoplasia)
What is Bells phenemenon?
It refers to the upward movement of the right eyeball with incomplete closure of the right eyelid in an attempt to close the right eye.
Why are the muscles of the upper face spared in a upper motor neurone lesion?
The upper facial muscles are preserved in an UMN lesion as there are bilateral cortical representations of these muscles.
What is Bells palsy?
An idiopathic facial paralysis, believed to be due to viral-mediated cranial neuritis from HSV
Typically presents with abrupt onset of weakness with worsening the following day, associated with facial or retroauricular pain,
hyperacusis and excessive tearing
What is Ramsay Hunt syndrome?
Herpes zoster infection of the geniculate ganglion
Presents with vertigo, hearing loss, facial weakness, pain in the ear with vesicles seen on the external auditory meatus and
palate
What is facial synkinesis?
Attempt to move one group of facial muscles results in movement of another group
Occurs as a result of anomalous regeneration of the facial nerve
Egs if nerve fibres which innervate the facial muscles later innervate the lacrmial glands, then patient shed tears on mastication
(crocodile tears)
How would you investigate?
Targeted Ix according to history and physical examination
Blood Ix eg Lymes disease
Imaging
How would you manage (Bells palsy)?
Educate
Lubricating eye drops, eye patch, physiotherapy
PO Pred 1mg/kg/d for 7-10 days and PO acyclovir 400mg 5X/d for 7 days (within first 72hrs)
Regular follow up to look for resolution and exclude new developing signs suggestive of other conditions
Surgical (tarsorraphy) for chronic non-resolving cases

81

How would you educate or counsel patient with Bells palsy?


Common condition
Course
Improvement onset: 10 days to 2 months
Plateau: 6 weeks to 9 months
Residual signs
Synkinesis
Frequency: ~50%; May be reduced by corticosteroid treatment
May be treated with botulinum
Probably due to anomalous regeneration of nerve
Crocodile tears: 6%
Face weakness: 30%
Contracture: 20%
Blepharospasm: May occur years after paralysis
Prognosis better
Incomplete paralysis
Early improvement
Slow progression
Younger age
Normal salivary flow
Normal taste
Electrodiagnostic tests normal
Nerve excitability
Electrogustometry

82

36. Myasthenia Gravis


Examination
Statement
Examine face, CNs eyes
Approach to Ptosis (See Ptosis) and weakness
Sequence
Eyes
Ptosis with fatigability
Variable strabismus and diplopia that occurs after some time
Check for hyperthyroid and thyroid eye disease
Check for anaemia
Check for malar rash of SLE
Face
VII show your teeth : snarls
Assessment of speech : Yeeee or count 1 to 20, nasal voice (bulbar palsy)
Masseter weakness but pterygoids normal\
Check neck for goitre and scars
ULs
Normal deep tendon reflexes (Eaton Lambert and Miller Fisher are reduced)
Normal sensation
Fatigability with weakness
RA and SLE features
Thymectomy scar and plasmapheresis line
Associations
Endo: Thyroid, DM, Pernicious anaemia
CTD: RA, SLE, Polymyositis
Requests
Drug Hx
D-penicillamine usage for RA or Wilsons
Antibiotics and CVS drugs that can worsen MG
Temperature chart for fever precipitate weakness
Presentation
Sir, this patient has myasthenia gravis as evidenced by presence of muscle weakness with fatigability. The patients deep tendon
reflexes and sensation are normal.
On examination of the face, there is presence of bilateral facial muscle weakness producing a mask-like facies with ptosis. I noticed
furrowing of her forehead musculature in an attempt to compensate for the ptosis. There is presence of variable strabismus and
diplopia after sustained gaze. I also noticed the presence of a nasal voice as well as a reduction in volume of his speech when
asked to count from 1 to 20. The patient does not require a nasogastric tube.
I did notice mid line sternotomy scar which suggest previous thymectomy.
There is no goitre or features of hyperthyroidism. Patient also does not have symmetrical deforming polyarthropathy to suggest RA
and has no cutaneous features of SLE. I also did not notice any diabetic dermopathy.
I would like to complete my examination by
Checking his negative inspiratory force
Temperature chart
Drug history
In summary, this patient has myasthenia gravis with mild weakness of which a thymectomy has been done previously.

83

Questions
What is Myasthenia Gravis?
Autoimmune condition with antibodies targeting the post-synaptic Ach receptors of the neuromuscular junction
Resulting in progressive muscle weakness with use of the muscle and recovery of strength after a period of rest
Weakness experienced once number of receptors is 30% or less
How common is the thymus involved?
75% of cases of which 15% are thymomas and 85% are thymic hyperplasia
What are the common presentations?
Age
2 peaks
20 to 30 years old with female predominance
>50 years old with male predominance
Ptosis, diplopia
Dysarthria, difficulty swallowing (isolated bulbar muscles involvement occurs in 20%)
Generalised weakness or reduced exercise tolerance
Respiratory failure in 1%
Tends to occur extraocular muscles first, then to facial to bulbar and to limbs and truncal
What can exacerbate MG or precipitate crisis?
Non compliance to medications
Infection
Emotions
Drugs
Antibiotics: aminoglycosides, tetracyclines, macrolides and fluoroquinolones
CVS : Beta blockers, Calcium channel blockers (verapamil)
Others : Chloroquine, quinidine, procainamide, Li, Mg, Prednisolone, quinine(in gin tonic drinks), penicillamine
What is cholinergic crisis?
Can cause confusion between myasthenic crisis from cholinergic crisis
Results from excess of cholinesterase inhibitors such as neostigmine and physostigmine
Causes flaccid paralysis and SLUDGE (Miosis, salivation, lacrimation, urinary incontinence, diarrhea, gastrointestinal
hypermotility and emesis)
How would you investigate?
Blood Ix
AchR Ab
Positive in 80% with generalised MG
Positive in only 50% with ocular involvement only
also present in 90% of patients with penicillamine induced MG
Antistriated musce Ab
Anti Muscle specific kinase Ab (Anti MuSK Ab positive in patients with AchR Ab ve)
FBC to rule out infection
Imaging
CXR thymus (anterior mediastinal mass), aspiration pneumonia
CT for thymus
Tensilon test
Dx and distinguishing from cholinergic crisis
Edrophonium (T1/2 10 mins)
Look for objective improvement in ptosis (require observer)
Cardiac monitoring for bradycardia and asystole (Rx with atropine)
1 mg test dose and up to 10 mg
In cholinergic crisis, will get increased salivation etc
Note that in ALS, improvement in muscle weakness also occurs
Ice Pack test
Ice applied with glove to eyelids for 2 mins
Improvement in ptosis is dx (positive in 80%)
Electrodiagnostic studies
th
th
Repetitive nerve stimulation test shows a decrease in the compound muscle action potential by 10% in the 4 or 5
response to a train of nerve stimuli
Single fibre nerve electromyography evidence of neuromuscular blockade with increased jitter

84

How do you grade the severity of the weakness?


Myasthenia Gravis Foundation of America
o Grade 1 affects the ocular muscles only
o Grade 2 mild weakness affecting muscles other than ocular muscles
2A Affects the limb and axial muscles
2B - Affects the respiratory and bulbar muscles
o Grade 3 moderate weakness (3A AND 3B)
o Grade 4 Severe weakness (4A and 4B)
o Grade 5 Intubation required
Ossermans grading
o I: Ocular
o II A: Mild generalised with slow progression
o II B: Moderate generalised
o III: Acute fulminant MG
o IV: late severe MG (takes 2 yrs to progress from I to II)
How would you manage?
Emergencies in crisis (ABC)
Treat exacerbating factors
Stop medications that can exacerbate
Treat fever with antipyretics
Treat infections
Oral pyridostigmine, neostigmine
Steroids, azathioprine, cyclosporine
Plasmapheresis
IVIG
Thymectomy
What are the complications?
Myasthenic crisis
Severe exacerbation of MG
10% require intubation
Treatment complications
Cholinergic crisis
Cx of medications
What is Eaton-Lambert syndrome?
Myasthenic disorder associated with malignancy such as small cell ca of lung
Affects the proximal (especially the pelvic girdle and thigh) and truncal musculature; bulbar muscles is rarely involved
Improves with exercise
Presence of Abs to calcium channels

85

37. Approach to Examination of the Eyes


1.

Isolated nerve palsies

2.

Combined nerve palsies


Non conforming
Dysthyroid eye disease
Myasthenia gravis/Lambert Eaton Syndrome
Miller Fisher syndrome
Mononeuritis multiplex
Others brainstem pathology (V1), Kearnes Sayre, botulism, Wernickes
Conforming
Superior orbital fissure syndrome
Cavernous sinus thrombosis

3.

Pupillary abnormalities (See Eye Short cases)

4.

Ptosis

5.

Gaze Abnormalities
Supranuclear gaze palsies PSP, Parinauds
Gaze palsies INO and One and half eye
Nerve Palsies (III and IV)
Wernickes

6.

Nystagmus (See Eye Short cases)

7.

Visual Acuity

8.

Visual Fields

9.

Fundoscopy
Papilloedema
Optic atrophy
Retinitis pigmentosa
DM
Hypertensive
Chorioretinitis
CRVO
CRAO

86

38. Gaze Palsies


INO
Examination (example right INO)
o Cs
Abduction of the left eye with nystagmus a/w failure of adduction of the right eye on leftward gaze
The right eye is able to independently adduction
Saccadic eye movement horizontal saccade is abnormal with the right eye lagging behind the left eye
o Lesion is in the
Pons convergence is intact
Midbrain convergence is lost
o Proceed with other CNs examination
Multiple sclerosis (RAPD)
Myasthenia gravis
o Limbs
Multiple sclerosis cerebellar signs
CVA DM dermopathy, xanthelasma, AF
o Request for fundoscopy (optic atrophy)
Presentation
o Sir this patient has a right INO as evidenced by Cs
o The lesion is in the midbrain (anterior INO) or pons (posterior INO)
o Evidence for MG
o Evidence of MS
o Evidence for CVA
Questions
o What causes a right INO?
Lesion in the right medial longitudinal fasciculus that affects connects the ipsilateral third nerve innervation to
the right medial rectus to the left gaze center (parapontine reticular formation ie PPRF)
o What are the causes of INO?
Multiple sclerosis
Brainstem infarction
Pontine glioma
Infections
Lymes disease, Syphilis, Viral
Drug intoxication (phenothiazines, TCAs, phenytoin, CMZ)
Trauma
o How would you Investigate?
As above etiologies (MG, MRI, FPG, lipids, lyme titre, VDRL, drug)
o How would you manage
Mx of Multiple sclerosis
Mx of infarction and risk factors
Typically resolves with time
WEBINO (Walled-eye Bilateral INO)
Bilateral INO with exotropia and failure of convergence
Lesions in the pons and midbrain
Due to multiple sclerosis, vascular, gliomas and Wernickes
Fishers one-and-a-half syndrome
INO and ipsilateral gaze palsy
Due to lesion in the MLF and adjacent gaze center
Conjugate upward vertical gaze palsy
Midbrain lesion
MS, vascular, tumor
Conjugate downward vertical gaze palsy
Midbrain or
Foramen magnum
o Arnold-Chiari, Dandy Walker
o Acquired lesions (tumor, vascular, demyelination, abscess)
Supranuclear gaze palsy
Progressive suprnuclear gaze palsies (see Parkinsons disease)
o Loss of saccadic(frontal lobe) and pursuit movements (Occipital lobe)
o Loss of downward gaze, then upward gaze then horizontal gaze
o Can be overcome by Dolls reflex
Parinauds syndrome
o Loss of vertical gaze, nystagmus on convergence, PseudoArgyll-Robertson pupils
o Causes Ms, vascular, pinealoma
87

39. Approach to Unilateral Ptosis


Rule out pseudoptosis
o Life up any droopy eyelids
Muscle
o Dystrophia myotonica (see Dystrophia Myotonica)
Neuromuscular
o Myasthenia gravis (See Myasthenia Gravis)
Nerve
o III nerve palsy (see III nerve palsy)
o Horners syndrome
Unilateral Horners syndrome
Examination
Examine the other cranial nerves
o Cavernous sinus syndrome
o Superior orbital syndrome
o Lateral medullary syndrome (see CN syndrome)
o Syringobulbia (V, VII, IX XII)
o Multiple sclerosis (INO, Cerebellar, RAPD)
Neck
o Scars trauma, surgery
o Neoplasia
o Carotid aneurysm
o Cervical rib
Upper limbs (examine in this sequence)
o Pronator drift then cerebellar signs (Lateral medullary syndrome)
o Wasting of ipsilateral small muscles of hands (T1)
o Clubbing
o sensory loss T1
o Dissociated sensory loss (Syringomyelia)
o Contralateral loss to pain and temperature (Lateral medullary syndrome)
o Axilla trauma to brachial plexus
Chest
o Pancoast tumor
Inspection, dullness, auscultation
Trachea deviation
Ask for Loss of sweating and level
Presentation
Sir, this patient has a right sided isolated Horners syndrome as evidenced by
o Partial ptosis of the right eyelid
o Miosis of the right pupil with an intact light reflex
o Enophthalmos
o Elevation of the lower eyelid
o (Anhydrosis if you were allowed to ask the patient)
There was no associated CN abnormalities in particular
o Cavernous sinus syndrome
o Superior orbital syndrome
o Lateral medullary syndrome (see CN syndrome)
o Syringobulbia (V, VII, IX XII)
o Multiple sclerosis (INO, Cerebellar, RAPD)
Examination of the neck
Upper limbs
o Present axilla findings (stat the other findings with CN syndromes)
Chest for Pancoast lesion
DM dermopathy, xanthelasma
Request to ask patient for loss of sweating.
Summary

88

Questions
What are the features of Horners syndrome?
Partial ptosis paralysis of the upper tarsal muscle (Mullers muscle)
Miosis paralysis of pupil dilator
Enophthalmos paralysis of muscle of Muller
Slight elevation of the lower eyelid paralysis of the lower tarsal muscle
Loss of sweating
How do you delineate the site of lesion clinically?
Loss of sweating
St
o Central lesion loss in the head, upper trunk and arm (1 order)
o Neck
nd
Proximal to the superior cervical ganglion loss in face (2 order)
rd
Distal to superior crvical ganglion no loss (3 Order)
Adrenaline 1:1000 in both eyes (denervation hypersensitivity)
o Above the superior cervical ganglion (peripheral) = dilates the affected eye
o Below/Proximal to superior Cx ganglion or normal eye = no effect
Cocaine 4%
o Dilates normal eyes
o No effect on the affected side if above/distal to superior cervical ganglion
What are the causes of Horners syndrome?
Hypothalamus or brainstem
o Stroke
o Pontine glioma
o Coning of temporal lobe
Cervical cord (C8-T2 : intermediolateral column)
o Syringomyelia
o Multiple sclerosis
o Tumor
nd
Superior Mediastinum (2 order nerves exits the spinal cord and synapses at the superior cervical ganglion)
o Pancoast lesion (SCC of lung)
o Trauma to brachial plexus
Neck (carotid sympathetic plexus and superior cervical ganglion)
o Neoplasia
o Trauma
o Surgery (cervical sympathectomy)
o Carotid aneurysm
o Carotid Dissection (triad of pain, ipsilateral Horners and cerebral or retinal ischaemia)
Idiopathic
Congenital heterochromia of the iris (grey-blue on the affected side)
Migraine causes intermittent Horners syndrome

89

40. Approach to Bilateral Ptosis


Muscular (usually no wrinkling of the forehead)
o Dystrophia myotonica (see Dystrophia myotonica)
o Ocular myopathy
o Oculopharyngeal dystrophy
o Chronic progressive external ophthalmoplegia (mitochondrial/Kearnes Sayrre)
Neuromuscular
o Myasthenia gravis (See Myasthenia gravis)
Nerve
rd
o Bilateral 3 (rare)
o Bilateral Horners (Syringomyelia)
o Tabes dorsalis
o Miller Fisher syndrome
Examination
General screen for dystrophia myotonica or fascioscapular dystrophy
Screen for myasthenia gravis
Check CNs
o III, Horners
o Argyll Robertson pupils (Tabes)
o Ophthalmoplegia (Kearnes Sayrre)
o Bulbar palsy (Syringomyelia)
Neck
Upper limbs
o Ataxia (Kearnes sayrre, Miller Fisher)
o Syringomyelia
Flaccid and wasted ULs
Dissociated sensory loss
Spastic paraparesis
o Areflexia (Miller Fisher)
Complete examination by fundoscopy for retinitis pigmentosa (CPEO)
Presentation
This patient has bilateral ptosis
No evidence of DM, MG
No evidence of
o Horners, Syringomyelia
o III
o Argyll Robertson
o Miller Fisher
The possible aetiologies include ocular myopathy, oculopharyngeal myopathy, CPEO and congenital ptosis.

90

41. Assessment of Higher Cortical function


Examine this mans cortical function
o Obvious gaze preference or hemiparesis
o

Determine which side first with sensory inattention (Opp to lesion)


Visual inattention (always check gross VA first)
Tactile inattention
Line bisection test (parietal lobe function)
Circling alphabets (frontal lobe)

Determine side (opp to lesion side)


Astereognosis (coins, pen)
Graphaesthesia (write 5 nos on each palm using pen with cover)
Visual field for hemianopia

If suspect left side


Dysphasia assessment (expressive, receptive, nominal and conductive)
Gerstmanns syndrome (AALF)

If suspect right side


Constructional apraxia (copy a cube)
Dressing apraxia
Spatial neglect (Right lesion = fill in nos on left only)

Parietal lobe function


Gerstmanns and nominal dysphasia vs apraxia and spatial

Frontal lobe
Grasp reflex and palmomental reflex, glabella tap
Optic atrophy
Anosmia
Expressive dysphasia
Labile emotion, personality changes
Urinary catheter
Gait apraxia

Temporal lobe
Superior quandrantonopia
Receptive dysphasia
Short and long term memory

Occipital Lobe
Cortical blindness
Hemianopia with macula sparing

Look for contralateral UMN VII or ipsilateral XII

Check for pronator drift

Aetiology
Proceed to check pulse, carotid bruit, murmur, hyperlipidemia, DM dermopathy and tar stains
Request BP, urinalysis for DM and fundoscopy (papilledema)

Complications
DVT, sacral sore, bedside swallow test, aspiration pneumonia

91

Examine this mans speech


Shake his right hand
o Look for weakness on the right UL (Dysphasia)
o Look for ataxia of the UL (cerebellar)
o Look for tremors (Parkinsonism)
Dysphonia
o Ask him for his name, how old he is, how he came to hospital and what he did this am
o Recurrent Laryngeal nerve or laryngitis
o Offer to examine the left chest, radiation marks, enlarged Cx LNs and look for Horners, and wasting of T1
Dysarthria
o British Constitution and count 1 to 20
Cerebellar : slow, slurred, explosive and irregular
Parkinsonism : Monotonous, low volume
o Exclude Cerebellar and Parkinsonism
Proceed accordingly if these are detected
o Say Ba Ba Ba VII nerve palsy
o Say La La La Pseudobulbar palsy
o Say Ke Ke Ke Bulbar palsy
Dysphasia
o Expressive
What is your favourite colour, what you have for breakfast this am
Brocas area
o Receptive
close your eyes, close your eyes and stick out your tongue, close your eyes and stick out your tongue
and lift both hands
Wernickes area posterior part of first temporal gyrus
o Conductive
No if and or but
Arcuate fasciculus linking Brocas and Wernickes area
o Nominal
Shirt, sleeve, button OR Watch, straps face
Angular gyrus (temporal-parietal)
o Global (Expressive and receptive)
Once dysphasia is found
o Do Gerstmanns syndrome
Acalculia (serial 7 subtraction)
Agraphia (inability to write)
Left-right disorientation
Finger agnosia
o Demonstrate other cortical signs
Visual fields, sensory inattention, graphaesthesia, astereognosis
o Demonstrate UMN VII and hemiparesis
o Determine aetiology as above (Vascular, tumour)
o Determine complications as above

92

UPPER LIMBS!
42. Upper Limbs Overview
Motor
o Unilateral vs bilateral
o Proximal vs distal vs entire UL
o Myopathy, Neuromuscular and Neuropathy
Sensory (See Lower Limbs)
o Peripheral neuropathy
o Mononeuropathy/mononeuritis multiplex
o Rediculopathy
Movement disorders
Motor
Distally
Wasted hands
o Myopathies
o Nerve (Think of levels)
Mononeuropathy
Ulnar
Median
Radial
Combination of above three
Peripheral neuropathy
Brachial plexus
Roots
Anterior Horn
Spinal cord
Claw hands
o Partial claw ulna claw hand (r/o dupytrens contracture)
o Total claw
Neurological combined ulna and median, leprosy, brachial, polio, syringomyelia
Non-neurological RA, ischaemic contracture, Scleroderma
Proximally
Proximal myopathy, Dystrophia myotonica
Myasthenia gravis
Syringomyelia, Radiculopathy, upper brachial plexus
Entire Upper Limb
Bilateral
o Cervical myelopathy
o Syringomyelia
o MND
o Spinal Muscular Atrophy
o Polyradiculopathy
o Bilateral Brachial Plexus (trauma or bilateral cx Rib)
Unilateral
o Polio
o Brachial neuritis
o Polyradiculopathy
o Hansens disease
o Hemiparesis
o Infantile hemiplegia
o Brown-Sequard

93

Movement disorders
Parkinsonism
Chorea and movement disorders
o Think: 3,3,3
Peripheral : Parkinsonsim, Rh heart, SLE
Face: Wilsons, Hyperthyroidism, Polycythaemia
Request: CVS, Drug, AMT for Huntington
Hemiballismus
Cerebellar
o Think

Unilateral: 4
Stroke and CNs and risk factors
SOL and CNs
Parkinsonism
MS
Bilateral (2/2/2/2/2/2)
ULs
o Alcohol
o Parkinsonism
o (NF)
Eyes
o Wilsons, MS
Mouth
o Hypothyroidism
o Phenytoin
o (Alcohol)
LLs
o FA
o (Ataxia telangiectasia)
General
o Wasting (paraneoplastic)
o Bilateral strokes
Requests
o Drugs
o Infection

94

43. Radial Nerve Palsy


Examination
Suspect this on an apparently normal looking pair of ULs
Proceed to examine the ULs as per normal, concentrating on median and ulnar nerve as well as brachial plexopathy
Once radial nerve palsy detected, proceed to look for level
o Demonstrate weakness of extension at the MCPJ
o Preservation of IPJ extension (lumbricals and interossei muscles)
o Weakness of wrist extension
o (dont forget to extend wrist before testing grip strength)
o (dont forget to test finger abduction and adduction with hands flat on a surface)
o Test brachioradialis
o Test triceps muscles, triceps jerk
o Thumb abduction and Oschners clasping test for median screen
o Finger abduction and Froments sign for ulnar screen
o Look for reduced sensation in the first dorsal interosseous or anatomical snuffbox
o Inspect the forearm, elbow, humerus and shoulder
o Check the gums for lead poisoning blue-black line on the gingival margin
o Test Function
Presentation
Sir, this patient has got an isolated right radial nerve palsy at the level of the upper third of the humerus or above.
I say this because of weakness of extension of the fingers at the MCPJ and at the wrist associated with weakness of the
brachioradialis muscle, triceps muscles with weakness of extension at the elbow. In addition, there is also numbness of the first
dorsal interosseous space. There is no evidence of concomitant ulnar or median nerve palsies.
I did not detect any scars or deformities over the humerus or the axilla. (Mentioned other areas if the level is lower) There is also no
clinical evidence of lead poisoning such as a blue-black line on the gingival margin.
Possible causes include compression of the right radial nerve such as crutch palsy at the axilla or Saturday night palsy at the
humerus.
I also note that there is presence of a splint for his wrist and finger drop. He is able to perform coarse and fine motor function.

95

Questions
What is the course of the radial nerve and its branches?
o C5, 6,7,8, T1 and emerges from the posterior cord of the brachial plexus
o Leaves the axilla and enters the arm between the long head and medial heads of the triceps and supplies the
triceps
o Spiral groove on the back of the humerus between the lateral and medial heads of the triceps
o Lower third of the humerus, it pierces the intermuscular septum to enter the anterior compartment of the arm
where it supplies the brachioradialis
o It gives off a branch supplying the extensor carpi radialis longus
o At the elbow, ie lateral epicondyle of the humerus, it gives off the posterior interosseous nerve which supplies all
the extensors of the forearms including the abductor pollicis longus and supinator except the extensor carpi
radialis longus
o The radial nerve continues as the superficial radial nerve which provides sensory innervation of the posterior
aspects of the radial 3 digits.
What are the various levels of lesions and what are the correlating clinical features?
o Axilla eg crutch palsy All gone including triceps and triceps reflex
o Humerus
Upper third all is lost
Middle third
triceps and triceps reflex preserved and brachioradialis and below is lost
Saturday night palsy
Lower third triceps and brachioradialis is preserved
o Elbow
Like lower third
Only the PIN involved
Extensors of the fingers at the MCPJ affected only
Wrist drop is not a feature as the extensor carpi radialis longus is intact and this alone can
effect wrist extension
o Forearm
PIN involvement
Superfical radial nerve palsy; aka Watenberg syndrome which is an entrapment syndrome where there
is pain and numbness over its distribution of the first web space dorsally only because of overlap
What are the causes?
o Trauma form accident or surgery
o Compression or entrapment
o Part of a mononeuritis multiplex
o Lead poisoning
o (for PIN, finger drop can be secondary to synovitis from RA)
How would you investigate?
o Detailed history for the cause
o X-ray for fracture, healing callus or tumor
o EMG and NCT to locate the level of injury and to monitor recovery progress
How would you manage?
o Education and counselling
o OT and PT with a wrist splint and cock-up splint for finger drops
o Surgical
Whats the prognosis?
o Neuropraxia with no disruption to the sheath or the axon
Recovery complete and rapid (weeks)
o Axonotmesis with disruption of the axon but an intact Schwann sheath
Recovery complete but slower (1mm/day)
o Neuronotmesis
Recovery is incomplete

96

44. Median Nerve Palsy


Examination
Upon suspecting median nerve palsy, rule out ulnar and brachial neuritis
Median nerve palsy
Motor
Wasted thenar eminence
Thumb is externally rotated into the plane of the thumb rather than perpendicular
Pen-touch test (for abductor pollicis brevis)
Oschner clasping test (flexor digitorum superficialis)
Flexion of the terminal digit of the thumb (flexor pollicis longus)
Flexion of the terminal digit of the index finger (flexor digitorum profundus)
Sensory
Test for reduced sensation in the lateral 31/2 fingers as well as thenar eminence.
Exclude ulnar and radial nerve palsy
Aetiology
Tinels sign
Look for RA hands
Look at the wrist and forearm, elbow, arm and axilla for scars.
Test for function
Rule out Myxedema and acromegaly
Presentation
Sir, this patient has an isolated unilateral right median nerve palsy with wasting of the right thenar eminence associated with an
externally rotated thumb. There is weakness of abduction of the thumb as demonstrated by the pen touch test associated with
reduced sensation to pinprick in the right lateral 3 1/2 fingers. Oschners clasping test is negative and flexion of the terminal
phalanx of the thumb and index fingers are preserved, indicating that the level of the lesion is at the wrist.
There is no ulna or radial nerve palsies.
In terms of aetiology, there is also no evidence of RA of the hands and patient does not have features of hypothyroidism or
acromegaly. Tinels sign is negative and there are no scars noted on the right upper limb.
Both fine and coarse motor functions are intact.
In summary, this patient has a right median nerve palsy at the level of the wrist. Possible aetiologies includes surgical causes such
as compression, trauma or surgery or medical causes such as mononeuritis multiples, infection, inflammatory and ischaemic
causes.

97

Questions
What is the course and branches of the median nerve?
It supplies all the muscles of the forearm except the flexor carpi ulnaris and the ulna half of the flexor digitorum profundus
and LOAF (lateral 2 lumbricals, opponens pollicis, abductor pollicis brevis and flexor pollicis brevis)
Formed by lateral(C5-7) and medial(C8,T1) cords of the brachial plexus
Enters the arm closely related to the brachial artery with no branches above the elbow
Enters the forearm lateral to the brachialis tendon and in between the pronator teres.
Gives off the anterior interosseous nerve
Above the wrist, gives off the palmar cutaneous branch
Enters the carpal tunnel and supplies LOAF and sensory branch to the lateral 3 fingers.
Branches
Forearm flexor carpi radialis, flexor digitorum superficialis (flexion of fingers at the PIPJ), pronator teres
AIN Flexor pollicis longus (flexion of the DIPJ thumb), flexor digitorum profundus of the lateral 2 fingers (flexion of at
the DIPJ), pronator quadratus
Palmar cutaneous (to the thenar eminence)
Terminal motor (LOAF) (NB for F for flexion at the MCPJ thumb)
What are the various levels of lesions and the clinical correlation?
Wrist
Wasting of thenar, ext rotated thumb, pen touch test positive; sensory loss of the lateral 3 fingers
Cubital fossa
Above plus
Oschner clasping test positive and failure of flexing the terminal digits of the thumb and index finger
Arm and axilla (same as cubital fossa)
(For forearm, depends where the lesion is eg AIN syndrome will affect flexor digitorum profundus and flexor pollicis longus
only)
What are the causes?
Trauma
Surgical
Compression
Mononeuritis multiplex
Infection Leprosy
Inflammatory CIDP
Ischaemic - Vasculitis
Causes of Carpal tunnel syndrome
Idiopathic
Pregnancy, OCPs
Endocrine Hypothyroidism, Acromegaly
Hands RA, gout, TB tenosynovitis, OA of
carpus
Amyloidosis, CRF, sarcoidosis

98

What are the tests to demonstrate a median nerve palsy?


Tinels sign (percussion)
Phalens test (flexion at the wrist for 60 s)
Hyperextension of the wrist (for 60 s)
Tourniquet test (sphygmomanometer for more than 2 mins above systolic)
Luthys sign Skinfold does not close tightly around a bottle or cup; secondary to thumb abduction paresis
Durkans test apply direct pressure over the carpal tunnel
What are the other areas of nerve compression?
Median nerve (CTS)
Ulna nerve (elbow tunnel)
Radial nerve (spiral or humeral groove)
Meralgia paraesthetica (lateral cut nerve of the thigh at the ing lig)
Common peroneal nerve (head of the fibula)
Posterior tibial nerve (Tarsal tunnel syndrome)
Plantar nerves of the 3rd/4th toes (Mortons metatarsalgia)
How would you investigate?
o
Blood Ix
o
Imaging X-rays
o
Nerve conduction test demonstrating slow sensory conduction across the transverse carpal ligament.
How would you manage?
Education
OT and wrist splint
Medications treatment of underlying disease, withdrawing OCPs, IA steroid
Surgical decompression
What is the prognosis?
o
Neuropraxia with no disruption to the sheath or the axon
Recovery complete and rapid (weeks)
o
Axonotmesis with disruption of the axon but an intact Schwann sheath
Recovery complete but slower (1mm/day)
o
Neuronotmesis
Recovery is incomplete

99

45. Ulnar Nerve Palsy


Examination
Rule out median, radial and brachial neuritis
Inspecting
th
th
Wasting of the muscles of the hands, hypothenar eminence and partial clawing of the 4 and 5 fingers, sparing of the
thenar eminence, ulnar paradox
Proceed to tests for finger abduction and Froments sign (weakness of the adduction of the thumb)
th
Test finger flexion of the 5 finger for flexor digitorum profundus involvement; test for wrist flexion at the ulna side and look
for the tendon of the flexor carpi ulnaris
Rule out median nerve (thenar eminence and ext rot thumb, pen touch test and Oschner clasping test) and radial nerve
Sensory testing in the medial 1 fingers; test T1 sensory loss
Examine the wrist and elbows (feel for thickened nerve, wide carrying angle))
Function
Thickened nerve (cf with Pb for radial and Acromeg etc for median)
Presentation
Sir, this patient has got a isolated left ulnar palsy as evidenced by a left ulnar claw hand with wasting of the small muscles of the
hands with dorsal guttering as well as wasting of the hypothenar eminence. There is sparing of the thenar eminence.
th

There is weakness of finger abduction and Froments sign is positive. There is preservation of the flexion of the DIPJ of the 4 and
th
5 fingers; when the hand is flexed to the ulna side against resistance, the tendon of the flexor carpi ulnaris is palpable. This is
associated with reduced sensation to pinprick in the medial 1/1/2 fingers. There are no associated median or radial nerve palsies
and T1 involvement.
In terms of aetiology, there is a scar at the wrist associated with a marked ulnar claw hand, demonstrating the ulna paradox. I did
not find any signs to suggest leprosy such as thickened nerves, hypopigmentation patches or finger resorption.
Both coarse and fine motor function of the hand is preserved.
In summary, this patient has a left ulna claw hand due to a traumatic injury to the left wrist.

100

Questions
What is the anatomical course of the ulnar nerve?
th
It provides motor to all muscles of the hands except the LOAF; flexor carpi ulnaris and flexor digitorum profundus to the 4 and
th
5 fingers.
Sensory to the ulna 1 fingers
Begins from the medial cord of the brachial plexus (C8 and T1)
No branches in the arm
Enters the forearm via the cubital tunnel (medial epicondyle and the olecranon process) and motor supply to the flexor carpi
ulnaris and ulna half of the flexor digitorum profundus
It gives off a sensory branch just above the wrist and enters Guyons canal and supplies the sensory medial 1 fingers and
hypothenar as well as motor to all intrinsic muscles of the hands except LOAF.
What is the level of lesions and its clinical correlation?
Wrist Hypothenar eminence wasting, Froments positive, weakness of finger abduction, pronounced claw and loss of
sensation
Elbow less pronounced claw and loss of terminal flexion of the DIPJ and loss of flexor carpi ulnaris tendon on ulna flexion of
the wrist
How do you differentiate ulnar nerve palsy vs a T1 lesion?
Motor wasting of the thenar eminence in addition for T1
Sensory loss in T1 dermatomal distribution
What is the ulna claw hand?
th
th
th
th
It refers to the hyperextension of the 4 and 5 MCPJ associated with flexion of the IPJs of the 4 and 5 fingers as a result of
th
th
ulnar nerve palsy. It is due to the unopposed long extensors of the 4 and 5 fingers in contrast to the IF and MF which are
counteracted by the lumbricals which are served by the median nerve.
What is the ulnar paradox?
It means that the ulnar claw deformity is more pronounced for lesions distally e.g. at the wrist as compared to a more proximal
lesion e.g. at the elbow. This is because a more proximal lesion at the elbow also causes weakness of the ulnar half of the flexor
th
th
digitorum profundus, resulting in less flexion of the IPJs of the 4 and 5 fingers.
What is Froments sign?
Patient is asked to grasp a piece of paper between the thumbs and the lateral aspect of the index finger. The affected thumb will
flex as the adductor pollicis muscles are weak. (Patient is trying to compensate by using the flexor pollicis longus supplied by
median nerve)
What are the causes of an ulnar nerve palsy?
Compression or entrapment (Cubital tunnel at the elbow and Guyons canal at the wrist)
Trauma (Fractures or dislocation cubitus valgus leads to tardive ulnar nerve palsy)
Surgical
Mononeuritis multiplex
Infection leprosy
Ischaemia Vasculitis
Inflammatory - CIDP
How would you investigate?
Blood Ix to rule out DM if no obvious cause
X-rays of the elbow and wrist (both must be done to rule out double crush syndrome) (KIV C-spine and CXR)
EMG(axonal degeneration for chronic) and NCT(motor and sensory conduction velocities useful for recent entrapment as well
as chronic) to locate level and monitor
How would you manage?
Education and avoidance of resting on elbow
OT, PT
Medical NSAIDs and Vit B6
Surgical decompression with anterior transposition of the nerve
NB: LOAF lateral 2 lumbricals, opponens pollicis, abductor pollicis brevis and flexor pollicis brevis

101

46. Wasted Hands


Unilateral vs Bilateral (think of levels!)
Unilateral
Think of (no myopathy, got brachial plexus)
Peripheral nerve (median, ulnar or combined)
Mononeuropathy vs peripheral neuropathy (asymmetric involvement)
Brachial plexus (trauma, tumor, radiation, Cx rib)
C8-T1 root lesions (Cx spondylosis)
Anterior Horn Cell (Poliomyelitis)
Cervical cord
Proceed as:
Long case as per protocol, check also neck and chest
Short case
On inspection, unilateral wasted hands noted
Neurological hand screen
Examine for ulnar and median nerve palsies.
Check for sensory for nerve vs root (peripheral nerve vs brachial plexus) and no loss (ie anterior horn cell)
Note sensory for ulnar, median and radial
Note sensory of peripheral neuropathy
Note dermatomal sensory
Feel for thickened nerves, look for hypoaesthetic macules, fasciculations
Look for scars in the axilla and neck (neck pain, tenderness), Cx rib
Check function
Requests
Palpate for cervical rib and features of Pancoasts tumor (dullness to percussion, Horners syndrome, hoarseness
voice)
Check for winging of scapula (for brachial plexus involvement)
If brachial plexus
Upper vs lower (wasting of muscles of hands) vs complete
Surgical(Cx rib, Pancoast) vs medical cause(brachial neuritis)
Test for proximal involvement
Serratus anterior (winging of scapula on pushing against wall) ie C5,6,7
Supraspinatus (abduction of UL from hands by your side position) C5
Infraspinatus (elbow flexed and push backwards) C5
Rhomboids (hand on hip and push backwards) C4,5,6
Reflexes (inverted supinator jerk)

102

Bilateral
Think of
Rule out the obvious (hand screen)
RA, gouty hands
Dystrophia myotonica
Levels (got myopathy, maybe brachial plexus if bilateral Cx ribs)
Distal myopathy (reflexes normal; rare), dystrophia myotonica
Peripheral nerve lesions
Combined CTS (see median nerve palsy)
Combined ulnar and median nerve
Leprosy (resorption, hypoaesthetic macule and thickened nerve)
HMSN (look at the feet for pes cavus deformities, thickened nerves)
Peripheral motor neuropathy
(Not likely to be brachial plexus unless bilateral Cx ribs)
Nerve roots
Cervical spondylosis (inverted supinator jerk, increased jerks for high cervical cord lesions)
Anterior Horn cell (no sensory loss)
MND (fasciculations)
Poliomyelitis
SMA
Spinal cord lesions
Intramedullary (Syringomyelia dissociated sensory loss)
Extramedullary
Request
LL spastic paraparesis ( if suspect Cx cord, MND)
Lower cranial nerve (bulbar palsy if suspect MND or syringomyelia)
Proceed as
Long case
Proceed as per normal
Examine or request to examine the neck (pain tenderness and pain on neck movements), chest, CNs and LLs
accordingly
Short case
Neurological hand screen
Median and ulnar nerve testing, and wrist drop( because this is also weak in C8 root lesions)
Sensory peripheral nerve vs neuropathy vs root
Check the elbows for thickened nerves
Look for fasciculations (peripheral nerve, neuropathy, MND), hypoaesthetic macules
Inspect the neck
Quick glance at the face (NG tube bulbar palsy, LLs HMSN)
Check function
Request for reflexes, percussion myotonia if deemed appropriate (if suspect Cx cord lesion or dystrophia myotonica)

103

Questions
What are the levels and causes?
Disuse atrophy (RA hands)
Myopathy (distal myopathies or dystrophia myotonica usually forearms more affected)
Peripheral neuropathy - motor (see causes in Neurology segment)
Mononeuropathy
Surgical, trauma or compression
Mononeuritis multiplex, infection, inflammatory and ischaemic
Brachial Plexus
Surgical, trauma compression (Pancoasts, Cx rib)
Brachial neuritis
Nerve root (Disc prolapse)
Anterior Horn cell
MND, poliomyelitis, SMA
Spinal cord
Intramedullary
Extramedullary
How would you Ix?
Blood Ix according to causes as above
Imaging X-rays, CT or MRI of spine
NCT/EMG
What are the causes of a claw hand?
Partial claw
Ulnar nerve palsy (See Ulnar nerve)
True Claw
Non-neurological
RA
Severe Volkmanns ischaemic contracture
Neurological (5)
Combined median and ulnar nerve
Leprosy (reflexes present. Pain loss, thickened nerves)
Lower brachial plexus ( C7-T1, selective loss of reflexes, pain loss)
Poliomyelitis (reflexes selective, pain intact)
Syringomyelia (reflexes absent, pain loss)

104

47. Syringomyelia
Examination
Proceed as per normal for the upper limbs
Once dx is made, request
o Examine the neck
Scars of previous Sx
Scoliosis
o The cranial nerves
Horners syndrome
Ataxia and nystagmus
Bulbar palsy (syringobulbia)
Loss of temperature and pain sensation from the outer part of the face progressing towards the center
o The lower limbs
Spastic paraparesis
Presentation
Sir, this patient has got syringomyelia as evidenced by
LMN pattern of weakness of both ULs
o Wasting and weakness of the small muscles of the hands and forearms
o Reduced tone and reflexes
There is dissociated sensory loss with
o Loss of sensation to pinprick in the ULs and upper chest
o With intact sensation to vibration and proprioception
I also noticed presence of
o Scars and old burn marks on his fingers
o But I did not detect any Charcots joints of the ULs
o La main succulente ugly, cold, puffy, cyanosed hands with stumpy fingers and podgy soft palms
Examination of the face
o There was no evidence of bulbar palsy
Palatal movements were normal, and CN XI and XII were intact
o There was also no Horners syndrome
o No ataxia or nystagmus
o However there is loss of sensation to pinprick of the face in an onion skin pattern
Examination of the neck
o No surgical scars noted
o No kyphoscoliosis
Examination of the lower limbs
o Spastic paraparesis
In summary, this patient has syringomyelia with presence of wasting of the upper limbs, dissociated sensory loss and spastic
paraparesis of the lower limbs. This has resulted with complications of repeated trauma of his hands.

105

Questions
What is syringomyelia?
Cavity formation with presence of a large fluid filled cavity in the grey matter of the cervical spinal cord which is in
communication with the central canal and contains CSF.
Triad of LMN weakness of the ULs, dissociated sensory loss in the ULs and UMN weakness in the LLs
How do patients present?
th
th
Rare disorder, 4 to 5 decades, male=females
Painless trauma or burns in the upper limbs, poorly localised pain in the ULs
What is the pathophysiology?
At the level of the syrinx
o LMN anterior horn cell affected
o Dissociated sensory loss affects the decussating fibres of the spinothalamic tract
Below the level of the syrinx
o Affecting the pyramidal corticospinal tract with spastic paraparesis of the LLs and preservation of sphincters
Extension into the upper cervical cord and medulla
o Horners syndrome
o Bulbar palsy (CN IX-XII)
o Ataxia and nystagmus ( affects the medial longitudinal bundle if lesion from C5 upwards)
o Onion skin pattern loss of pain in the face (spinal nucleus of V CN which extends from the pons to the upper cervical
cord)
What are the differential diagnoses for dissociated sensory loss?
Anterior spinal artery occlusion (affects the spinothalamic tract)
DM neuropathy, leprosy, hereditary amyloidotic polyneuropathy
What are your differential diagnoses for syringomyelia?
Craniovertebral anomalies
Spinal cord injuries
Intramedullary tumours of the spinal cord
Arachnoiditis around the foramen magnum obstructing CSF flow
Hematomyelia
What are the associated abnormalities?
Arnold-Chiari malformation
Bony defects around the foramen magnum
Hydrocephalus
Spina bifida
Spinal cord tumours
How would you Ix?
MRI scan of the spinal cord
How would you manage?
Drainage of the syrinx to the subarachnoid space
Syringoperitoneal drainage
In AC malformation, cervical laminectomy and removal of the lower central portion of the occipital bone
Intramedullary tumour excision
What is syringobulbia?
Syrinx in the medulla of the brainstem
Usually extension of the syringomyelia but can be isolated
Results in
o Horners
o Ataxia and nystagmus
o Bulbar palsy
o CN V, VII, IX and X especially
o Onion skin pattern of loss of pain sensation of the face

106

48. Dystrophia Myotonica


Approach to Congenital Myopathies
1. Duchennes, Beckers
2. Myotonia
Dystrophia myotonica (fascioscapular dystrophies can mimic appearance)
Congenital myotonia
Hereditary paramyotonia
3. Fascioscapulahumeral dystrophies, limb-girdle dystrophies, distal myopathies
Examination
Examine patients face or hands (Can be short case of locomotor or in CNS station)
Examine the hands
Demonstrate difficulty opening hands after shaking
Repeatedly open and close the hands
Percussion myotonia of the thenar eminence
(proceed with hand examination with function assessment if locomotor station)
Demonstrate weakness in the forearms (especially) and hands
No sensory loss
Loss of reflexes
Check the pulse (dysrhythmias, small volume pulse)
Examine the face
Myopathic facies
Expressionless
Triangular facies
Wasting of the temporalis, masseter (palpate these muscles when patient clenches teeth)
Frontal balding
Bilateral ptosis
Close his eyes and open
Tongue for percussion myotonia
Gum hypertrophy from phenytoin toxicity
Swan-neck appearance with wasting of the SCM (test for weakness of SCM), weakness of flexion of the neck
Nodular thyroid enlargement
Request
Face
Cataracts - posterior subcapsular and stellate
Assess Speech slurring due to myotonia of the tongue and pharyngeal muscle
Chest examination
Gynecomastia
Cardiovascular examination dilated cardiomyopathy (split S1, mitral murmur, low BP and pulse volume)
Testicular atrophy
Urine dipstick for diabetes mellitus
Lower limbs bilateral footdrop
Presentation
Sir, this patient has got dystrophia myotonica as evidenced by
A myopathic facies that is triangular in appearance with an expressionless look. There is wasting of the facial muscles
involving the temporalis and masseter muscles associated with frontal balding and bilateral ptosis. He had difficulty opening his
eyes after firm closure. There was myotonia affecting the tongue.
There is also a swan-neck appearance with wasting of the sternocleidomastoid muscles with weakness of flexion of the neck.
On shaking his hand, there was a delay in releasing his grip. In addition, after making a fist, he was unable to quickly open it
especially after doing this repetitively. There was also presence of percussion myotonia of the thenar eminence. There is
presence of proximal myopathy and wasting with involvement of the forearms and hands. There are also reduced reflexes with
no sensory loss detected. Function is relatively preserved.
With regards to complications
His pulse is regular at 80 bpm with a small volume pulse suggesting dil CMP
There was no gum hypertrophy to suggest chronic phenytoin use.
There is nodular thyroid enlargement.
I would like to complete my examination by
Face
Cataracts - posterior subcapsular and stellate
Assess Speech slurring due to myotonia of the tongue and pharyngeal muscle
Chest examination
Gynecomastia
Cardiovascular examination dilated cardiomyopathy (split S1, mitral murmur, low BP and pulse volume)
Testicular atrophy
Urine dipstick for diabetes mellitus
Lower limbs foot drop with high steppage gait (tibial nerves are affected early)
107

Questions
What are the types of muscular dystrophies you know of?
1. Duchennes
Sex linked
Pseudohypertrophy of the calves or deltoids
Gowers sign, proximal weakness
Cardiomyopathy
Beckers
Sex linked
Later onset and less severe form of Duchennes
2. Limb-girdle
Autosomal recessive
Shoulder and pelvic girdle affected
Third decade
Sparing of the face and heart
Fascioscapulohumeral
Autosomal dominant
Bilateral, symmetrical weakness of the facial and SCM with bilateral ptosis
Weakness of the shoulder muscles and later the pelvic girdle muscles
3. Dystrophia myotonica
Congenital myotonia
Hereditary paramyotonia
4.

Distal myopathies eg Welanders myopathy

What is myotonia?
Continued contraction of the muscles after voluntary contraction ceases, followed by impaired relaxation.
What is dystrophia myotonica?
Characteristic clinical appearance with myotonia and weakness with no sensory loss
Autosomal dominant with a trinucleotide (AGC) repeat disorder on chromosome 19
Anticipation phenotypic expression worsens with each successive generation
rd
th
Onset in the 3 or 4 decade
Males>females
In addition to the characteristic facies and musculoskeletal involvement
Intellectual and personality disorder
Cataracts posterior subcapsular cataracts which are stellate type
CVM dilated cardiomyopathy and conduction defects
Resp recurrent infection from weakness of the bronchiolar musculature, hypoventilation and post-anaesthetic respiratory
failure
Abdomen dysmotility and dysphagia
Testicular atrophy and gynecomastia
Diabetes mellitus
Nodular thyroid enlargement
How would you investigate?
Confirm Diagnosis
EMG dive bomber pattern ie waxing and waning of the potentials
Muscle biopsy shows no inflammatory changes with type 1 fibre atrophy which is characteristic but not diagnostic
DNA analysis
Muscle enzymes are normal
Screen for Complications
FPG screen for diabetes mellitus
ECG heart blocks, small P, prolonged PR, notched QRS and prolonged QTc
CXR enlarged heart
Slit-lamp examination for cataracts
How would you manage?
Education, genetic counselling
PT/OT eg foot orthosis for foot drop
Medications phenytoin for myotonia, other anti-myotonic medications such as quinine and procainamide should be avoided
due to aggravation of cardiac conduction defects; however it is the weakness that causes disability and not myotonia
rd
Pacemaker for 3 degree heart block or symptomatic such as syncope

108

How would you counsel the patients family?


Vertical
Autosomal dominant, children 1 in 2
Anticipation
DNA analysis is available for some families for prenatal diagnosis
Horizontal
Screen with clinical examination
Slit-lamp examination
EMG
What are the other types of myotonia disorders?
Myotonia congenita (Oppenheims disease)
Autosomal dominant or recessive
Presence of myotonia without other features of dystrophia myotonica
Present at infancy with difficulty feeding with subsequent improvement
No weakness and reflexes are preserved
Herculean appearance
Channelopathies
Hereditary paramyotonia
Autosomal dominant
Cold-induced myotonia
What are your differential diagnoses for dystrophia myotonica?
Facies appearance Facioscapulohumeral dystrophy
Autosomal dominant, onset at age 10-40, Chr4, normal lifespan
Face ptosis, difficulty closing eyes, facial weakness and speech impaired
Normal IQ
Neck wasted SCM and weakness
Shoulder winging of scapula, weakness of pectoralis, trapezius, biceps and triceps and hypertrophy of deltoids
Occasionally affecting the anterior tibialis
Normal CK
Proximal weakness FSH, limb-girdle, prox myopathy causes, MG
Limb Girdle dystrophy (see prox myopathy)
Distal weakness Welanders distal myopathy, nerve problem
Myotonia Congenital myotonia, hereditary paramyotonia

109

49. Cerebellar Signs


Examination
Stem statement
Giddiness, falls, unsteadiness
Face, Speech, ULs and LLs
Unilateral
Upper limbs
Screen for pronator drift, ensure patient can see your finger!
Cerebellar signs
Dysmetria with intention tremor
Dysdiadochokinesia
Dyschronometria
Power for ataxic hemiparesis
Sensory
Temperature/Pain loss in syringomyelia and LMS
Tone for cogwheel and leadpipe rigidity
Skin for neurofibromatosis
Pulse for AF
Face
Gaze evoked nystagmus (in the direction of gaze), INO, RAPD
Speech (Count 1 to 20; British Constitution; West Register Street)
Cerebellar speech jerky, explosive and loud; irregular syllables
CNs
CPA
LMS
III nerve palsy in Benedikts syndrome
Xanthelesma
Lower limbs
Dysmetria and intention tremor for toe to finger test
Dyssynergia for heel-shin test
Dysdiadochokinesia for foot tapping test
DM dermopathy
Sit up with hands folded and tests for pendular jerks
Gait
Broad based gait with veering towards the side of the lesion
Request to test visual fields for hemianopia
Bilateral
Upper Limbs
Cs of cerebellar (dysmetria, dysdiadochokinesia and dyschronometria)
Sensory loss of temperature/pain for syringomyelia
Parkinsonism
NF features
Alcoholic features dupytrens contracture, stigmata of chronic liver disease
Face
CNs
Bilateral CPA tumor
Multiple sclerosis
Eyes
Gaze evoked nystagmus
KF rings
INO, RAPD
Mouth
Gingivial hypertrophy
Macroglossia
Telengiectasia
Parotidomegaly
Goitre
Speech
Cerebellar speech
Hoarseness of voice
Lower limbs
Cerebellar signs
Clawing of toes (Friederichs ataxia)
Sit truncal ataxia and pendular jerks
Gait cerebellar gait
110

Presentation
Unilateral
Sir, this patient has a right sided unilateral cerebellar lesion as evidenced by presence of a right dysmetria, dysdiadochokinesia and
dyschronometria of the right upper limb. The right lower limb also demonstrates presence of right dyssynergia on heel shin test,
with right dysmetria and intention tremor on toe-finger test and dysdiadochokinesia. This is associated with a gazed evoked
nystagmus on rightward gaze with a broad based gait with veering towards the right. I did not detect any cerebellar speech or any
truncal ataxia.
There are no associated cranial neuropathies. In particular there was no evidence of any cerebello-pontine angle lesion with CN V,
VI, VII and VIII intact. (There are also no signs of neurofibromatosis such as neurofibromas or caf-au-lait spots.) There is also no
evidence of lateral medullary syndrome or III nerve palsy to suggest Benedikts syndrome. There is also no pronator drift on the
right to suggest a right ataxic hemiparesis.
Patient is in sinus rhythm and not in atrial fibrillation with no xanthelesma or diabetic dermopathy. There are also no bruises to
suggest overanticoagulation.
There are no signs of Parkinsonism to suggest presence of multiple system atrophy. There are also no associated features of
multiple sclerosis such as RAPD or INO.
I would like to complete the examination by looking at
1. The patients temperature chart for fever (abscess in posterior fossa)
2. Visual fields for a left sided hemianopia, which can occur with a right posterior circulation stroke
3. I would also like to do a fundoscopy for papilloedema for a SOL in the right cerebello-pontine lesion as well as for optic atrophy
from demyelinating disease.
In summary, this patient has got an isolated right cerebellar lesion. The differential diagnoses include cerebral vascular infarction or
haemorrhage or a space-occupying lesions such as a mitotic lesion or an abscess.
Bilateral
Sir, this patient has bilateral cerebellar lesions as evidence of dysmetria with intention tremor bilaterally associated with
dysdiadochokinesia. Similar findings were also present on examination of the lower limbs. There is also presence of multidirectional gaze evoked nystagmus associated with a cerebellar speech, truncal ataxia and a broad based gait.
There is no evidence of bilateral CPA lesion with no CN V, VI, VII and VIII abnormalities. Patient is in sinus rhythm and not in AF
with no xanthelesma or diabetic dermopathy.
There is no evidence of KF rings to suggest presence of Wilsons disease. There is also no RAPD or INO to suggest multiple
sclerosis. There is also no gingival hypertrophy to suggest chronic phenytoin use. Patient has no goitre or features of
hypothyroidism such as a cream and peaches complexion, no hoarseness of voice or macroglossia. There are also no features of
chronic ethanol ingestion such as Parotidomegaly, dupytrens contracture or stigmata of chronic liver disease. There is no
associated Parkinsonism signs to suggest multiple system atrophy such as presence of cog-wheeling or leadpipe rigidity. There are
also no neurofibromas present to suggest presence of NF type 2. Patient is also not cachexic looking and there is no clubbing to
suggest underlying malignancy.
I did not detect any telengiectasia to suggest presence of Ataxia telengiectasia and there is pes cavus to suggest Friederichs
ataxia. (Think of Wilsons, MS, Phenytoin, Hypothyroid, Alcohol, Parkinsonism, NF, paraneoplastic, telangiectasia and FA)
I would like to complete the examination by
1. Looking at the temperature chart for fever
2. Performing a neurological examination of the lower limb to look for spastic paraparesis
3. I would also like to do a fundoscopy for Optic atrophy, which may suggest demyelinating disease.
In summary, this patient has bilateral cerebellar syndrome. Possible causes include drugs such as phenytoin, metabolic conditions
such as hypothyroidism, chronic ethanol ingestion, paraneoplastic conditions and infection such as enteroviruses and bilateral
cerebellar strokes.

111

Questions
What are the differential diagnoses for a unilateral cerebellar syndrome?
Isolated
Cerebrovascular accident infarction or haemorrhage
SOL in posterior fossa abscess or mitotic (primary vs secondary)
Associated
CN
CPA and/or neurofibromatosis
Lateral medullary syndrome
Jugular foramen (Arnold-Chiari or Dandy-Walker)
Benedikts syndrome
Ataxic hemiparesis (lacunar stroke)
Parkinsonism in Multiple system atrophy
Demyelinating such as Multiple sclerosis
What are causes of bilateral cerebellar syndrome?
Acquired
Infection
Viral HIV, Enteroviruses
Spirocheatal Lymes and Tabes dorsalis
Others Toxoplasmosis and CJD
Metabolic
Wilsons disease
Hypothyroidism
Drugs
Phenytoin and Carbamazepine
Lithium
Alcohol
Causes bilateral cerebellar signs
Causes isolated lower limb cerebellar signs
Affects the anterior vermis
Due to thiamine deficiency
Multiple system atrophy
Neurofibromatosis type 2 with bilateral CPA tumor
Bilateral Strokes
Paraneoplastic Ca lung or ovary
Hereditary
Ataxia telangiectasia
Autosomal recessive
Childhood with death by 20s or 30s
Ataxia, choreathetosis and telengiectasia on the face, eras and conjunctiva and skin crease
Low IgA with recurrent chest infections and lymphoreticular malignancy
Friederichs ataxia
Scoliosis, pes cavus
Spastic paraparesis, dorsal column loss, absent ankle jerks
What are the signs of a midline lesion (cerebellar vermis) and what are the causes?
Signs : truncal ataxia, abnormal heel-toe walk test, cerebellar speech
Causes : Midline tumor, paraneoplastic
What are the causes of cerebellar signs with spastic paraparesis?
Friederichs ataxia
Spinocerebellar ataxia
Arnold-Chiari Malformation
Lesion at the craniospinal junction eg meningioma
Syringomyelia
Multiple sclerosis
Syphilitic meningomyelitis

112

How are cerebellar signs located?


Limb ataxia = cerebellar lobes
Gait ataxia = anterior vermis
Truncal ataxia = posterior vermis
What are the differences between cerebellar and sensory ataxia?
Cerebellar
Sensory
Site
Cerebellar
Posterior column, nerves
Tone
Reduced
Normal
Reflexes
Normal or pendular
Reduced
Sensory
Normal
Reduced
Sphincter disturbance
Nil
Affected when posterior column involved; overflow incontinence
How would you investigate?
Imaging MRI brain
Blood tests according to the causes
How would you manage? Depends on underlying cause

113

50. Chorea
(Beware the Parkinsonism with dyskinesia!)
Approach
1. Introduce, sit the patient
2. Lift up hands
a. Involuntary athetoid movements/choreiform movements
b. Choreic posture
c. Dish spooning
d. Pronator drift
e. Milk-maids grip
f. Look for wasting of the muscles and joint deformities
g. Look for erythema marginatum and subcutaneous nodules
h. Check for Parkinsonism
i. SLE signs
3. Check for long tract signs especially if hemiballismus or one sided
4. Eyes
a. KF rings
b. Conjuctival suffusion
c. Dysthyroid eye disease, nystagmus
d. Plethoric facies
e. Darting tongue
f. Goiter
5. Walk the patient effeminate gait, Parkinsonian gait
Presentation
Sir, this patient has chorea/athetosis affecting her left hand. I say this because of presence of brief, abrupt, irregular, quasipurposeful movements of the left hand with writhing and twisting movements (athetosis). There is choreic posturing of the left hand
with a flexed wrist and an extended mcpj; with dish spooning and milk maid grip, associated with darting tongue and an effeminate
gait.
There were no features of Parkinsonism to suggest that dyskinesia is secondary to L-dopa therapy. There was no evidence of
erythema marginatum or subcutaneous nodules which can occur in rheumatic heart disease. There is also no cutaneous rash to
suggest SLE. There is also no pronator drift.
There are also no KF rings or nystagmus to suggest Wilsons disease. There are no signs of polycythemia rubra vera as I did n ot
notice any plethoric facies, conjunctival suffusion or pruritic scratch marks. There are also no goiter or thyroid eye signs.
I would like to complete the examination by performing a cardiovascular examination to look for evidence of rheumatic heart
disease, a mini-mental state examination for dementia as this occurs in Huntingtons chorea, as well as take a drug history of
neuroleptics and L-dopa and a past history of encephalitis.

114

Questions
What are the different types of movement disorders that you know about?
Tremors
Resting tremor of Parkinsonism
Intention tremor of Cerebellar
Postural tremor of outstretched hands
Anxiety
Thyrotoxicosis
Alcohol
Drug induced salbutamol, terbutaline, theophylline, Li
Drug withdrawal BZD, opiates
Familial
Chorea (globus pallidus)
Athetosis
Hemiballismus (subthalamic nucleus)
Infarct
Others abscess, tumor, MS, AVM
Search for CVS risk factors
Rx haloperidol, treat CV risk factors and Sx eg contralateral thalomotomy or pallidotomy
Orofacial dyskinesia
Secondary to antipsychotics usually, in pts with SZ
One of the 4 EPSE
Acute dystonia (oculogyric)
Parkinsonism
Akathisia (restless legs syndrome)
Tardive dyskinesia (or orofacial dyskinesia)
What are the causes of choreathetosis?
CVA/tumors affecting the globus pallidus (Benedikts syndrome - III)
Metabolic Wilsons disease
Endocrine Hyperthyroidism, post-hyperglycemia
CTDs SLE
Polycythemia
Rheumatic heart disease Sydenhams chorea
o Most recover within one month
Huntingtons Chorea
Drugs neuroleptics, L-dopa, phenytoin, OCPs
Post encephalitis
CO poisoning
What is Huntingtons disease?
Young adult, chorea and dementia
AD, Chr 4, CAG trinucleotide repeats

115

LOWER LIMBS!
51. Lower Limbs Overview
Pes cavus
CMT
Spina Bifida
Poliomyelitis
Spinal cord tumours
Freiderichs ataxia/spinocerebellar degeneration
Syringomyelia
Cerebral Palsy
Muscular dystrophies
Fasciculations (LMN type, MND)
Wasting
Bilateral
Proximal weakness
Wasting distally (Pes Cavus, Peripheral neuropathy)
Spastic paraparesis (L&P 104)
Cerebellar (MS/FA/Syphilitic meningomyelitis/Craniospinal jn/SCA)
Sensory level (Lumbar/Thoracic/Cervical ULs/Above high Cx, CP)
Dorsal Column Loss (SACD/Taboparesis/MS/FA)
Mixed (Babinski + absent reflexes see below)
Friederichs ataxia
SACD
Tabo-paresis
MND
UMN + cauda equina or peripheral neuropathy(CVA+alcoholic/DM)
MND
Flaccid paraparesis
Wasted
GBS/CIDP/HMSN/Hansens
Poliomyelitis
Spina Bifida
No wasting
Peripheral GBS, HMSN, paraneoplastic, paraproteinemia, amyloid
Cord compression
Others Miller-Fisher, MG, Periodic paralysis, botulism/diphtheria/organophosphate/Hg/Pb, AIP(BP)
MND
Bilateral Footdrop
Unilateral
Foot Drop
Bilateral (Peripheral neuropathy motor predominant, flaccid, spastic)
Unilateral
Peripheral neuropathy, CPN
Sciatic nerve
Root or anterior horn cell
Look for complications- trophic ulcer, interventions walking callipers
Unilateral Peripheral neuropathy, lumbosacral plexus, polyradiculopathy, polio (LMN)
Brown-sequard (UMN)
Diabetic amyotrophy
Hemiparesis (UMN)
Sensory loss
Peripheral neuropathy
Mononeuropathy
Polyradiculopathy
Lumbosacral Plexus
Dissociated sensory loss, spinal cord level

116

Others
Gait
Cerebellar
Unilateral
Cerebellar Vascular, MS, SOL eg abscess or tumour
Combined Lateral medullary syndrome, CPA tumour, ataxic hemiparesis
Bilateral
hypothyroidism, Wilson, Alcoholic cerebellar degeneration(spares the ULs), drugs phenytoin, paraneoplastic,
Parkinson plus
large CVAs, SOL, MS
plus all causes of spastic and cerebellar
Midline paraneoplastic, midline tumour
Spastic and Ataxic combined
Spinocerebellar degeneration
Friederichs ataxia
Multiple sclerosis
Syphilitic meningomyelitis
Craniospinal junction abnormalities Arnold-Chiari, meningioma
Non conforming
Myasthenia Gravis
Mononeuritis multiplex
Motor neurone disease
Giddiness/Unsteadiness protocol
Giddiness
o Cerebellar
o Vestibular
o Postural BP
Unsteady gait
o Cerebellar
o Parkinsonism
o Sensory ataxia (Proprioception)
o Others hemiplegic gait, cervical myelopathy etc etc
Examination for unsteady gait
o Start with Lower Limbs FIRST
As per LL protocol
Concentrate on cerebellar, sensory ataxia and Parkinsonism
Examine the gait!
o Proceed with Parkinsonism protocol if Parkinsonian gait
o Proceed with cerebellar protocol if cerebellar signs

117

52. Flaccid Paraparesis


Examination
Complete the LL examination
Commonly
HMSN
Polio
(infantile hemiplegia)
Spina Bifida
Cauda Equina Syndrome
GBS/CIDP
MND (see spastic paraparesis)
Diabetic amyotrophy (See proximal myopathy)
Concentrate on
Ataxia Miller Fisher Variant, Tick Paralysis
Sensory
No sensory abnormalities
Myopathies
Neuromuscular
Nerves certain conditions eg GBS, multifocal motor neuropathy
Anterior Horn Cell
Glove and stocking
Peripheral neuropathy
HMSN, paraneoplastic
Mild and patchy = GBS
Sensory level (Acute)
Cord compression
Cord infarction
Transverse myelitis
L5 and S1 sensory loss in spina bifida
Typical features of HMSN
Pes cavus, clawing of toes, contractures of Achilles tendon, inverted champagne bottles (wasting of distally and stops
abruptly at the lower one third of thighs; also similar distal wasting distally in the ULs)
LMN reduced tones and no clonus, reduced reflexes and downgoing plantar response, weakness, bilateral footdrop
Sensory no sensory or mild glove and stocking
Gait high steppage gait of foot drop
Marked deformity with minimal disability
Others
Feel for thickened nerves (lateral popliteal nerve)
Examine the hands for small muscle wasting and clawing
Examine spine for scoliosis
Feel for thickened Greater Auricular nerves
Wheelchair, calipers
Examine
Back
Kyphoscoliosis
Spina bifida scars, tuft of hair, dimples, sinus or naevus
Per rectal examination
Saddle anaesthesia and cauda equina syndrome
Incontinence fecal and urinary
Upper limbs
CNs- fatiguibility, GBS (bilateral VII)
Functional aids

118

Presentation
Obvious disease
HMSN
Sir, this patient has got HMSN/CMT as evidenced by
Bilateral pes cavus with clawing of toes and distal wasting of the lower limbs with a inverted champagne bottle
appearance; there is hypotonia with reduced reflexes and downgoing plantar responses a/w weakness of the lower
limbs of power 4/5 with bilateral foot drop; there is no associated sensory disturbance; she has a high steppage gait
form bilateral foot drop and is able to walk independently inspite of the marked feet deformity; I also noticed presence
of wasting and clawing of the upper limbs; there is no palpable thickened lateral popliteal nerve.
I would like to complete my examination by examining the spine back for scoliosis and palpate for other sites of
thickened nerves
Mention walking aids or wheelchair
Polio
Sir this patient has monoparesis of the right LL most likely due to polio
A shortened right lower limb associated with wasting. It is hypotonic with reduced reflexes and downgoing plantar
response and is flaccid with a power of 3/5. There is no sensory weakness.
There is no UMNs or shortened wasted right UL to suggest infantile hemiplegia
Examination of the back did not reveal any cutaneous signs of spina bifida.
Mention any walking aids/wheelchair
Not so obvious
Sir, this patient has got flaccid paraparesis as evidenced by
Presence of hypotonia with reduced reflexes a/w with downgoing plantar responses bilaterally; I did not detect any
fasciculations. There is weakness of the LLs with a power of 3/5. There is no associated cerebellar signs in the LLs and no
sensory loss to pin prick, propioception and vibration.
Complete my examination
Back
Per rectal
ULs for ataxia, flaccid paresis
CNs for cranial neuropathies

119

Questions
What are the causes of flaccid paraparesis?
Acute myopathies
Inflammatory myopathy (polymyositis, dermatomyositis)
Rhabdomyolysis (extreme exertion, drugs, viral myositis, crush injury etc.)
Acute alcoholic necrotizing myopathy
Periodic paralyses (hypokalemic, hyperkalemic)
Metabolic derangements (hypophosphatemia, hypokalemia, hypermagnesemia)
Thyroid or steroid myopathy
Neuromuscular
Myasthenia gravis
Botulism
Tick paralysis
Other biotoxins (tetradotoxin, ciguatoxin)
Organophosphate toxicity (can also cause neuropathy)
Lambert-Eaton Myasthenic Syndrome (LEMS)
Nerve
Diphtheria
Porphyria
Drugs & Toxins (arsenic, thallium, lead, gold, chemotherapy cisplatin / vincristine)
Vasculitis (incl. Lupus, polyarteritis)
Paraneoplastic and Paraproteinemias
Multifocal motor neuropathy
Nerve roots
Guillian Barre Syndrome
Lyme disease
Sarcoidosis
HIV
other viruses (CMV, VZV, West Nile)
Cauda equina syndrome (lumbar disc, tumour, etc.)
Plexus lesions (brachial plexitis, lumbosacral plexopathy)
Anterior Horn Cell (motor neuron diseases):
Amyotrophic lateral sclerosis (ALS) with UMN findings
Poliomyelitis
Kennedys disease (spinobulbar atrophy / androgen receptor gene)
other spinomuscular atrophies (inherited)
Anterior spinal artery syndrome (with grey matter infarction)
Spinal Cord (corticospinal tract diseases):
Inflammatory (Transverse myelitis)
Subacute combined degeneration (B12 deficiency)
Spinal cord infarction
other myelopathies (spondylosis, epidural abscess or hematoma
Brain
Pontine lesions (eg. Central pontine myelinolysis, basis pontis infarct or bleed)
Multifocal lesions (multiple metastases, dissemination encephalomyelitis [ADEM], multiple infarcts or hemorrhages eg.
DIC, TTP, bacterial endocarditis)

120

What is Charcot Marie Tooth disease?


Hereditary sensory motor neuropathy
Consisting of 7 types of which types 1,2 and 3 are the most common types
Type 1 A demyelinating neuropathy, aut dominant, absent tendon reflexes, enlarged nerves; Chr 17
Type 2 An axonal neuropathy, aut dominant (mild and present later), normal deep tendon reflexes, nerves not enlarged;
Chr 1
Type 3 rare, hypertrophic neuropathy of infancy, thickened nerves, aut recessive (Dejerine Sottas disease)
Physical findings
Above plus
Others optic atrophy, retinitis pigmentosa and spastic paraparesis
Ix
Rule out other causes of neuropathies
EMG/NCT
Biopsy
Genetic testing
Mx
Eductaion and counselling and family screening
PT/OT, AFOS
Medical Rx pain relief, avoid obesity
Surgical treatment
Px
Normal life expectancy
Disease usually arrest in middle life
Disability varies
Dy/Dx of hereditary disease
Hereditary amyloidosis
Refsums disease accumulation of phytanic acid
Fabrys disease deficiency of alpha galactosidase
What is poliomyelitis?
Enterovirus, picorna virus, with IP of 5-35 days, oro-fecal route or contaminated water, 3 serotypes
Replicate in the nasopharynx and GIT and then to lymphoid tissue and then hematological spread with predilection to the
anterior horn cells of the spinal cord or brainstem with flaccid paralysis in spinal or bulbar distribution
4 forms
Inapparent infection
Abortive nauseas, vomiting and abdominal pain
Nonparalytic above plus meningeal irritation
Paralytic paralysis and wasting; bulbar or spinal distribution
Occasionally, can get postpolimyelitis syndrome which results in weakness or fatigue in the initially involved muscle
groups 20-40 years later
Ix
Viral c/s from stool, throat and CSF
Antibodies
Mx
Educationa and counselling
Non-medical
PT/OT
Care of limbs
Medical
Rx complications
Pain
Respiratory failure
Clear bowels
Prevention
Inactivated polio vaccine Salk vaccine which is administered parenterally
Oral live vaccine can result in poliomyelitis in immunodeficient individuals
Dy/Dx
Spina bifida
Infantile hemiplegia hypoplasia of the entire side of the left side with UMN sign on the affected side

121

What is Spina Bifida?


Incomplete closure of the bony vertebral canal with similar anomaly of the spinal cord
Usually in lumbosacral region, can also involve the cervical region and is associated with hydrocephalus
Look for
Scars, tuft of hair, dimples, sinus, naevus, lipoma
Asymmetric LMN signs of LLs
L5 and S1 dermatomal sensory loss
Bladder involvement
X-ray: sacral dysgenesis, laminar fusion of the vertebral body. Scoliosis
Multifactorial aetiologies, with folic deficiency and use of Na Valproate, siblings with spina bifida has higher risk
Prevented with use of folic acid early in pregnancy
Can be tested with amniotic serum AFP, serum AFP or USS
What is cauda equina syndrome?
th
The cauda equina refers to the nerve roots that are caudal to the termination of the spinal cord; any lesion below the 10
Thoracic vertebrae
Low back pain, unilateral or bilateral sciatica, saddle anaesthesia, bladder and bowel disturbances and variable motor and
sensory LL abnormalities
Causes trauma, PID, spondylosis, abscess, tumor (ependymoma and NF)
Anatomy
Spinal cord starts from the foramen magnum to the level of L1 vertebrae
Add 1 to Cx vertebrae
Add 2 to Tx vertebrae 1-6
Add 3 for Tx vertebrae 7-9
T10 and T11 vertebrae = lumbar segments
T12 and L1 = sacral and coccygeal
Conus medullaris = T9 to L1 vertebrae
Conus Medullaris
Cauda equina
Presentation
Acute
Chronic
Reflexes
Knees preserved; ankle absent
Both knees and ankles absent
Motor
Spastic para; symmetrical
Flaccid para; asymmetrical
Sensory
More LBP, less radicular
Less LBP, more radicular
Sensory
Perianal
Saddle
Impotence
Frequent
Less frequently
Sphincter
Occurs early
Occurs late
What is Guillain Barre Syndrome?
Auto immune, antecedent Campylobacter infection
Bimodal young adults or the elderly
Motor, sensory and autonomic dysfunction
Progressive ascending muscle weakness, variable patchy sensory loss, hyporeflexia and autonomic disturbances such as
tachycardia and labile BP
Post GI or resp infection, 2-4 weeks of onset of symptoms which may progress over hrs to days and recovery over months;
complicated by respiratory failure
Subtypes
Acute inflammatory demyelinating neuropathy
Acute motor axonal neuropathy
Acute motor-sensory axonal neuropathy
Miller Fisher Syndrome (Ataxia, areflexia and ophthalmoplegia; antiGQ1b Ab)
Acute panautonomic neuropathy
Ix
CSF shows albuminocytologic dissociation (<10 mononc cells and high prot)
AntiGQ1B Ab in MFS and anti GM1 implies poorer Px
NCT demyelination
FVC (15-20ml/kg < or NIF 25cmH2O<) = may require ventilation
Mx
Emergency
ABCs, and pacing maybe required IVIG Plasma exchange Steroids
PT/OT
Prevention Cx DVT prophylaxis
Px
Most 85% will have full recovery by 6-12 months
Complications from respiratory failure and cardiac dysrythmias and labile BP

122

53. Spastic Paraparesis


(Think: Cerebellar, sensory level, dorsal column, mixed, ULs and others)
Presentation
Sir, this patient has spastic paraparesis as evidenced by
Increased tone and clonus
Hyper- reflexia with upgoing-plantars
Weakness of the lower limbs of power 4 with wasting
Complications of
o DVT
o Pressure sores
o Urinary catheter, diapers
Functionally
o Wheelchair by the bedside
o Walking aids, orthotic shoes
o Gait (Rhombergs sign positive with dorsal column loss, broad based gait)
This is associated with
1. Cerebellar signs
o Dyssynergia on heel-shin test
o Dysmetria and intention tremor on toe to finger test
o Dysdiadochokinesia on foot tapping test
o There is no sensory level and no dorsal column loss
o The possible differential diagnoses includes
Spinocerebellar degeneration such as FA
Young
Pes cavus, loss of ankle reflexes, dorsal column loss
Multiple sclerosis
Craniospinal junction abnormalities
Congenital - AC malformation
Acquired - meningioma at the CS junction, syringomyelia
Lewitic meningomyelitis
o Hence I would like to complete the examination by
Features of MS RAPD, INO and optic atrophy
Features of CS junction CNs, neck, dissociated sensory loss
Argyll Robertson pupils for lewitic disease
2. Sensory level at T4
o Loss of sensation to pinprick up to the level of the nipples bilaterally
o There are no cerebellar sign and dorsal column sensory system is intact
o Hence the lesion is likely due to spinal cord lesion at T4 level and possible etiologies are
Cord compression
Extramedullary (Root pains, LMN localised, spasticity early, no sacral sparing, abnormal CSF)
o Vertebral spondylosis, trauma, PID, tumor, infection
o Extradural abscess, metastases, lymphoma
o Intradural meningioma, NF
Intramedullary (root pains rare, LMN signs extend over several segments, late spasticity, may
sacral spare, normal CSF)
o Syringomyelia
o Tumor glioma, ependymoma
o Hematomyelia
Cord infarction
Anterior spinal artery thrombosis
Vasculitis (PAN, syphilis)
Thoracic/AAA and dissection
Causes of cord compression
Myelitis
Infective
o Mumps, measles, EBV, HIV
o Mycoplasma, syphilis, TB
Neoplastic - Carcinomatosis meningitis
Nutritional B12
Demyelinating MS
o I would like to complete the examination
PR for saddle anaesthesia and a lax anal tone, although I did notice that he is not on ant urinary catheter or
diapers
Back for any scars or bony tenderness
ULs (if sensory level is at or above the ULs)

123

3.

Dorsal column loss


o With loss of proprioception involving the first MTPJ and ankle joints
o As well as loss of vibration sense up to the level of the knees
o The sensation to pinprick is intact and there are/no cerebellar signs
o The possible etiologies include
Spinocerebellar degeneration
Multiple sclerosis
Subacute combined degeneration of the cord
Taboparesis
o Hence I would like to complete the examination by
Features of MS
Features of SACD pallor, splenomegaly
Argyll Robertson pupils

4.

Upper limbs
o Intact cerebellar and sensation to pinprick, proprioception and vibration (sensation may be lost)
o I would like to examine the upper limbs for
LMN signs, wasted hands
Syringomyelia, cervical myelopathy, MND
Inverted supinator jerks
C5-6 lesion
UMN signs
Bilateral strokes, high cervical myelopathy

5.

Absent ankle jerks +/- knee jerks


o Fasciculations MND
o Cerebellar SCD (FA)
o Dorsal column loss SACD, Taboparesis
o Conus medullaris lesion
o Combined pathologies
Cord compression and pre-existing peripheral neuropathy
DM and Cx myelopathy
Alcohol and Cord compression
Cx and lumbar myelopathies

6.

Others Cerebral palsy, parasagittal falx meningioma

124

Questions
What is spinocerebellar ataxia?
A inherited disorders with multiple subtypes >20
Cs by cerebellar and spinal degeneration, slowly progressive with atrophy of cerebellum
Ataxia
o Acquired (see cerebellar)
o Hereditary
AD
Some involve trinucleotide repeats with high penetrance and anticipation
AR (FA)
X-linked
What is Friedreich Ataxia?
Hereditary ataxia, autosomal recessive, Ch 9, trinucleotide repeat
Cs by
o Symptomatic - during childhood and teenagers, inability to walk by 20s; onset <25
o Physical
pes cavus, distal wasting, spastic LL (pyramidal weakness), cerebellar signs, dorsal column loss (marked
loss of cells in the dorsal root ganglion) and absent knee and ankle reflexes (degeneration of peripheral
nerves)
Scoliosis
Request for
o Spine - scoliosis
o CVS HOCM
o Fundoscopy optic atrophy
o Urinalysis for glycosuria
o IQ intellectual deterioration
Differential diagnosis for spasticity, cerebellar and dorsal column loss is Multiple sclerosis; note that multiple sclerosis usually
has increased reflexes
Forme fruste of this condition is pes cavus or hammer toes without other signs
Dx Hardings criteria
o Clinical as above
o Ix small or absent sensory nerve action potentials, increased motor conduction velocity
Other well know conditions for spinocerebellar degeneration
o Refsums disease (elevated serum phytanic acid from defective lipid alpha oxidase optic atrophy, retinitis
pigmentosa, cardiomyopathy, ichthyosis and ataxia)
o Olivopontocerebellar degeneration
o Machado-Joseph disease (CAG repeats)
What is Multiple sclerosis?
See History Taking Notes

125

What is subacute combined degeneration of the cord?


Progressive disorder due to Vit B12 deficiency with degeneration of the corticospinal tracts (pyramids/lateral columns) and
dorsal column of the spinal cord
It also affects
o Nervous system CNs, peripheral neuropathy, transverse myelitis, dementia, psychiatric
o Pernicious anaemia
Typically patient feels paraesthesia
B12 deficiency
o Low intake, Vegan
o Impaired absorption
Stomach
Gastrectomy
Pernicious anemia
o Anti parietal cell antibody
o Anti IF antibody
Small intestines
Ileal disease (Crohns disease), coeliacs disease
Bacterial overgrowth
Fish tapeworm
Colchicine
Ix
o FBC macrocytic anaemia
o B12 and folate
o Anti IF Ab and Anti Parietal cell Ab
o Schilling test
IM B12 to saturate all B12 protein binding sites
PO Co-B12
If >10% excretion, normal
If <10%, abnormal
Repeat with oral IF
o If normal, means IF deficiency
o If abnormal, Rx with antibiotics
If normal = bacterial overgrowth
If abnormal = terminal ileal disease
o Screen with OGD if pernicious anemia, with higher risk of Gastric Ca
o MRI of spinal cord degeneration of the lateral and dorsal columns
Rx with parenteral B12 or PO B12 high doses
Px may recover if treated early; otherwise damage is usually permanent

126

What is taboparesis?
Cs
o Lightning pains electric shocks in the limbs, throat, stomach or rectum
o Physical
Spasticity with dorsal column loss (high stoppage gait), absent ankle jerks
Charcots joints, trophic ulcers
LL before ULs, rarely ULs involved first (= cervical tabes)
Incontinence and constipation
Argyll-Robertson pupils
Due to lewitic disease, neurosyphilis of which:
o Acute syphilitic meningitis
o Acute transverse myelitis
o Meningovascular disease (stroke in young patient, CN abnormalities)
o Tabes dorsalis
3 stages
Pre-ataxia
Ataxia
Paresis (= taboparesis)
o Taboparesis
o Generalised paralysis of the insane (GPI)/Dementia paralytica
Chronic progressive frontoparietal meningoencephalitis with atrophy
Dementia which classically progresses to grandeur and delusions
Trombone tremor (hands, lips and tongue)
o Gummata in the CNS
Caused by spirochetal infection, Treponema pallidum
Stages
o Primary painless chancre
o Secondary maculopapular rash, acute syphilitic meningitis
o Tertiary
Neurosyphilis obliterative small vessel endarteritis, affecting the vasa vasorum
Cardio syphilis
Gummatous syphilis
o Quaternary
Fulminant anergic necrotising encephalitis in HIV patients
Ix with VDRL/RPR, TPHA/FTA
o VDRL
Non-specific
False positive (EBV, malaria, SLE, RA, pregnancy, non syphilitic treponemal infection)
Titre use to monitor treatment and reinfection
o TPHA/FTA
Specific
Once positive, will remain positive even after treatment
Rx
o Symptomatic treatment
Lightning pains analgesia, TCAs, carbamazepine
Ataxia PT/OT
Bladder avoid anticholinergics, self catheterisation
o Penicillin
Beware the Jarisch Herxheimer reaction
From toxins released from killed spirochaetes
Starts 3-4 hrs and peaks at 6-8 hrs
Fever, HR, RR, myalgia, lethargy
Rx with steroids 1 day before and with salicylates

127

What is motor neuron disease?


Cs
o Onset >40 years old usually, progressive
o Cs by presence of UMN and LMN of a single spinal segment, with motor dysfn involving at least 2 limbs or 1 limb and
bulbar
o No sensory deficit, no cerebellar, no extrapyramidal, no ocular involvement
Types
o Bulbar/pseudobulbar palsy (25%)
o Amyotrophic lateral sclerosis (50%)
UMNs and LMNs
UMN from degeneration of the lateral corticospinal tracts and Betz cells
LMNs from degeneration of CN nuclei or anterior horn cell
o Progressive muscular atrophy (25%)
Severe muscular dystrophy with distal wasting
Preserved deep tendon reflexes
Due to lesions in the anterior horn cells
o Primary lateral sclerosis (rare)
UMN progressing to LMN
Dy/Dx
o Cervical cord compression
o Syphilitic amyotrophy
o Anterior horn cell disease
Polio
Spinal muscular atrophy
No definitive Ix test
Mx
o Counselling and support
o Symptomatic
Dysphagia NG, PEG
PT, OT
Antidepressants
o Riluzole glutamate antagonist
o Rx complications
Pneumonia, UTI
Pressure sores
DVT
Poor Px progressive with median survival of 3-5 years
What are the clinical features of cervical myelopathy?
Symptoms exceeds signs and spasticity exceeds weakness
Cs
Motor
look for wasting and weakness of C5 and C6 level as this is commonest area for spondylosis involvement
passive abduction of the little finger (myelopathy hand sign)
spasticity of LL
Inverted supinator jerk
Sensory often no sensory loss but may have pseudoathetosis with loss of proprioception and vibration
What causes fasciculation?
Sign of LMN disease with denervation
Spontaneous firing of large motor units formed by branching fibres of surviving axons striving to innervate the muscles that
have lost their nerve supply
What is Lhermittes phenomenon?
Tingling or electric sensation that passes down the spine with flexion of the neck
Causes includes
Multiple sclerosis
Cervical myelopathy (spondylosis and tumor) also has similar sensation on extension of the neck i.e. reversed
Lhermittes sign
SACD
What are the various manoeuvres to elicit Babinskis sign?
Chaddocks manoeuvre (dorsolateral aspect of the sole of feet)
Oppenheims sign (pressure applied on the inner border of the tibia)
Gordons reflex (pinching the Achilles tendon)

128

54. Peripheral Neuropathy


(Think: Sensory, motor, or mixed. Are nerves palpable nerves?)
Presentation 1
Sir, this patient has predominantly sensory peripheral neuropathy as evidenced by
o Loss of sensation to pinprick and light touch and
o Impairment of vibration and joint position sense
o In a stocking distribution
The motor system is intact; I did not notice any
o Wasting or fasciculations of the lower limb muscles
o Tone and reflexes are normal with downgoing plantars
o Power is normal
OR
Sir, this patient has mixed motor and sensory peripheral neuropathy as evidenced by
o Loss of sensation to pinprick and light touch and
o Impairment of vibration and joint position sense
o In a stocking distribution
Associated with
o Wasting and fasciculations of the lower limb muscles
o Reduced tone and reflexes with downgoing plantars
o Diminished power of 4 in the lower limb muscles especially affecting plantarflexion, dorsiflexion and flexion and
extension of the knees bilaterally
AND
There is presence of
o Loss of hair on the lower half of the legs bilaterally
o No charcot joints
The most likely underlying aetiology is diabetes mellitus as I noticed
o Presence of diabetic dermopathy
o I screened for other possible causes:
No thickened nerves or hypopigmentation patch (leprosy)
Parotidomegaly, dupytren (chronic ethanol ingestion)
Not sallow (uremia)
Not pale (B12 deficiency)
Not cachexic and no clubbing of toes (paraneoplastic)
No symmetrical deforming polyarthropathy (RA)
No clinical features of acromegaly, hypothyroidism
I would like to complete the examination
o Gait (if not done) to look for high steppage gait (sensory ataxia)
o Upper limbs for distal sensory impairment although I noticed that there is no obvious wasting of the hands
o Urine dipstick for glycosuria (DM)
o Ask history
Drug history INH, nitrofurantoin, phenytoin, chloroquine, penicillamine, vincristine, cyclosporine A
Chronic ethanol ingestion
Presentation 2
Sir this patient has predominantly motor neuropathy as evidenced by
o Wasting and fasciculations of the lower limb muscles
o Reduced tone and reflexes
o With diminished power of 4 affecting knee flexion and extension as well as plantar and dorsiflexion
Sensation is intact with normal pinprick sensation, vibration sense and propioception.
The most likely aetiology in this patient is
o diabetes mellitus as I noticed presence of diabetic dermopathy in the lower limbs
o Other possible aetiologies for a predominantly motor peripheral neuropathy
Drugs cyclosporine A, Gold, penicillamine
Pb, Hg
Metabolic DM and AIP
Infectious/Inflammatory HIV, GBS, Amyloid, sarcoid
PAN
HSMN type 1

129

Questions
What are the causes of peripheral neuropathy (mixed, sensory and motor)?
DAMIT BICH
o Drugs
INH, nitrofurantoin, chloroquine
Penicillamine, gold, cyclosporin A, phenytoin
vincristine, cisplatinum
o Alcohol, Arsenic(Mees, raindrop pigmentation), Pb(wrists and Pb lines in gums), Hg
o Metabolic DM, Uremia, AIP
o Infectious - Leprosy, HIV, botulism, diphtheria
o Inflammatory GBS (look for facial diplegia), CIDP
o Tumor paraproteinemia, paraneoplastic (Ca Lung), Hodgkins
o B12, B6 and B1
o Infiltrative Amyloid (look for thickened nerves and autonomic), sarcoid
o Immunological PAN, SLE, RA
o Congenital HMSN, Refsums disease, porphyria
o Cryptogenic
o Hormonal Acromegaly, hypothyroidism, hyperthyroidism
o POEMS (Polyneuropathy, Organomegaly, Endocrinoapthy, Monoclonal gammopathy, Skin changes a/w
osteosclerotic myeloma)
(NB: DM can be sensory, motor or mixed)
What are the causes of a painful peripheral neuropathy? (DAB, CAP)
DM, Alcohol, B12 deficiency
Carcinoma, porphyria, Arsenic
What are the causes of thickened peripheral nerves?
Median nerve (wrist), ulna nerve (elbow), common peroneal nerve (head of fibula), Greater auricular nerve (neck)
CHAOS
o CIDP
o HMSN
o Acromegaly, Amyloid
o Others
LS DNR Leprosy, sarcoid, DM, Dejerine Sotta disease (hypertrophic peripheral neuropathy), NF, Refsums
disease (retinitis pigmentosa, optic atrophy, cerebellar and deafness, cardiomyopathy ad ichthyosis)
What are the causes of mononeuritis multiplex (separate involvement of more than one peripheral or cranial nerve by the same
disease)?
Endocrine
o DM, Hypertension, Acromegaly
AI
o RA, SLE, PAN, Sjogren, Churg-Strauss, Wegeners
Infection
o Leprosy, Lyme, HIV
Infiltrative
o Amyloid, sarcoid
Carcinomatosis
What are the types of neuropathy in DM?
Symmetrical sensory neuropathy (glove and stocking)
Predominantly motor, asymmetrical (diabetic amyotrophy)
Mixed motor and sensory peripheral neuropathy
Mononeuropathy
Mononeuritis multiplex
Autonomic neuropathy
What are the neurological complications of alcohol?
Wernickes (confusion, ophthalmoplegia, cerebellar, neuropathy)
Korsakoffs psychosis (recent memory loss and confabulation)
Cerebellar degeneration
Central pontine myelinosis
Epilepsy
Myopathy and rhabdomyolysis
Peripheral neuropathy

130

55. Charcots Joint


Examination
Inspection
Skin
DM dermopathy, callus, ulcer, amputations, loss of concavity of the foot arch, loss of leg hair, shiny skin;
Leprosy skin changes, i.e. hypopigmented macules
Muscle - wasting
Bone - Joint involvement and stage of Charcots
Notice any nearby foot orthoses
Palpate
Joint for tenderness, osteophytes and crepitations, passive ROM for hypermobility
Feel for thickened nerves
Feel pulses
Sensory testing pinprick, temperature; vibration and proprioception
Motor testing
Presentation
Sir, this patient has a right ankle Charcots joint. The right ankle is enlarged and deformed with crepitus and hypermobility of the
joint. It is not warm or tender. There is loss of sensation to pinprick in a stocking distribution and there is also loss of vibration and
proprioception up to the ankles.
In terms of function, I noticed the presence of an ankle-foot orthoses by the patients side. There is no muscle wasting to suggest
disuse atrophy.
The most likely aetiology is diabetes mellitus as evidenced by presence of DM dermopathy, with callus formation over the pressure
points of the feet and loss of concavity of the foot arch. There is also loss of skin hair on the lower limbs and the skin has a shiny
appearance. There is no hypoaesthetic, hypopigmented macules or palpable thickened nerves to suggest leprosy.
To complete the examination
Stand the patient for Rhombergs sign, walk for functional assessment
Check the back for meningocele
Check the upper limbs for dissociated sensory loss
Examine the eyes for Argyll-Robertson pupils
Urine dipstick for glycosuria
Check for signs for chronic ethanol ingestion
In summary, this patient has a right ankle Charcots joint secondary to diabetes mellitus and able to ambulate with an ankle foot
orthosis.

131

Questions
What is Charcots foot?
It is a chronic, progressive, degenerative, neuropathic arthropathy resulting from a disturbance from the sensory innervation of the
affected joint.
What are the causes of a Charcots foot?
Diabetes mellitus (toes and ankles)
Leprosy
Alcoholic neuropathy
Tabes dorsalis (hips and knees)
Myelomeningocele
Syringomyelia (upper limbs eg shoulder)
Others HSMN, congenital insensitivity to pain
What are the stages of Charcots foot?
Atrophic form usually forefoot with osteolysis of the distal metatarsal; X-ray shows MT resembling a pencil point
Hypertrophic form (mid or rear foot and ankle); Eichenholtz classification system:
Stage 0 Clinical stage with signs and symptoms but no joint deformity yet
Stage 1 Acute (Developmental or fragmentation stage)
Periarticular fracture with joint dislocation with unstable deformed foot
Tender, red and swollen, mimicking infection (but afebrile, normal TW and good DM control and no break in skin) or
gout
Stage 2 Subacute (Coalescence stage)
Resorption of bone debris
Stage 3 Chronic (Reparative stage)
Restabilization with fusion of the involved fragments
Enlarged and deformed, non tender
What is the pathogenesis of Charcots foot?
2 theories:
Neurotraumatic loss of pain sensation and proprioception combined with repetitive and mechanical trauma to the foot
Neurovascular Autonomically stimulated vascular reflex that causes hyperemia and periarticular osteopenia with contributory
trauma
How would you investigate?
Establish the diagnosis
In acute stage
X-ray to rule out OM; MRI or Indium scanning for infection
FBC for a normal TW; ESR or CRP normal in Charcots joint
Stage the Charcots joint from radiographs
Demonstrate loss of protective sensation of the foot
Semmes-Weinsten (10-g or 5.07 gauge) monofilament
Applied with just enough pressure to bend the monofilament
Positive if 4 out of 10 sites affected
Establish the cause
FPG for DM, VDRL
How would you manage?
Education and counselling
Early Dx to prevent deformity
PT/OT immobilization with total contact cast initially, paying particular attention to proper foot care and footwear; with ankle
foot orthoses and custom made footwear
Medications
Symptomatic in the acute stages, bisphosphonates
Surgical in chronic stage eg foot stabilisation procedures, exostosectomy

132

56. Proximal Myopathy


Examination
On detecting proximal weakness, proceed to test sensation to r/o neuropathic weakness; skip the cerebellar; waddling gait
If unilateral proximal weakness, think of diabetic amyotrophy (a/w pain and sensory impairment)
Check the ULs
Acromegaly, Cushings
Dupuytren contracture
Dermatomyositis/Polymyositis
Proximal weakness
Check the Face
Eyes for MG
Cushings, Acromegaly, Thyroid
Parotids
Presentation
Sir this patient has proximal weakness of the upper and lower limbs that is due to proximal myopathy.
There is presence of weakness with a power of 4 on the upper and lower limb girdle muscles. I was able to overcome his abduction
of the arms and he has difficulty standing from a sitting/squatting position. There is presence of a waddling gait.
There is no evidence of any sensory involvement. There is also no fatigability.
There were no features of
Dermatomyositis/polymyositis
Acromegaly/Cushings/Thyroid
Chronic ethanol ingestion Dupuytren and parotidomegaly
Sarcoid lupus pernio
Cancer cachexia, clubbed
I would like to take a drug history
Cholesterol lowering drugs
Corticosteroids
Cyclosporine A
Chloroquine

133

The possible etiologies include:


Congenital
Duchennes
Sex linked, young male child, onset 3-4yrs
Pseudohypertrophy of the calves
Proximal weakness Gowers sign, usually cannot ambulate by 15yrs
No facial involvement
Low IQ
Dilated cardiomyopathy
Beckers
Similar but less severe to Duchenne
Can ambulate beyond 15 years
rd th
Usually onset 5-15 but maybe 3 /4 decades
th th
Majority survive to 4 /5 decades
Dx Western blot of muscle biopsy abnormal/reduced dystrophin
Limb Girdle
AR, 10-30 yrs old, progressive with severe disability in 20yrs
Shoulder and pelvic girdle affected
Deltoids are spared initially pseudohypertrophy
Biceps and brachioradialis are involved late
Hip flexors and glutei are weak
Early wasting of medial quads and tibialis anterior with lateral quads and calves being pseudo hypertrophied
Face is never involved and normal IQ and lifespan
Normal muscle enzymes
Fascioscapular and oculopharyngeal see ULs
Acquired
P Polymyositis/Dermatomyositis, polymyalgia rheumatica
A Alcohol
C Cancer
H HIV
E Endocrine (Acromegaly, Cushings, Thyroid), ESRF
M Mitochondrial myopathy (Myopathy, External ophthalmoplegia, red ragged fibres and lactic acidemia), McArdles
syndrome (weakness after exercise)
P Periodic Paralysis
O osteomalacia
D Drugs
S - Sarcoid

134

57. Brown-Sequard Syndrome


Presentation
Sir, this patient has a Brow- Sequard syndrome with a right hemisection of the cord at level T10. This is because
Monoparesis of the right lower limb
Reduced vibration and proprioception on the same side
Loss of pain sensation on the contralateral limb below the level of T10
Loss of pain sensation on the right T10 dermatome
There are no complications of
DVT
Pressure sores
Charcots
Incontinence
Functionally, he has difficulty with ambulation as I notice a wheel chair at his bed side.
The causes for a hemisection of the spinal cord at the level of T10 on the right are
Trauma
PID
Spondylosis
Tumor
Abscess
Multiple sclerosis
Therefore, I would like to complete the examination
Examining the back
Per rectal examination for lax anal tone
Multiple sclerosis INO, optic atrophy with RAPD

135

58. Footdrop
Approach
Bilateral
LMN
Peripheral neuropathy (see peripheral neuropathy)
UMN
Cord lesion
Unilateral
Once dorsiflexion impaired
Check eversion (Common peroneal nerve = dorsiflex and eversion)
Check inversion and plantarflex = posterior tibial nerve
If foot drop and inversion and eversion is lost with normal plantarflexion, then L5 nerve root
If all gone = posterior tibial+common peroneal, sciatic nerve or plexus/roots
Knee flexion intact
Go to sensory
Peripheral neuropathy
Common peroneal nerve palsy (sensory loss over dorsum of the foot)
Determine if common peroneal nerve or
Deep branch only or
The superficial branch only
If knee flexion weak, test hip abduction and internal rotation and intact
Go to sensory
Sciatic nerve
If hip abduction and internal rotation is weak
Go to sensory
Nil = anterior horn cell
L4 and L5 dermatome = plexus or root
Once site is located, go for the cause
Note walking aids
Questions
Common peroneal nerve palsy (L4 and L5)
Anatomy
the sciatic nerve divides at the popliteal fossa into the tibial and common peroneal nerves
The posterior tibial nerves effects plantar flexion and inversion of the foot
The common peroneal nerves winds round the neck of the fibula, covered by s/c tissue and skin only and prone to
extrinsic compression
It then divides into the
Superficial branch: foot everters and sensation to lateral calves and dorsum of the foot
Deep branch : toe dorsiflexors and dorsiflexion of the ankle and sensation to the first interdigital web space
Therefore wasting of the peroneous and anterior tibialis muscles; weakness of dorsiflexion of the foot and eversion;
foot drop and high steppage gait and loss of sensory over the lateral aspect of the calf and dorsum of the foot
Causes of mononeuropathy (3 Sx and 3 Medical causes)
Trauma
Surgical
Compression at the neck of the fibula (habitual leg crossing, cast, brace)
Infection Leprosy
Inflammatory CIDP
Ischaemic - Vasculitis
Part of mononeuritis multiplex (Endo, AI, infection, infiltrative and cancer)
Ix = NCT and EMG
Mx
PT/OT 90 degrees splint at night
Sx for severed nerve or excision of ganglion
Sciatic nerve (L4 L5 S1 S2)
Weakness of the knee flexion also
Knee jerk is intact but ankle jerks affected and plantar response absent (for common peroneal nerve, all reflexes intact)
L5 nerve root
Weakness of hip abduction and internal rotation as well as loss of foot inversion (cf with common peroneal nerve)

136

59. Hemiparesis/Hemiplegia
Examination
This man has a left hemiparesis, please examine him
For this, examine the UL and LL
Locate
o Brainstem
Webers syndrome (III and contralateral hemiplegia)
Millard-Gubler (VI and VII and contralateral hemiplegia; usually a/w contralateral loss of proprioception
and light touch as the medial lemniscal damage)
o Subcortical lacunar; a/w UMN VII
Pure motor (50%)
Pure sensory (5%)
Mixed motor and sensory (35%)
Ataxic hemiparesis (10%)
Dysarthria clumsy hand syndrome (rare)
o Cortical signs
Do abbreviated version
Gaze preference, sensory and visual neglect, hemianopia and dysphasia if dominant lobe involved
Causes or risk factors
o Pulse, Carotid bruit, murmur
o Dyslipidaemia stigmata (xanthelasma, xanthomas, thickened TA)
o DM dermopathy
o Tar stains
o Bruising, telangiectasia
Function and complications
o Upper limb
o Gait
o Pressure sores, NG, urinary catheter
Request
o BP
o Urine dipstick
o Fundoscopy for papilledema (to rule out SOL which is a possible differential)
Presentation
Sir, this patient has got a left hemiparesis as evidenced by
State the UL and LL findings
State the level of the lesion and justify as above
Mentioned the causes as above
Mention the functional status and complications

137

Questions
What are your differential diagnoses?
Vascular
o Ischaemic (80%)
Intracranial thrombosis
Extracranial embolism heart, extracranial arteries, paradoxical
Lacunar strokes small vessel disease from DM or hypt as a result of lipohyalinosis
Dissection
o Haemorrhagic (Intracerebral, SDH, SAH)
Space occupying lesion
Infective abscess, meningoencephalitis
Seizures
Toxic-metabolic Hypoglycaemia, HypoNa
What are the 4 neuroanatomic stroke syndromes?
Anterior cerebral artery - affect frontal lobe function, producing altered mental status, impaired judgment, contralateral lower
extremity weakness and hypoesthesia, and gait apraxia.
Middle cerebral artery (MCA) - contralateral hemiparesis, contralateral hypoesthesia, ipsilateral hemianopsia (blindness in one
half of the visual field), and gaze preference toward the side of the lesion. Agnosia is common, and receptive or expressive
aphasia may result if the lesion occurs in the dominant hemisphere. Since the MCA supplies the upper extremity motor strip,
weakness of the arm and face is usually worse than that of the lower limb.
Posterior cerebral artery occlusions affect vision and thought, producing homonymous hemianopsia, cortical blindness, visual
agnosia, altered mental status, and impaired memory.
Vertebrobasilar artery occlusions are notoriously difficult to detect because they cause a wide variety of cranial nerve,
cerebellar, and brainstem deficits. These include vertigo, nystagmus, diplopia, visual field deficits, dysphagia, dysarthria, facial
hypoesthesia, syncope, and ataxia. Loss of pain and temperature sensation occurs on the ipsilateral face and contralateral
body. In contrast, anterior strokes produce findings on one side of the body only.

138

How would you investigate?


Confirm the diagnosis
o Imaging CT or MRI if posterior stroke (Diffusion-weighted imaging)
For diagnosis and type
For complications eg hydrocephalus
Blood Ix
o FBC polycythemia
o Coagulation profile
o Biochemical HypoNa
ECG AF, MI (60% a/w with AF or MI)
CXR Enlarged mediastinum suspicious of a dissection
2D echo (cardioembolic course)
Carotid ultrasound scan (significant stenosis - >70%) and transcranial doppler
Young patient young stroke work up (10)
o ANA, dsDNA, ESR
o Protein C, S
o Anti Thrombin III
o Factor V leiden or APC resistance
o Anticardiolopin IgM/IgG
o Homocystine
o VDRL
What are the limitations of CT brain?
Unable to visualise the posterior fossa structures such as the brainstem and the cerebellum
Maybe normal up to 6 hours of onset
o After 6 hours hypodense area
o Early signs on CT (5) loss of grey-white differentiation, insular ribbon sign, sulcal asymmetry, hyperdense
MCA sign and obscuration of the LN
MRI Diffusion weighted imaging which has a high sensitivity looked for hyperintense signal
How would you manage?
Multidisciplinary approach
Education and counselling
PT/OT and ST speech and swallowing, caregiver training, prevention of bed sores
Medications
o Antiplatelets (Aspirin, persantin, Clopidogrel, Ticlid)
o Anticoagulation
Correct risk factors
o Hypertension
o Hyperlipidaemia
o Diabetes mellitus
Surgical
o Intracranial bleeds
o Hydrocephalus
How would you manage the patient acutely?
Airway, breathing and circulation
Control BP if bleed otherwise allow high BP in ischaemic stroke up to 220/120
Treat fever
Control of blood sugar
Determine if bleed or ischemic
If ischaemic stroke, assess for possibility of reperfusion therapy ie National Institute of Health Stroke Scale (NIHSS) using
alteplase ie recombinant tissue plasminogen activator; within 3 hours and important to note inclusion and exclusion
criteria.
Treat complications
o Seizures
o Raised intracranial pressure (Hyperventilation, elevate the head, mannitol)

139

60. Gait Assessment


Examine this patients gait
Procedure
o Stand and Rhombergs
o Walk and turn and return
o Heel to toe
o Heel walk
o Tip toe
o Squat
Look for the obvious
o Ankylosing spondylitis
o Chorea
o Hemiplegic gait
o Antalgic gait
Small paces
o With stooped posture and paucity of arm swing
Parkinsonian
o With upright gait, normal armswing
March a petit pas
Diffuse cerebrovascular disease
Feet separation
o Broad

Cerebellar
Unilateral or bilateral
+ high stepping = Sensory ataxia
Peripheral neuropathy or dorsal column loss
Crossing over
Scissoring gait
Spastic cerebral palsy, MS, cord compression

High stepping with normal feet separation


o Unilateral or bilateral footdrop
Pelvis rotating
o Waddling gait
Proximal myopathy or congenital dislocation of the hips
Apraxic gait disjointed
o Frontal lobe CVA, SOL, hydrocephalus
Bizarre in consistent = functional gait, Huntingtons chorea

140

61. Parkinsonism
Examination
Introduce
Mask like facies, monotonous speech, dyskinesias
Upper limbs
Resting tremors which disappears with use
Bradykinesia (thumb to finger, rotate wrist and twinkle stars
Leadpipe rigidity and cogwheeling
Acute dystonia or alien limb syndrome
Pronator drift and cerebellar signs
Palmomental reflex, grasp reflex
Face
Eye movements, vertical Dolls if vertical gaze impaired
Close eyes for blepharospasm
Feel for seborrhea
Look for KF rings
Count 1-20
Unbutton shirt, write, cap a pen, comb his hair
Gait typical parkinsonian gait; also rule out gait apraxia
Request
Speech if not done
Swallowing
Handwriting
Postural BP
AMT
Presentation
Sir, this elderly gentleman has Parkinsonism with mask like, expressionless facies. He has asymmetrical resting tremor of the right
hand with characteristic pill rolling movements of the thumb that disappears with use of the hand. There is also presence of
bradykinesia with leadpipe rigidity at the elbows and cogwheeling at the wrist. Movement of the contralateral upper limb
accentuates these features.
There is presence of seborrhea and Myersons sign or the glabella tap sign is positive.
He has difficulty initiating his gait and has a stooped posture associated with shuffling gait with festination and lack of normal arm
swing. He also turns in numbers. His gait is not apraxic and he is not on any urinary catheter to suggest NPH.
Functionally he is able to walk unaided and can perform keyturning movements and unbutton his short unaided.
There is no evidence of dyskinesias which can result as a result of L-dopa therapy.
He dose not have features suggesting presence of Parkinson-plus syndrome. There is no evidence of Progressive Supranuclear
Palsy such as impairment of the vertical gaze, blepharospasm or frontal lobe signs such as palmomental reflex and the grasp
reflex. There are also no cerebellar signs to suggest multisystem atrophy. There is also no evidence of corticobasal ganglia
degeneration such as dystonic arm or alien limb syndrome.
In summary, this patient has Parkinsonism most likely due to Parkinsons disease and relative preservation of his function; there is
no evidence of dyskinesia currently to suggest side effects of L-dopa therapy.

141

Questions
What is Parkinsons disease?
It is a progressive neurodegenerative disorder associated with degeneration of the dopaminergic nigrostriatal neurons. Dx clinically
th
with 2 out of 3 signs comprising of resting tremors (3-5Hz), bradykinesia and rigidity. The 4 sign of postural instability occurs later
in the course of the disease.
What are the causes of Parkinsonism?
1. Parkinsons disease
2. Parkinson-plus syndromes
3. Drugs (Neuroleptics, antiemetics, MPTP- 1 methyl 4 phenyl 1,2,3,6 tetrahydropyridine)
4. Anoxic brain damage ( Post cardiac arrest, Manganese, CO)
5. Post encephalitis ( encephalitis lethargica or von Economos disease)
6. Tumor such as giant frontal meningioma
What are the pathologic findings in Parkinsons disease?
Loss of pigmented dopaminergic neurons in the substantia nigra
Presence of Lewy Bodies (eosinophilic cytoplasmic inclusions)
What are the Parkinson-plus syndromes?
Progressive supranuclear palsy (most common) (frontal lobe) (3)
Vertical gaze palsy
Downgaze affected first, then upgaze, then horizontal
Can be overcome by vertical Dolls
Other features such as blepharospam and slow pursuit or saccadic eye movements
Postural instability and axial rigidity with falls early in the course of the disease
Frontal lobe signs
Multiple sytem atrophy (Cerebellar)
MSA-P = Parkinsonism features
MSA-C = Cerebellar features
Features (3)
Cerebellar signs
Autonomic features orthostatic hypotension, urinary dysfn and erectile dysfn
Corticospinal signs hyperreflexia and extensor plantar response
Corticobasalganglionic degeneration (frontoparietal lobe)
2 features
Limb apraxia or alien limb syndrome
Dystonia
Parkinsonism-dementia-ALS complex
Diffuse Lewy Body disease (Parkinsonism, dementia and neuropsychiatry)
What is the significance of diagnosing Parkinson Plus syndrome?
Poorer prognosis
Poor response to L-dopa therapy
What are the features that suggest that patient may have Parkinson plus syndromes?
Early onset of dementia
Presence of hallucination or psychosis
Early onset of postural instability
Truncal symptoms more prominent than appendicular symptoms
Marked symmetry of signs early in the stage of the disease
Lack of response to levo-dopa therapy in the early stage of the disease
Presence of symptoms and signs suggestive of Parkinson-plus syndromes.
What are the stages of Parkinsons disease?
Staged via the Hohen and Yahr staging system comprising of 5 stages:
Stage 1 symptoms and signs unilateral and mild
Stage 2 Bilateral and minimal disability
Stage 3 Generalised dysfunction with sig bradykinesia and gait impairment
Stage 4 Rigid and bradykinesia, severe symptoms with limited walking
Stage 5 Completely invalid and requires nursing care

142

How would you investigate?


Brain scan to rule out
NPH
Multi-infarct syndromes
Frontal meningiomas
Parkinson-plus syndrome
MSA atrophy of the cerebellum, brainstem
PSP Atrophy and hyperintensity of the midbrain and red nucleus
CBGD Frontoparietal cortical atrophy
If patient is young, ie<50, rule Wilsons disease
Slit-lamp examination
Serum ceruloplasmin and 24Hr Urinary Copper
How would you manage?
Multidisciplinary approach
Education and counselling, PT/OT/ST
Medications
Dopamine agonist, eg pramipexole or ropinirole
Early in the course of the disease or younger patients
Delay onset of motor fluctuations and dyskinesias
Nausea, orthostatic hypotension, hallucinations or somnolence, edema
L-dopa therapy (usually combined with a peripheral decarboxylase inhibitor)
Should be used if there is disability
Never used in patients with melanoma
Peak dose dyskinesia and end of dose rigidity
Nausea, orthostatic hypotension, hallucinations
Tremors anticholinergic (dry eyes and mouth, urinary retention, arrhythmia), e.g. arcane or benztropine.
Rigidity beside Dopa and D agonist, can use MAO-B inhibitors or amantadine (cognitive side effects)
COMT inhibitors (diarrhea and hepatotoxicity)
Surgical
Lesion surgery thalomotomy (tremors) and pallidotomy (for all features)
Deep brain stimulation (for all features especially tremors)
How would you manage dyskinesia?
Peak dose dyskinesia
Reduce the dose and increased frequency
Add on COMT inhibitors i.e. entacapone which increases half life of L-dopa therapy
Initiating with dopamine agonist and adding on L-dopa therapy resulted in less motor fluctuations
End of dose
Increase dose, frequency
Switch to CR formulations
Add DA, MAO-B inhibitors, COMT inhibitors
What is the prognosis?
Chronic
Progressive
Some will have mild while other will have severe symptoms
Some will be troubled mainly with tremors, other by rigidity and bradykinesia
What is the abbreviated mental test?
Use as a screening for possible dementia in the elderly
A score of less than 6 warrants further assessment
Includes
Age
DOB
Remember this address 42 West Street
Time (nearest hour)
Year
Recognition of 2 persons
Place
Prime Minister of UK
First Year of WW1 (1914-1918)
Count backwards 20 to 1

143

RHEUMATO!
62. Rheumatoid Arthritis
Presentation
Sir, this patient has Rheumatoid arthritis affecting the hands as evidenced by
Presence of symmetrical deforming polyarthropathy
PIPJ/MCPJ
Swan neck, Boutonnieres, Z-thumb, ulna deviation
Subluxation (MCPJ, dorsal subluxation of the ulna at the carpal joint)
Active arthritis/quiescent
Intrinsic muscle wasting
CTS
Dropped fingers from tendon rupture
Synovial thickening
Vasculitic lesions, nail-fold infarcts
Palmar erythema
No nail changes and skin lesions of Psoriasis
SLE skin changes
Elbows for Rh nodules
Function
Preserved vs impaired
Coarse and fine functions
Treatment
Steroid atrophied skin, bruisability
Surgical intervention CTS decompression, tendon release
Requests
Other joint involvement (MTPJ, knees)
Extra articular features of RA
Questions
What are the extra-articular features of RA?
Eye
o Conjunctiva Keratoconjunctivitis sicca, pallor
o Sclera episcleritis, scleritis, scleromalacia perforans
o Lens Cataracts from chronic steroid usage
o Retina vasculitis, drug induced (Gold, Hydroxychloroquine)
o Extra-ocular muscles mononeuritis multiplex, myasthenia sec to penicillamine
Respiratory
o Upper airways Cricoarytenoid
o Pleura pleurisy, effusions
o Airway BOOP
o Parenchyma Pulmonary fibrosis, pneumonitis, PHT ( RA or MTX)
o Caplans, Nodules
Neurological
o Peripheral neuropathy
o Mononeuritis multiplex
o Nerve entrapment
o Cx atlanto-axial subluxation +/- Cx myelopathy
o Muscle atrophy, proximal myopathy sec to steroids, penicillamine induced myasthenia
Abdomen
o Splenomegaly in Feltys syndrome
What are the causes of anaemia in RA?
Fe deficiency GI bleed from NSAIDS
Megaloblastic anaemia Pernicious anaemia
Anaemia of Chronic disease
Hypersplenism from Feltys Syndrome
Aplasia Gold, Penicillamine

144

What are the poor prognostic indicators?


Insidious onset and high activity at onset
Rh nodules or early erosions within 1 year
Extra-articular features
Persistent activity after 1 year Active arthritis, ESR
High levels of Rh factor and Anti CCP Ab (anti cyclic citrullinated peptide Ab)
What are the criteria for Dx RA (American college of Rheumatology)?
Any 4 of the following:
Morning Stiffness for >1 hr duration for > 6 weeks
Arthritis of 3 or more joints for > 6 weeks
Arthritis of wrists, MTCP, PIPJ
Symmetric
Rh nodules
Rh factor
Radiographic changes typical changes including erosions or unequivocal decalcification
How would you investigate this patient?
Blood Ix Rh factor, anti-CCP, ESR, CRP
X-rays of the joints erosions and periarticular osteopenia
How would you manage this patient?
Education and counselling
Non-pharmacological
o OT, PT
Pharmacological which will depend on the severity
o Analgesia NASIDS
o DMARDS
Methotrexate (Check FBC and LFT)
Sulphasalazine
Hydroxychloroquine
Low dose prednisolone
Newer agents
Leflunomide
Tacrolimus
Cyclosporine A
Rapamune (sirolimus)
o Immunomodulators (biologics)
Anti TNF Etarnacept (FDA approved), infliximab, humira
Beware of TB and atypical pneumonia resulting from their use
Anti CD20 Rituximab
Surgical
What is Z thumb deformity?
Deformity that occurs in RA hands
With hyperextension of the first IPJ and fixed flexion and subluxation of the first MCPJ
Resulting squaring appearance of the hands
What is Boutonnires deformity?
Hyperflexion of the PIPJ and hyperextension of the DIPJ
Due to rupture of the central slip of the extensor tendon over the PIPJ with imbalance of the flexion and extension forces
of the finger
What is swan neck deformity of the fingers?
Hyperextension of the PIPJ and hyperflexion of the DIPJ
Due to synovitis of the flexor tendons leading to flexion at the MCPJ with constant effort to extend the finger; leading to
stretching of the collateral ligaments and the volar plate of the PIPJ; intrinsic muscle balance leads to swan neck
deformity
What are the differential diagnoses for deforming polyarthropathy of the hands?
Rheumatoid arthritis
Psoriatic arthritis of the RA type
nd
th
Jaccouds arthropathy which is ulna deviation with subluxation of the 2 to 5 fingers at the MCPJ which is voluntarily
correctable; initially described in patients with Rh fever but now used synonymously with SLE deforming arthropathy

145

63. Gouty Hands


Examination of the Hands Sequence
Tophi, joint deformity
Feel joints for active arthritis
Palmar erythema, dupytrens contracture (alcohol), finger pulps for tophi
Test function pincer and grip, coarse and fine
Look at the extensor surface and elbows (olecranon bursae)
Sallow appearance, dialysis (Renal failure)
Pinna or helix of the ear
Pleithoric, parotidmegaly, bleeding and hypertrophic gums
Look at the feet for joints, deformity, active arthritis, diabetic dermopathy
Feel the achilles tendon and infrapetallar region
Request
o Walk patient if feet are involved
o BP
o Urine dipstick for glycosuria, hematuria (stones)
Presentation
Sir, this patient has chronic tophaceous gout affecting his hands and his feet. On examination of the hands, there is asymmetrical
swelling affecting the small joints of the hands with tophi formation which has resulted in severe deformity of the hands and feet. I
also noticed that these tophi are exuding chalky material. On palpation, there is no tenderness and joints are not warm to suggest
active arthritis. There is wasting of the intrinsic muscles of the hands. There is also presence of tophi on the extensor aspects of the
forearms, the left olecranon bursae, the right helix/pinna of the ear as well as the small joints of the feet. I looked for but did not
detect any tophi on the achilles tendon or the infrapetallar region.
In terms of function, he is able to perform pincer and handgrip movement and his hand function is relatively preserved; able to
perform door knob turning and cap a pen, as well transfer coins and unbutton his shirt.
I noticed that the patient is not obese looking, no DM dermopathy or xanthelasma as these are a/w gout. There is also no evidence
of chronic ethanol ingestion such as palmar erythema, dupytrens contracture and parotidomegaly. There is no sallow appearance
to suggest chronic renal failure. I also did not detect any conjunctival pallor or suffusion, hypertrophic or bleeding gums and patient
is not pleithoric which may suggest presence of lymphoproliferative disease or polycythaemia. There are no psoriatic skin lesions
I would like to complete the examination by walking to patient to assess function as I noticed that his feet is affected by gouty
arthritis, take his blood pressure as well as a urinalysis to look for glycosuria as well as hematuria for UA stones and proteinuria for
UA nephropathy. A detailed drug history, dietary history and alcohol consumption.

146

Questions
What is gout?
Gout is a disorder of purine metabolism, resulting in hyperuricaemia either from overproduction(75%) or undersecretion of uric acid,
resulting in deposition of urate crystals in the joints or bursae.
Patients typically present with acute monoarthritis of the first MTPJ, with pain swelling and exquisite tenderness which peaks within
hours and lasts for days. It affects the joints of the lower limbs initially in the majority of patients which includes the MTPJ, ankles
and knees. It can also subsequently affects the joints of the upper limb.
What are the stages of gout?
Acute gouty arthritis
Intercritical period
Chronic tophaceous gout
What does tophi indicate?
Severe, recurrent and chronic gout.
Where are the commonly areas to look for gouty tophi?
Hands, extensor aspect of the forearms, olecranon bursae
Helix if the ears
Toes, Achilles tendons, infrapetaller regions
What are the clinical manifestations of gout?
Asymptomatic hyperuricaemia
Acute arthritis
Chronic, recurrent arthritis
Tophaceous gout
Uric acid nephrolithiasis
Uric acid nephropathy
What are the triggering factors of gout?
Alcohol ingestion
Foods sweetbreads, liver, kidneys and sardines
Drugs Thiazide diuretics, aspirin, cyclosporine, pyrazinamide and ethambutol
Dehydration and fasting
Surgery, Trauma
What are the causes of gout?
Primary associated with obesity, diabetes mellitus, hypertension and high TGs
Secondary
o Drugs
o Chronic ethanol ingestion
o Chronic renal failure
o Polycythaemia, lymphoproliferative, myeloproliferative
o Psoriasis
How would you investigate?
Definitive investigation would be aspiration of the involved joint, looking for
intracellular deposition of needle-shaped crystals that is negatively birefringent under polarised light, within leukocytes.
They react with nitric acid and NH4OH to give a purple color (Murexide test)
Blood Ix Uric acid levels which may be normal during an acute attack
X-ray of the joints may show erosive arthropathy from tophi with overhanging
edges associated with punctuate to diffuse calcification.
How would you manage?
Education and counselling, including dietary advice and avoidance of alcohol
PT/OT if tophaceous gout for preservation of function
Manage associated hypertension and diabetes mellitus
Medications acute attack and prophylaxis
Surgery rarely for cosmetic reasons, arthroplasty

147

How would you treat an acute attack?


NSAIDS Indomethacin (50mg tds)
Colchicine 2 ways:
o 0.5mg hrly till GI side effects or max of 5 mg, or
o 0.5mg tds
Intrarticular steroids (triamcinolone 20mg)
Systemic steroids (Prednisolone 30mg OM and tails over 7-10 days)
How would you prophylax against gouty attacks?
Prophylactic agents used are iniated under colchicine cover which includes:
Xanthine oxidase inhibitor
o Allopurinol
o New agents Uricase, febuxostat
Uricosuric acid agents
o probenecid or sulfinpyrazone
o losartan
o fenofibrate
What are the indications for allopurinol?
Recurrent gouty attacks > 3 times a year
Chronic tophaceous gout
Uric acid nephropathy
Persistently high uric acid level
Conditions that may predispose an individual to gouty attacks, prior to chemotherapy or radiotherapy which may induce tumor
lysis
What are the side effects of allopurinol?
Side effects occur in 3-5%
Rash, diarrhea, drug fever
Leucopenia, thrombocytopenia
Allopurinol hypersensitivity syndrome
o Erythematous rash, fever, hepatitis, hypereosinophilia and renal
o failure
What are the other crystal arthropathy that you know about?
Pseudogout Acute arthritis resulting from deposition of calcium pyrophosphate dihydrate crystals in the joints which are
rhomboid shaped positively birefringent crystals under polarised light.
Calcium hydroxyapatite crystals deposition in the large joints such as knees and shoulders, affecting the elderly.
What are the differential diagnoses?
Septic arthritis
Overlying cellulitis
Trauma
What is your differential diagnosis for chronic tophaceous gout?
Florid tendon xanthomata
Yellow and not chalky
Adherent to tendon and not joint
Does not involve the bursae, ie no olecranon or pinna lesions
No active arthritis

148

64. Psoriasis Locomotor (10% of Psoriasis with Joint involvement)


Presentation
Sir, this patient has psoriatic arthropathy affecting the hands of the
1. Arthritis mutilans type (bilateral deforming arthropathy, telescoping of the digits)
2. RA type (symmetrical joint involvement)
3. OA type (asymmetrical terminal joint involvement)
4. mono/oligoarticular type
5. AS type (Sacroilitis, but the syndesmophytes arise from the lateral and anterior surface and not at the margins unlike AS)
With
1.
2.
3.
4.
5.
6.

Bilateral deforming polyarthropathy, and joint deformities, tender (activity)


sausage shaped fingers, tenosynovitis
wasting dorsal guttering and wasting of the thenar and hypothenar eminence
nails pitting, onycholysis, subungal hyperkeratosis, discoloration of the nails (80% involvement with arthropathy)
Skin patches well circumscribed plagues on the extensor surfaces of the elbows and scalp, with salmon pink hue and
silvery scales
surgical scars

Joint function
1. Impaired or preserved
2. able to grip and do pincer movement
3. coarse function turn a doorknob
4. fine function cap a pen, transfer coins, unbutton clothes
5. able to abduct and internally rotate her shoulder joints which are important for her ADLS
Treatment complications Steroids for arthritis
Mention no evidence of Gout (as this is associated with Psoriasis)
Complete my examination by
examining for other joint involvement
Skin especially scalp, knees, natal cleft, intragluteal folds, submammary folds, Koebners phenomenon
Enquire on aggravating factors
Questions
What are the types of skin lesions?
Plague
Guttate (numerous small papular, hx of streptococcal infection
Pustular (localized or generalized, superficial pustules may stud the plagues)
Erythrodermic (generalized erythema and scaling which may be life threatening)
Inverse psoriasis (plagues evolving in the intertriginous area without typical silvery scales due to moisture and maceration)
Where are the typical sites of distribution?
Extensor surfaces of knees, elbows, scalp, navel, natal cleft, submammary and intragluteal folds
How do you assess severity?
Psoriasis Area and Severity Index area, thickness, redness and scaling
Total score 72 - <10, 10-50, >50 for mild, moderate and severe respectively
What are the types of joint involvement in psoriasis?
OA
RA
AS
Oligo/mono
Arthritis mutilans
Radiological features of psoriatic arthritis?
Periostitis fluffy
Destruction of small joints
Pencil in cup appearance
Non marginal syndesmophytes in AS type
What are the unique characteristics of psoriatic lesions?
Salmon pink hue with silvery scales
Koebners - New psoriatic skin lesions at site of cutaneous trauma
Moist red surface on removing of scales (Bulkeleys membrane)
Auspitzs sign capillary bleeding when silver scales are picked from the plague

149

What is Koebners phenomenon and which other conditions is it seen?


New skin lesions at the site of cutaneous trauma
Occurs in 30% of patients with psoriasis, usually occurring 10-20 days postTrauma, ranges from 3 to 2 years
Also occurs in eczema, lichen planus, vitiligo and lichen sclerosus et atrophicus
What is the pathology?
Hyperproliferation of the epidermis with inflammation of the dermis and epidermis.
What are the differential diagnoses for onycholysis?
Psoriasis
Fungal infection
Thyrotoxicosis (Plummers nails)
Lichen Planus
What are the aggravating factors?
Emotional stress
Alcohol
Drugs beta blockers, ACE inhibitors, Indocid, Lithium & antimalarials
Streptococcal infection (classically associated with guttate psoriasis)
Injury to the skin mechanical injury and sunburn
What are the principles of management?
Education
Avoidance of aggravating factors
Topical WSP, salicyclic, aqueous cream
Topical Topical steroids, coal tar, Dithranol, Calcipotriol (Vit D3 which acts to increase keratinocytes differentiation as a
result of increased extracellular calcium therefore decreased cellular proliferation and scaling), topical retinoids
Systemic UVB, MTX, Retinoids, systemic steroids, cyclo, tacrolimus and MMF
Novel immunodulators (infliximab, etarnacept)
What is the prognosis?
Deforming and erosive in 40%
10% are disabled by the arthropathy
What other joint pathology can patients have especially if disease is active?
Gout because of hyperproliferation
Others
30% have family history
Psoriasiform lesions on the fingers, toes, nose and ears exclude SCC of the Oropharynx, tracheobronchial tree and
esophagus Bazex syndrome.

150

65. Osteoarthritis of the Hands


Examination
Herbedens nodes and Bouchards nodes, squaring of the thumb
Presence of active arthritis
No muscle wasting
Tinels sign
Function
o ROM
o Coarse fn
o Fine fn
Establish cause
o Primary
o Secondary to Acromegaly, Hemochromatosis
Request
o Examine other joints
Knee varus/valgus deformity, crepitus, wasting of the quadriceps
Hips
Cervical spondylosis, Lumbar spondylosis
Gait (Trendenlenbergs sign downward tilting of the pelvis on the affected side)
Presentation
Sir, this middle-age lady has OA of the hands as evidenced by presence of Herbedens nodes which are bony swelling affecting the
DIPJ. I did not detect any Bouchards nodes but there is presence of squaring of both hands as a result of subluxation of the first
MC. There is no significant muscle wasting with preservation of function. ROM was good and patient is able to perform coarse fn
such as turning a door knob and fine motor fn such as transferring coins. Tinels sign is negative.
I would like to complete the examination by examining other joints for OA in particular
Knees
Hips
Gait for Trendelenbergs sign
Cx and Lx spondylosis
There are no features of acromegaly or hemochromatosis.
I would like to offer the dx of Nodal OA or primary generalized OA with OA of the hands occurring in this middle aged lady.

151

Questions
What are Herbedens nodes?
Bony swellings at the DIPJ in OA
What are Bouchards nodes?
Bony swelling at the PIPJ in OA
Why is there squaring of the hand?
This is due to subluxation of the first MC
What are the types of OA?
Primary generalized OA aka nodal OA
Middle-aged women, Autosomal dominant
OA of the DIPJ with Herbedens with marked deformity and preservation of fn
Also affects the carpometacarpal joint of the thumb, knees and hips
Secondary
Trauma
Inflammatory arthropathies RA, Septic arthritis, gout
Endocrine Acromegaly, hyperparathyroidism
Metabolic chondrocalcinosis, hemochromatosis
Neuropathic joints DM, Tabes, syringomyelia
How would you Ix?
Radiographical
Subchondral bone cysts and sclerosis
osteophytes
narrowed joint space
Varus/varus deformity
If a synovial aspirate is done to r/o other causes
<100wbc/ml
How would you manage?
Education and counseling
o Appropriate footware
o Weight management
PT/OT
Pharmocotherapy
o Glucosamine, chondroitin sulphate, glycosaminoglycans
o Drugs
Analgesia paracetamol, NSAIDs, Opiods
Tetracycline (inhibit enzyme that breaks down cartilage)
Diacerin (Anti IL-1)
Hydroxychloroquine
o Intra-articular steroids
o Visco-supplementation (hyaluronic intra-articular injection)
Surgery - Knee and hip replacement

152

66. Scleroderma
Seated (hands on pillow if available)
This is a middle-aged lady has got scleroderma as evidenced by
Hands evidences of sclerodactyly and smooth/shiny/tight/taut shiny skin of the hands which extends proximal to the MCPJ
(double pinch test). There is also digital tip pitting, finger pulp atrophy and acroosteolysis. There is also presence of Raynauds
th
phenomenon, beaking of nails (pseudoclubbing), atrophic nails, nail fold telangiectasia (especially 4 digit via magnifying glass)
and vasculitic rashes at the finger tips. There is also calcinosis and subcutaneous calcification located fingers, elbows and extensor
aspect of the forearms. This is associated with wasting of the intrinsic muscles of the hands and vitiligo/hyperpigmentation (salt and
pepper appearance).
In terms of function, she is able to perform pincer movements and hand grip is good with a power of 5; however there is limitation of
finger extension with flexion contractures and she finds it difficult to unbutton/button of clothes as well as to perform turning door
knob manoeuvres.
There is no proximal myopathy (myositis)
There is also involvement of her face as evidenced by:
Face Bird like facies, smooth/shiny/tight/taut skin of her face with difficulty closing her eyes, blotchy telangiectasia, pinched
nose, microstomia, perioral tethering with pseudorhagades. I also noticed that the patient is cachexic looking; note pallor
Legs On examination of her legs, I also noticed presence of scleroderma as well as vasculitis, telangiectasia and ulcerations,
vitiligo.
Treatment complications - Steroids
I would like to complete the examination by taking the patients blood pressure, urine dipstick, cardiovascular examination,
respiratory examination and abdominal examination and ask her about dysphagia (examine stools for steatorrhea), Raynauds
phenomenon as well as dry eyes and mouth.

153

Questions
What are the criteria for diagnosis of scleroderma?
Major: Proximal scleroderma (affecting MCPJ/MTPJ)
Minor: Sclerodactyly
Digital tip pitting/pulp atrophy
Bibasal pulmonary fibrosis
Require 1 major or 2 minor
What are the types of Scleroderma?
CREST (a/w anticentromere antibodies)
Limited cutaneous (extremities)
Diffuse cutaneous (involving of skin of trunk)
Scleroderma sine scleroderma (systemic complications without skin)
What are the phases of skin changes? Edematous phase... Dermal phase (induration)... Atrophic phase with contractures
What are the complications?
Lung
Pulmonary fibrosis
Reflux pneumonitis
Pleural effusion
Alveolar cell carcinoma
Cardiovascular
Primary Pulmonary Hypertension
Cor Pulmonale from Pulm fibrosis
Pericarditis, Pericardial effusion
Myocardial fibrosis
Abdomen
Esophageal dysmotility
Malabsorption with steatorrhea from dilated second part of
the duodenum resulting in bacterial overgrowth
Kidneys Renal failure (Malignant hypertensionresponsive to ACE -)
Primary Biliary Cirrhosis (Woman rare)
What are the possible etiologies for anaemia in a patient with scleroderma?
Anaemia of chronic disease
Fe deficiency anaemia from esophagitis
B12 and folate deficient anaemia from malabsorption
Microangiopathic hemolytic anaemia (MAHA)
Aplasia from medications such as MTX
What is Thibierge-Weissenbach syndrome?
Acrosclerosis associated with deposition of calcium in the subcut tissue
What is mixed connective disease?
2 or more - SLE, polymyositis, dermatomyositis, SSc
Antiribonuclear protein antibody (Speckled pattern)
Why must you avoid high dose corticosteroids? This may precipitate renal crisis.
How would you investigate?
ANA Speckled pattern
Anti-topoisomerase DNA 1 (Scl-70) for systemic sclerosis
Anti-centromere antibody (CREST)
Outline your management.
Education and support
PT/OT for hand function
Treat symptoms and complications
Raynauds avoid cold, calcium channel blockers, prostacyclin
Esophagitis PPIs
ACE inhibitors for hypertension
Bosentan (endothelial receptor antagonist for pulmonary hypertension)
Treatment of disease
Immunosuppressants Steroids, MTX, Aza, cyclophosphamide
Antifibrotic penicillamine, interferon
What is the Px?
Male (poor)
Renal involvement (poor)
70% for skin involvement only 10 yr survival
20% 10 yr survival if kidneys and lung involved

154

67. Ankylosing Spondylitis


Approach
Stem Statement
1. Look at this patient (no apparent endocrine, rheumatology or neurological abnormalities)
2. Examine patients gait
3. Low back pain, chest, back, neck, look at the ceiling
Proceed
1. Introduce (thank you for letting me exam you, my name is how do I address you)
2. Ask if there is pain
3. Stand the patient, walk him, turn around and return to original position
4. Touch toes with his fingers
5. Look left and look right
6. Touch your chest with your chin
Presentation
Sir, this patient has Ankylosing Spondylitis as evidenced by a stooped, question-mark posture with loss of lumbar lordosis and a
fixed kyphosis with extension of the cervical spine in an attempt to maintain a horizontal visual gaze. There is also a protuberant
abdomen. Spinal movements are restricted as evidenced by the finger-to-toe test, with limited flexion and lateral movements of the
cervical spine.
I would like to complete the examination by doing the heels, hips and occiput test and measure the occiput-to-wall distance, as
well as a modified schoebers test and chest expansion (5cm).
I would like to look for
shoulder and knee joint involvement
extra-articular involvement and complications
Differential diagnoses
o Skin to rule out Psoriasis which is a possible differential diagnosis
o Abdominal examination to look for signs of inflammatory bowel disease
o Reiters syndrome
Questions
1. Tell me about AS.
a. Seronegative spondyloarthropathies, chronic inflammatory arthritis affecting the SI joints with fusion of the spinal vertebrae,
associated with HLA b27
b. 3rd to 4th decades, males 3X
c. Symptoms: back pain worse in the morning and with rest and improves with activity
d. Signs : limited lateral flexion of the lumbar spine is the first sign of spinal involvement followed by loss of lumbar lordosis
e. Investigation and management
2.

What are the associated conditions?


a.
b.
c.
d.
e.
f.
g.

Anterior uveitis, iritis


Atlanto-axial subluxation or dislocation
Apical fibrosis
Aortic regurgitation
AV nodal block, arrythmias
Amyloidosis
Achilles tendonitis, plantar fasciitis

3.

What is the heels-hips-occiput test?


a. Ask the patient to place his heels, hips and occiput against a wall all at once
b. Inability of the occiput to touch the wall
c. Can measure the wall-occiput distance

4.

What is Schoebers test?


Draw a line joining the dimple of Venus 5cm mark below 10cm mark above
Forward flexion <5 cm implies limited spinal mobility

5.

Why is there a protuberant abdomen?


a. This occurs as a result of restricted chest expansion from a fixed spine
b. Hence resulting in a predominantly diaphragmatic breathing
c. With resultant protuberant abdomen

6.

What are the other types of conditions that can present with sacroilitis?
a. Psoriasis
b. Reiters (reactive arthritis)
i. Can be urogenital (Chlamydia) or gastrointestinal (Shigella, campylobacter, salmonella)
ii. Triad of urethritis, arthritis and conjunctivitis
iii. Cs have circinate balanitis (small shallow painless sores) and keratoderma blenorrhagica (small hard papules on
palms and soles)
c. Enteropathic arthritis (these are the seronegative spondyloarthropahy which are associated with HLA B27)

155

7.

How do you diagnose AS?


a. Rome or New York Criteria
b. Based on:
i. Radiological features of sacroilitis
ii. Symptoms of back pain (lumbar spine or dorsolumbar junction)
iii. Physical signs of limited spinal mobility in all 3 planes and chest expansion <2.5cm

8.

How would you investigate?


a. Imaging AP views and the SI joints and AP/lat of spinal vertebrae
i. Early erosions and sclerosis of the SI joints
ii. Later Syndesmophytes in the margins of lumbosacral vertebrae
iii. Advanced bamboo spine
b. Blood test (not so important)
i. Elevated ESR
ii. HLA B 27 (95% AS is positive but small percentage of B27 positive develop AS)

9.

How would you manage?


a. Education and counseling
i. Chronic disease
ii. Genetic counseling (HLA positive, siblings has 30%)
1. 50% chance of transferring genes to children
2. 1/3 of children who has HLA B27 will have AS
3. Overall risk is 1/6
b. Non-pharmological
i. Lifelong regular exercises
ii. Involving the PT and OT
c. Pharmological
i. NSAIDs
ii. MTX, sulphasalazine
iii. Anti TNF and anti CD-20
d. Surgical therapy

What are the indications for starting immunomodulators?


TNF blocking agents are recommended for the treatment of active AS after having failed treatment for the patients predominant
clinical manifestation

156

68. Marfans Syndrome


Examination (seated)
Overall
Tall, disproportionately long limbs compared to trunk
Upper Limbs
Arachnodactyly, thumb sign, wrist sign(overlap > 1cm)
collapsing pulse
reduced extension of elbows
Face
Dolichocephalic(long-headed)
Blue sclera, iridodonesis, myopia, ectopia lentis (superolateral)
High arched palate
Meishers elastoma
No thyroidectomy scar
Chest
Pectus excavatum or carinatum
Thoractomy scar (Hx of repair of aortic aneurysm)
No gynaecomastia
LL
Arachnodactyly
Stand up
Kyphoscoliosis
Abdomen: inguinal or femoral herniae, hernia scars, striae atrophicae
Genu recurvatum
Pes planus
Request
Cardiovascular examination: MVP, AR
Respiratory: scar suggestive of chest tube for pneumothorax, pleurodesis
Lower limbs for weakness and numbness (complications of dural ectasia)
Measure his arm span to height ratio (>1)
Measure pubis-sole to pubis-vertex ratio (>1)
Presentation
Sir, this patient has Marfans syndrome as evidenced by tall stature with disproportinately long limbs (also known as
dolichostenomilia). He has got arachnodactyly with hyperextensible joints with positive thumb sign (Steinberg), wrist sign (Walker),
hyperextension of the elbows and genu recurvatum and pes planus.
There is presence of dolicocephaly, with iridodenesis, blue sclera and is myopic. He has a high arched palate. I did not detect any
Meishers elastoma (small papules of the skin of the neck).
There is also kyphoscoliosis with pectus excavatum. Of note there are chest wall scars suggestive of previous chest tube
insertions.
There is no obvious inguinal or femoral hernia, scars or striae atrophicae.
There is no collapsing pulse.
I did not detect any evidence of malar rash or calve swelling suggestive of a DVT which are features of homocystinuria. There is
also no neck scars, mucosal neuromas or hyperpigmentation to suggest MEN type 2B as these patients have a marfanoid habitus.
There is also no gynaecomastia or eunuchoid habitus to suggest Klinfelters syndrome (say this if patient is a man).
I would like to complete my examination by measuring his arm span to height ratio as well as his sole-pubis to pubis-vertex ratio; in
addition I would like to perform a cardiovascular examination to look for MVP, AR; a respiratory examination for plurodesis, as well
as lower limb examination for weakness or numbness secondary to dural ectasia.

157

Questions
What are the differential diagnoses for a patient who has a tall stature?
Marfans syndrome
Homocystinuria
Malar flush, mental retardation, inferomedial ectopia lentis
Hx of epilepsy, IHD(CABG scar), DVT, osteoporosis
Presence of homocystine in the urine via cyanide-nitroprusside test
Autosomal recessive inborn error of metabolism of amino acid with deficiency of cystathionine beta synthetase
MEN type 2b
Hyperpigmentation, mucosal neuromas(lips, tongue, palate, conjunctiva and cornea), proximal myopathy
MEN 1: Pituitary, parathyroid, pancreatic (PPP)
MEN 2a: Parathyroid, adrenals(phaechromocytoma), thyroid (MTC) (PAT)
MEN 2b: PAT and hyperpigmentation, mucosal neuromas, marfanoid
Klinefelters syndrome
Male patient, eunuchoid habitus (arm span> height, sole-pubis>pubis vertex, femenine fat distribution
Gynaecomastia, lack of beard and axillary hair, voice is not masculine, pea-sized testes (normal >3.5cm), varicose veins
Mentally subnormally, infertile
Rule out hypo-osmia for Kallmans syndrome (idiopathic hypogonadotrpic hypogonadism with hypo-osmia, cleft palate/lip,
congenital deafness or blindness which can be treated with gonadotropins and GnRH for fertility)
Raised FSH and estradiol with low testosterone and chromosomal analysis 47XXY(buccal smear for karyotyping)
Infertile as majority are 47XXY (80%) and others can be due to more than 2 X or > 1Y or mosaicism (can be fertile)
Most common cause of male hypogonadism, 1:500
Increased risk of DM, Br cancer and SLE
Increases with increasing maternal or paternal age
What is Marfans syndrome?
It is an inherited autosomal dominant connective tissue disorder
Affecting the skeletal system, cardiovascular system with ocular abnormalities
1 in 15 000
Male=Female
What is the mode of transmission of Marfans syndrome?
Autosomal dominant
Chromosome 15q21
Defects in fibrillin gene
How is Marfans syndrome diagnosed?
Based on the Ghent criteria which takes into account
o Family history
o Molecular studies
o 6 organ systems
Skeletal
Skin
Eye
CVS
Pulmonary
Dura (dura ectasia)
What are the ocular features of Marfans syndrome?
Small spherical lens
Cataracts
Lens subluxation
Glaucoma
Hypoplasia of dilator pupillae, therfore difficulty with pupillary dilatation
Flat cornea
Myopia
Retinal detachment
Increased axial length of the globe
How would you investigate?
Molecular studies
Annual echocardiography
Monitor aortic diameter (normal <40mm, composite graft required if >50mm)
MV function
Ophthalmic examination
How would you manage?
Education and psychological counselling
Annual cardio review
Beta-blockade (retards rate of aortic root dilatation)
Aortic root graft >50mm
IE prophylaxis
Eye review
158

What is the prognosis?


Death due to cardiovascular complications
Aortic dissection
CCF secondary to AR
Life span is about mid forties
What are the complications of pregnancy in Marfans syndrome?
Early premature abortion
Death from aortic dissection (safe if aortic root<40mm)
How would you counsel patient?
Affected individuals can transmit the condition to 50% of their offspring.
The recurrence risk is 50% if 1 parent is affected. The recurrence risk is small if neither parent is affected.
During counseling, the variability of the disease should be emphasized, as an affected child may be more or less affected than
the parent.
How do you assess hypermobility?
Beightons 9 point scale
passive dorsiflexion of the little finger beyond 90
passive apposition of the thumb to the flexor aspects of the forearm
hyperextension of the elbow beyond 10
hyperextension of the knee beyond 10
forward flexion of the trunk, with the knees straight, so the palms of the hands rested easily on the floor
If 4 or more => Joint laxity
What are the causes for hypermobile joints?
Benign Joint Hypermobility Syndrome (Majority)
Ehlers Danlos Syndrome
Marfans syndrome
Osteogenesis Imperfecta
What are the causes of blue sclera?
Marfans syndrome
Ehlers Danlos Syndrome
Osteogenesis Imperfecta
Pseudoxanthoma elasticum
Chronic steroid intake

159

69. Dupytrens Contractures


Typically
o elderly males
o pitting and thickening of the palmar skin progressing to a firm, painless nodule fixed to the skin and fascia, with
subsequent cord involvement
th
th
o involving bilateral 4 and 5 fingers resulting in fixed flexion of the MCPJ and PIPJ
o Garrods knuckle pads
Check function
Feet (plantar fibromatosis), Peyronies disease (penile shaft) and retroperotineal fibrosis
Establish etiology
o Primary
AD
Young males
o Secondary/ associations
Chronic ethanol ingestion parotidomegaly, hepatomegaly, CLD
Antiepileptics gum hypertrophy, nystagmus
DM especially insulin dependent
Hyperlipdaemia (xanthelesma)
Smoking (nicotine staining of the fingers)
Manual labor and hand trauma
Fibroproliferative disorder
Mx
o Most do not require
o Annual follow up for contracture developing
o Intralesional injection of collagenase or interferon gamma
o Surgical correction (if >30 contractures of MCPJ or any degree of contracture of PIPJ)
70. Clubbed Fingers
Rule out
o Pseudoclubbing of scleroderma and clubbing of thyroid acropachy
o Grade
1 Fluctuation of the nail bed (sponginess)
2 Scaramoths sign (obliteration of the diamond when dorsal surfaces of the terminal phalanges are opposed
together), loss of Lovibonds angle
3 Drumstick appearance ie enlarged finger tips
4 associated with hypertrophic pulmonary osteoarthropathy of wrist and ankle (subperiosteal reaction and new
bone formation)
o Causes
Lung
Abscess, bronchiectasis
Pulmonary fibrosis
Ca lung
CVM
Cyanotic congenital heart disease
Eisenmengers syndrome
Infective endocarditis
Abdomen
Cirrhosis
Inflammatory bowel disease
Coeliacs disease
Thyroid
Familial
Idiopathic
o Unilateral causes
AVM of the Lung
Axillary artery aneurysm

160

71. Painful/Swollen Knee Joint


Examine the joint
o Look, feel, move approach, disuse atrophy
o Gait
o Make sure its not Charcots joint (see Charcots joints)
o Examine other joints and proceed to exclude causes
OA
RA palindromic type (acute recurrent arthritis, affecting one joint)
Seronegative spondyloarthropathy (4)
Crystal arthropathy gout, pseudogout or chondrocalcinosis
Septic arthritis
Haemophilia
Trauma
Others Lymes, Rh heart Disease
72. Stills disease/Juvenile Chronic Arthritis
Cs (Stills) micrognathia, arthropathy of the DIPJ, splenomegaly, LNs, Hx maculopapular rash
Dx of JCA
o <16 yrs, >6 weeks of arthritis with no other cause, > 6 months evolve
Stills disease arthritis with daily temperature spikes
Polyarticular (>5 joints) with early fusion of cervical and mandibular joints
Pauciarticular (4 or less) iritis in girls and sacroilitis in boys
Joint involvement with RF and ANA negative
Systemic symptoms of lethargy, anaemia, growth disturbance, amyloidosis
Rx
o Education, counseling andsupport
o PT/OT
o Analgesia, intra-articular steroids, MTX, hydroxychloroquine
73. Enteropathic Arthropathy
2 types
o Peripheral arthropathies
Non-erosive, migratory and reversible of the large joints (knees, ankles and elbows), occasionally MCPJ and
PIPJ of the hands
Parallels disease activity and improves when bowel disease improves
o Axial arthropathies
Similar to AS (X-ray are similar)
Independent of bowel disease activity
Look for abdominal scar, erythema nodosum

161

74. Old rickets


Consider this diagnosis
o Pagets (see Pagets)
o Short stature
Achondroplasia (AD, short stature, prominent forehead, saddle nose, short limbs with no bowing, exaggerated
lumbar lordosis, normal trunk, occ spinal cord compression) with normal sexual and mental fuction and life span
Noonans syndrome (AD)
Short stature, facial abnormalities(hypertelorism, triangular facies, webbed neck, ptosis, down-slanting
eyes), pectus excavatum or carinatum, hyperextensible joints
Cardio PS, ASD, VSD
Mental retardation
Impaired blood clotting
Turners syndrome
45XO
In females only
th
Cs short stature, shield chest, short 4 MC/MT, cubitus valgus, webbed neck, absent breast
development with normal pubic hair, cutis laxa(excessive skin), hypoplastic/hyperconcex nails, naevi
CVM coarctation of aorta, biscupid aortic valve, aortic dissection
Hypothyroid
Ovarian failure with high FSH and LH
Cs of Rickets
o Bilateral
o Symmetrical
o Bowing (lateral curvature of the tibia and femur, radius and ulna)
o Short stature
o No increased warmth
o Frontal bossing and parietal flattening
o Rickety rosary (thickening of costochondral junction), Harrisons groove (indentation of the lower ribs at the diaphragmatic
attachment)
Causes of rickets or osteomalacia
o Decreased Vit D sun exposure, malnutrition
o Malabsorption gastrectomy, coeliacs, pancrease
o Abnormal metabolism CRF, liver dz, RTA, anticonvulsants
o Others
Familial hypophosphataemia
Vit D resistant rickets
Rickets occurs before closure of epiphyses compared to osteomalacia
o Osteomalacia with bone pain, prox weakness with pseudofractures or loosers zones in the ribs, pelvis and
clavicles/scapula
Raised ALP, low Ca, high PTH and low Vit D

162

ENDOCRINE!
75. Acromegaly
Stem Statement
Please examine hands, face, look and proceed.
Patient has headaches, increased sweatiness
Approach
1. Hands
a. Palm downwards large, doughy, spade shaped, OA, double pinch test
b. Palm upwards sweatiness, CTS, wasting of thenar eminence, numbness
2. Elbows ulnar nerve thickening
3. Proximal myopathy
4. Face Transfrontal scar, prominent supraorbital ridges, greasy skin, broad nose, hirsute, thickened lips, macroglossia,
teeth indentation marks on the side of the tongue, prognathism, splaying of teeth, malocclusion of teeth
5. Neck Goitre
6. Lower limb bowed legs, OA, pitting edema from CCF/CCB, heelpad thickened
7. Request for patient to remove shirt to inspect the trunk and axillae
a. Skin tags
b. Coarse body hair
c. Acanthosis nigricans
d. Gynaecomastia, galactorrhoea
e. Kyphosis
8. Request
a. Visual fields bitemporal hemianopia, fundoscopy for angiod streaks
b. CVS cardiomegaly
c. Abdomen organomegaly, testicular atrophy, PR bleed for Ca colon
d. BP - Hypertension
e. Urine dipstick glycosuria
Presentation
Sir, this patient has acromegaly as evidenced by presence of coarse facial features with prominent supraorbital ridges, broad nose
and thick lips; a/w macroglossia with teeth indentation marks on the side of the tongue. There is also presence of splaying of the
teeth with malocclusion and prognathism. I did not notice any scars on the forehead to suggest previous Transfrontal surgery.
There is also no goitre
There is presence of a large, spade like doughy hands with no sweating detected. There is no wasting of the thenar eminence and
Tinels sign was negative. There are also no thickened ulna nerves at the elbows and no proximal myopathy. No features of OA of
the hands and no bowing of the tibia. No pedal edema but presence of thickened heelpads.
I would like to complete the examination by
1. Asking the patient to remove his shirt to look for Acanthosis nigricans, coarse body hair, skin tags, kyphosis and
gynaecomastia/galactorrhea
2. Visual fields for bitemporal hemianopia
3. CVS cardiomegaly
4. Abdomen exam for organomegaly
5. BP
6. Screen for DM
7. Ask for symptoms of headache, increase sweatiness and recent increase in shoe or glove size.
Questions
1. What is acromegaly?
a. Due to excess GH activity as a result of a pituitary macroadenoma occurring post puberty
2.

What are the indicators of activity?


a. Skin tags
b. Increased sweatiness, headache
c. Increased size of goitre/visual field loss/size of hands/Shoe size
d. Hypertension
e. Glycosuria

3.

What are angiod streaks?


a. Degeneration and fibrosis of Bruchs membrane

4.

List causes of macroglossia.


a. Acromegaly
b. Hypothyroidism
c. Amyloidosis
d. Down syndrome
e. Haematological malignancy

5.

What is the pathology of acromegaly?


a. Pituitary macrodenoma

163

6.

What are the complications?


a. Metabolic and endocrine
i. Diabetes mellitus in 20% of patients
ii. Hypertriglyceridemia in 40% of patients
b. Cardiovascular
i. Hypertension
ii. Cardiomyopathy and CCF
c. Respiratory
i. Acute dyspnea and stridor (upper airway narrowing)
ii. Obstructive sleep apnea
d. Abdomen
i. Colonic polyps and malignancies (ie, colon cancer)
ii. Organomegaly, testicular atrophy
e. Neuromuscular
i. Proximal myopathy
ii. Nerve root compression CTS, radiculopathy
iii. Spinal stenosis
f.
Calcium and bone metabolism
i. Hypercalciuria
ii. Hyperphosphatemia
iii. Urolithiasis

7.

How do you investigate?


a. Confirm the diagnosis by OGTT to look for non supressibility of GH (2ng/ml), can also screen for DM
b. Other useful blood Ix
i. IGF-1 as a baseline and monitoring disease activity and treatment
ii. Prolactin levels as 20% are associated with hyperprolactinaemia
1. low in hypopit
2. High because 1. Co-secretion 2. compression of pit stalk with interference of dopaminergic suppression
of prolactin production
iii. Pituitary function (SST, TFT, FSH/LH/Testos/Oestradiol)
iv. Calcium levels MEN type 1 syndrome
c. Imaging (after diagnosis is confirmed)
i. MRI of the pituitary fossa macroadenoma
ii. X-rays
1. Skull Enlarged sella turcica, enlarged frontal, ethmoid and mastoid sinuses, thickened calvarium,
enlarged mandible
2. CXR cardiomegaly
3. Hand and feet X-rays terminal phalangeal tufting and thickened heel pad (>23mm thick on a lateral Xray)
d. Others
i. Formal perimetry
ii. Obtain old photos
iii. ECG - LVH

8.

How would you manage?


a. The definitive therapy is surgical which can be via transphenoidal or the transfrontal approach
b. Radiation therapy if pt is not a suitable candidate
c. Medical therapy
i. Bromocriptine dopamine agonist (PO)
ii. Octreotide or long acting type (SC, daily vs monthly)
iii. GH receptor antagonist pegvisomant which is a recombinant DNA analogue (SC daily)

9.

What are the conditions with excess GH besides acromegaly?


a. MEN type 1 (PPP)
b. McCune Albright syndrome Polyostotic fibrous dysplasia, sexual precocity and caf-au-lait spots
c. Carney Complex multicentric tumors in multiple organs, pigmented skin lesions and pigmented nodular hyperplasia (aut
dominant)

164

76. Cushings Syndrome


Examination
Exposure remove shirt
Seated
Face
Round or moon facies
Plethora
Telangiectasia
Cataracts, anaemia
Oral thrush, buccal pigmentation
Hirsutism
Acne
Neck
Supraclavicular fat pads
Dorsal hump or buffalo hump (interscapular fat pad)
Upper limbs
Hands
clubbing, nicotine staining, hyperpigmentation
RA, SLE
Bruising
Papery thin skin (use 2 index fingers in a circular fashion)
Peripheral wasting of Uls
Proximal myopathy
Acanthosis nigricans
Abdomen
Truncal obesity
Purple striae
Lower Limb
Edema
Bruising
Stand the patient up
Look for kyphoscoliosis
Palpate/percuss for tenderness (osteoporosis and vertebral collapse)
Ask the patient to squat and then stand up
Request
BP
Urine dipstick
Examine the lungs for asthma and pulmonary fibrosis
Visual field assessment although majority of pituitary adenoma are microadenoma
Panhypopituirism
MEN type 1
Presentation
Sir, this patient has got Cushings syndrome. There is presence of moon-facies with facial plethora and telengiectasia. There is
presence of hirsutism, acne, oral thrush and cataracts or conjunctival pallor. This is associated with supraclavicular as well as
interscapular fat pad deposition. There is truncal obesity associated with purple striae. There is bruising of the skin and the skin is
papery thin skin with proximal myopathy and lower limb edema. There is no evidence of acanthosis nigricans. There is
kyphoscoliosis with tenderness of the spine.
There was no clinical evidence of RA such as symmetrical deforming polyarthropathy or SLE.
I would like to complete the examination by
Respiratory examination for evidence of asthma or pulmonary fibrosis
BP
DM
Ask history of exogenous steroid intake
Virilisation deepening of voice, breast atrophy, clitoromegaly

165

Questions
What are the causes of Cushings syndrome?
Rule of 90:10
90% exogenous and 10% endogenous
of the 10% endogenous
90% ACTH dependent and 10% ACTH-independent (adrenal adenoma and carcinoma)
of the 90% ACTH dependent
90% are Pituitary(Cushings disease) and 10% are ectopic ACTH (bronchial carcinoid, small cell lung ca, pancreatic
carcinoma, non-teratomatous ovarian tumor)
of the Pituitary adenoma
90% are microadenoma
10% are macroadenoma
ACTH independent
Adrenal adenoma
Adrenal carcinoma
Micro/macronodular adrenal hyperplasia
Part of Carney complex (pigmented skin lesions with endocrine and mesenchytmal tumors)
McCune Albright syndrome
What are the causes of PseudoCushings?
DOA
Depressions, drugs
OCPs, obesity
Alcoholism, acute illness
What are purple striae?
Purple striae are due to the weakening and disruption of the collagen fibres of the dermis leading to exposure of the underlying vascular
tissue. They can be found on the abdomen, the upper arms and on the medial aspects of the thighs.
What are the signs suggesting ectopic ACTH secretion?
Absence of Cushingoid habitus, prominent edema and hypertension and marked muscle weakness.
What are the features that suggest adrenal carcinoma?
Virilisation in the female, gynaecomastia in a male and a palpable abdominal mass.
What is the significance of hyperpigmentation in a Cushingoid patient?
It implies that Cushings syndrome is due to ACTH excess due to presence of MSH like activity of the ACTH molecule.
What is Nelsons syndrome?
Nelson syndrome occurred formerly as a result of bilateral adrenalectomy for Cushings disease
Resulting in absent negative feedback of cortisol on the pituitary adenoma, with expansion of the pituitary adenoma with headache,
bitemporal hemianopia and panhypopit eventually
Occurs in 20% of such patients in the past
Hyperpigmentation occurs due to melanocyte stimulating component of the precursor molecule of ACTH.
How would you investigate this patient?
Screen with
24H urinary cortisol or
overnight dexamethasone suppression test
1mg between 11pm to 12 midnight
Serum Cortisol at 8 am the following morning (>2mcg/dl)
Confirm diagnosis with a low dose dexamethasone suppression test
Determine the cause with
Plasma ACTH
High dose dexamethasone suppression test
Imaging studies (MRI pituitary or CT adrenals)
Others
CXR if ectopic ACTH suspected
AXR for adrenal calcification
CRH test (distinguish ectopic CRH vs Cushings disease)
Inferior petrosal sinus sampling (distinguish primary and ectopic source of ACTH when above tests are inconclusive)

166

How would you manage this patient?


Treatment is directed at the primary cause of the syndrome:
Exdogenous steroids
Withdrawal if possible
If not possible
Monitor for complications and treat them
BP
DM
PUD
Osteoporosis
Endogenous
ACTH dependent
Cushings disease
Transphenoidal hypophysectomy
Transfrontal hypophysectomy
Pituitary irradiation
Bilateral adrenalectomy with pituitary irradiation
Ectopic ACTH
Locate the source and treat appropriately
If unable to, adrenalectomy or medical therapy
ACTH independent
Adenoma/carcinoma unilateral adrenalectomy
Hyperplasia bilateral adrenalectomy
Medical therapy only used if surgical not possible, eg metastatic adrenal carcinoma or ectopic ACTH; can use mitotane, ketoconazole,
metyrapone, aminogluthithemde, trilostane and etomidate.

167

77. Goitre
Examination
Look and proceed, Look at the eyes/face (Graves ophthalmopathy)
Examine hands (thyroid signs)
Examine lower limbs (pretibial myxoedema)
Examine her neck (start from neck)
Assess her thyroid status (start from peripheries)
General inspection thin, fidgety and may have choreoathetoid movements
ULs
o Both ULs up with dorsum facing upwards
Tremors
Acropachy (thyroid clubbing)
Onycholysis (Plummers nails especially ring finger)
Skin for vitiligo
o Both ULs with palm facing upwards
Sweaty palms
Palmar erythema
o Proximal weakness
o Pembertons sign
o Measure pulse for ST or AF
o Reflexes
Eyes
o Look
Chemosis, keratitis, prominent caruncle and tarsorrhaphy
Lid erythema and periorbital edema
Exomphthalmos and lid retraction (Dalrymples sign)
o Move
Lid lag (von Graefes sign)
Ophthalmoplegia
Order of muscles affected IM So Lazy
o Inferior, medial, superior and lateral recti
Neck
o Goitre swallow water
o Look for scar (think of hypothyroid and hypoparathyroid) and distended neck veins
o Walk to the patients back
Observe for proptosis
Palpate the goitre (soft, smooth vs nodular, large, tender)
Palpate for Cx LNs, carotid pulsations
o Listen for bruit
o Palpate for tracheal deviation and SCM weakness on MNGs
o Percussion of sternum
LL
o Pretibial myxedema
Complete examination
o Reflexes for hyperreflexia
o Cardiovascular examination
Wide pulse pressure (if clinically hyperthyroid) and systolic hypertension
ESM,CCF
Gynaecomastia
o If there is a scar, request to perform Trousseaus sign and Chvosteks sign for hypoparathyroidism, assessment
for hoarseness of voice
o Abdominal examination may reveal hepatosplenomegaly in Graves disease

168

Presentation
Graves disease
Sir, this patient has got Graves disease and is clinically hyperthyroid complicated by Graves ophthalmopathy.
There is presence of a diffusely enlarged, smooth and firm goitre which is associated with a bruit and is non-tender. There are no
palpable LNs and tracheal is central with no dullness to percussion of the sternum. Pembertons sign is negative.
There is evidence of hyperthyroidism. Patient is thin looking and is anxious and fidgety with presence of fine tremors of the
outstretched hands, sweaty palms, with palmar erythema and a resting sinus tachycardia. I did not notice any thyroid acropachy or
onycholysis. There is also no evidence of proximal upper limb weakness.
Examination of the eyes reveals presence of lid retraction with a staring appearance. There is no chemosis, keratitis or evidence of
tarsorraphy. There is evidence of exomphthalmos and proptosis. There is no ophthalmoplegia.
There is no evidence of pretibial myxedema.
Multinodular Goitre
Sir, this patient has MNG and is hyperthyroid complicated by atrial fibrillation.
There is presence of an enlarged goitre with multiple nodules bilaterally with a dominant nodule in the right lobe of the thyroid
gland. This is non tender. There is no associated Cx LN and the carotid artery is palpable.
There are no signs of compression such as stridor, negative Pembertons sign with no dullness to percussion of the sternum.
There are signs of hyperthyroidism.
The patient is in atrial fibrillation; did not notice any easy brusibility or obvious hemiplegia
Questions
What is Graves disease?
Autoimmune disease
TSI binds to and stimulates the TSH receptor on the thyroid cell membrane
Resulting in excessive synthesis and secretion of thyroid hormone
nd
th
2% in women and 0,2% in men; 2 to 4 decades
What are the clinical signs specific to Graves disease?
Graves ophthalmopathy
Pretibial myxedema
Thyroid acropachy
Diffuse goitre
Lymphoid hyperplasia
What is Graves Ophthalmopathy?
Characterised by
edema and inflammation of the extraocular muscles
increase in orbital connective tissue and fat
edema is due to hydrophilic action of the glycosaminoglycans secreted by fibroblast
inflammation is due to infiltration by lymphocytes and macrophages
Worst in
Smokers, elderly males
Post radio-iodine treatment
Severe hyperthyroidism
Can occur pre, during or post diagnosis of hyperthyroidism
How do you assess activity of the eye disease?
Retrobulbar pain
Pain on eye movement
Eyelid erythema
Conjunctival injection
Chemosis
Swelling of the caruncle
Eyelid edema
Points system together with degree of proptosis (Hertels ophthalmometer), reduced VA and eye movements

169

What is pretibial myxedema?


Specific feature of Graves disease
Types
o Lymphedema type
Symmetrical, well defined, waxy and shiny peau dorange appearance
Red but not inflamed, swollen but not edematous
o Nodular type
o Plague type
Occurs on the shins, anterior lateral aspects
Can also occur as localised dermopathy at sites of trauma
Characterise by edema, accumulation of glycosaminoglycans and lymphocytic infiltrates
Usually after treatment of hyperthyroidism, especially after radioactive iodine
What are the signs of hyperthyroidism?
Resting tachycardia (important)
Sweaty palms
Tremors
Hyperreflexia
Thyroid bruit
What are the causes of hyperthyroidism?
Primary
Graves disease
Toxic MNG (Plummers disease)
Toxic adenoma
De Quervains thyroiditis
Post partum thyroiditis (Characteristics: Reduced radionuclide uptake, low T3/T4 ratio and raised Thyroglobulin level)
Secondary
Pituitary
Struma ovarii, hydatidiform mole or choriocarcinoma (ectopic TSH)
Exogenous
Overtreatment (eg in thyroid cancer)
Factitious
Drug induced Lithium, amiodarone (type 1 i.e. iodine induced and type 2 i.e. inflammatory thyroiditis)
What are the differential diagnoses of swellings in the neck?
Midline
o Thyroid gland which rises on swallowing
o Thryroglossal cysts which also rises on swallowing but also moves on sticking out the tongue
o Submental LNs
Lateral
o LNs
o Salivary Glands
o Skin sebaceous cysts or lipoma
o Cystic hygroma
o Pharyngeal pouch
How would you grade the goitre?
WHO grading:
Grade 0 : not palpable or visible
Grade 1A: palpable goitre
Grade 1B : palpable and visible only on neck extension
Grade 2: Visible goitre at primary position
Grade 3: Obvious goitre from a distance
What is Pembertons sign?
Elicited by asking the patient to lift his arms above her head
Development of plethora, cyanosis, inspiratory stridor and respiratory distress and distension of neck veins
Test for thoracic inlet obstruction due to a retrosternal mass
How do you differentiate between thyroid acropachy and HPOA?
Radiographically
o Thyroid acropachy new bone formation has a soap bubbles appearance on the bone surface with coarse spicules
o HPOA new bone formation in a linear distribution

170

What are the associated clinical conditions with Graves disease?


Diabetes mellitus
Vitiligo
Pernicious anaemia
Addisons disease
Myasthenia gravis
Alopecia areata
How would you investigate this patient with Graves disease?
Confirm the diagnosis
o Thyroid stimulating hormone levels (aka thyrotropin levels)
o Free thyroxine levels; KIV serum free tri-iodothyronine
o Autoantibodies such as TSH receptor Ab(TRAb), thyroid peroxidase antibodies and thyroglobulin antibodies
o Occasionally, to differentiate between Graves and autoimmune thyroiditis, radionuclide scan which shows diffuse
uptake in Graves and no/low uptake in autoimmune thyroiditis
Ophthalmopathy
o CT or MRI orbits to rule out retrobulbar tumor or AVM especially in unilateral exomphthalmos
How would you investigate this patient with MNG?
Activity assessment fT4 and TSH
Imaging CT neck to look for obstruction
Radionuclide in a predominantly hot nodule
What is T3 thyrotoxicosis?
Hyperthyroid symptoms and sign
normal fT4 (thyroxine) level
elevated T3 (triiodothyronine).
What is sick euthyroid?
Occurs in patients with severe illness or physical trauma
Alterations of peripheral transport and metabolism of thyroid hormones
Low fT4 and T3 and inappropriately low TSH

171

How would you manage this patient?


(1) Graves disease:
Medical therapy
o Symptomatic treatment with propranolol
o Carbimazole, methimazole and propylthiouracil
o All inhibit thyroid peroxidase and hence thyroid hormone synthesis
o PTU also inhibit conversion of fT4 to fT3, useful in crisis
o CMZ and MTZ useful as fewer tablets and once daily dosing
o Treated for 12-18 months and 30-40% will remain euthyroid
o If it recurs, likelihood of remission on medications is low
o Minor adverse effects (5%)
fever, rash, urticaria and arthralgia
o Major adverse effects (0.5%) (CMZ and MTZ are dose related and PTU is not)
agranulocytosis
Advised to stop the drug if develop fever, sorethroat or mouth ulcers
Severe hepatotoxicity
Vasculitis
Lupus-like syndrome
Radio-iodine (131-I at dose 5 to 15 mCi)
o 90% will become euthyroid within 2 months.
o Contraindicated in pregnant and breast-feeding mothers, children and adolescent
o Side effects
Almost all will become hypothyroid
Neck pain
Worsened thyrotoxicosis for several days post treatment
Prevented with CMZ/MTZ pre-treatment for 1-2 months and stopped 3-5 days before
treatment; try not to use PTU as this decrease efficacy of I -131 treatment
Observed if mild or treat with beta blockers
Should not give antithyroid medications unless severe or expected to be severe due to poor
control at the time of I-131 administration
Worsening of ophthalmopathy
especially in smokers and severe hyperthyroidism
Administration of glucocorticoids can prevent worsening
Thyroidectomy
o Indications includes Cs: Cancer (dominant nodule), cosmesis, compression
o Effective in 90%
o Not a/w worsening of Graves ophthalmopathy
o Side effects : recurrent laryngeal nerve, hypoparathyroidism (1-2%)
o Medications given prior to surgery and Lugols iodine given 7-10 days prior to surgery
(2) MNG
Render euthyroid with thionamide
As spontaneous remission does not occur, ablative therapy required
No obstruction Radio-iodine
Obstruction Surgical
(3) Toxic Adenoma
Render euthyroid with thionamide
Radio-iodine hypothyroidism side effect is less compared to Graves disease as the toxic adenoma suppresses the
other thyroidal tissue
Surgical Lobectomy
(4) Subacute thyroiditis
Should not Rx with thionamides
Rx with propranolol, aspirin, NSAIDs and glucocorticoids
How would you counsel a young woman with thyrotoxicosis who wishes to be pregnant?
Ideally, pregnancy should be avoided until hyperthyroidism is adequately treated because the rate of fetal loss is high
If it occurs or recurs during pregnancy, then
o Treat with PTU
Lowest dose possible such that fT4 is at the upper range of normal
Combination therapy contraindicated because PTU passes the placenta but thyroxine doesnt, resulting
in fetal hypothyroidism
PTU better because of better binding to proteins and therefore less transplacental transfer theoretically;
also CMZ a/w rare side effects of aplasia cutis congenita, esophageal and choanal atresias
o Can also be safely treated with surgery in the second trimester with almost no risk of death in experienced hands
rd
o In the 3 trimester, TSI levels declines and remission of hyperthyroidism occurs; stopping medications is possible
then
o 1-5% of fetuses may be hyperthyroid resulting in IUGR and tachycardia
o up to 750mg/d PTU or 20mg CMZ can be safely used in lactating mothers

172

How do you treat Graves ophthalmopathy?


General measures
o Maintenance of euthyroidism
o Stop smoking
o Sleep with head raised
o Use of artificial tears
o Diuretics
o Tinted glasses
Specific measures (for severe disease)
o Glucocorticoids 40-80mg OM then taper over 3 months
o Radiotherapy
o Surgical decompression
Stable disease
o Surgery for lid retraction, exomphthalmos or diplopia

173

78. Pagets disease


(Spot diagnosis large head with hearing aid and its not Acromegaly!)
Examination
Head
Enlargement of the skull, especially in the frontal and parietal
Measure circumference (>55cm = abnormal)
Prominent scalp veins
Palpate skull for warmth
Auscultate the skull for bruit
Face
RAPD, VA and visual fields
Hearing aids, tests for deafness (conduction vs SNHL)
Other CNs
Neck
Platybasia (basilar invagination) short neck, low hairline
JVP
Back for kyphosis, tenderness, warmth and systolic bruits
Upper limbs
Obvious bowing of the long bones
Cerebellar signs from platybasia
Lower limbs
Lateral bowing of the femur
Anterior bowing of the tibia
Warmth
OA knees
Obvious paraplegia
Bilateral pedal edema
Request for
Fundoscopy to look for optic atrophy and angiod streaks
Neurological examination of the LLs and ULs for cord compression or nerve root compression signs
Urinalysis
Hematuria from urolithiasis
Presentation
Sir, this patient has Pagets disease as evidenced
Bony features
enlarged skull, >55cm, short neck and low hairline, back, UL and LL bowing
Warmth, tenderness, systolic bruits
OA knees
Neurological
VIII nerve (hearing aid), CNs
Cerebellar
Obvious paraplegia
CVS no raised JVP or bilateral pedal edema
Complete my examination
Fundoscopy
Urinalysis
Neurological examination
History of increase in hat size
In summary, patient has Pagets disease with complication of left-sided deafness requiring a hearing aid.

174

Questions
What are the differential diagnoses for bowing of the tibia?
Pagets disease (Asymmetrical)
Rickets disease (bilateral symmetrical)
Congenital syphilis
Yaws
Periosteitis with apparent bowing
What is Pagets disease?
Metabolic disease characterised by excessive and abnormal remodelling of bone
Primary defect lies in increased osteoclastic activity with increased bone resorption and increased osteoblastic activity
There is excessive bone resorption with compensatory disorganised deposition of new bone
Males 2X more common and increases with age
Association with measles and paramyxovirus, cause is unknown
Stages
Lytic phase then
Mixed intermediate phase (lytic and blastic)
Sclerotic phase
What are the complications?
Bony and immobilisation
Pathological fractures
Sarcomatous change in 1%
OA
Protrusio acetabuli
Neurological
Obstructive hydrocephalus
CNs
Hearing loss
Conductive more commonly for otosclerosis of the ossicles
Sensori-neural hearing loss from auditory nerve compression
Optic atrophy
Spinal cord compression(basilar invagination) or nerve root compression
High-output cardiac failure
Metabolic
Gout hyperuricaemia from rapid bone resorption during prolonged immobilisation
Urolithiasis from hypercalciuria
Hypercalcaemia from immobilisation
How would you investigate?
Urinary hydroxyproline increased
Reflects increased osteoclastic activity and bone resorption
Fasting sample required
False positive if diet contains hydroxyproline, skin disease
Blood Ix
Serum Ca and PO4 normal but high in prolonged immobilisation or malignancy
Serum ALP high (increased osteoblastic activity)
Imaging
Skull - cotton wool appearance, osteoporosis circumscripta
Pelvis brim sign thickening of the iliopectineal line
Vertebrae picture frame sign with sclerotic margins
Long bones increased trabeculation and localised bone enlargement
Bone scan increased uptake reflects activity and useful for monitoring disease
How would you manage?
Education and counselling
Most are asymptomatic and do not require treatment
PT, OT and ST
Symptomatic
Painkillers
Treat disease
Indications
Bone pain, osteolytic lesions in weight bearing bones, delayed or non-union of fractures, neurological complications
(except hearing loss), cardiac complications
Bisphosphonates and salmon calcitonin
Treat complications
What are angiod streaks?
Linear disruptions of Bruchs membrane with proliferative connective tissue emerging through the defects.

175

79. Panhypopituitarism (Simmonds disease)


Clinical
o Pale, soft skin, loss of axillary hair, breast atrophy, hypogonadism, gynaecomastia and galactorrhea
(hyperprolactinaemia), pallor and hairlessness (=alabaster skin)
o Features of hypothyroidism
o Postural hypotension
Etiology
o Visual fields for bitemporal hemianopia (Chromophobe adenoma)
o Fundoscopy for papilloedema
o Radiation marks or signs
o Surgical scar marks
o Others
Postpartum Necrosis (Sheehans syndrome)
Pit apoplexy (spont infarct or hemorrhage)
Craniopharyngioma
TB, sarcoid, metastatic
Ix
o Blood test
FBC NCNC anemia
U/E hypoNa,
LF/FSH, GH, PRL, TSH, ACTH
Testosterone, estradiol, T4, Cortisol level
Stimulation test Synacthen test, insulin hypoglycemia test (gluc, 2.2 and check cortisol an GH)
o Imaging MRI
o Formal perimetry
Notes
o Fail in order of FSH/LH, GH, PRL, TSH and ACTH and lastly ADH
o Usually macroadenoma for acromegaly and non functioning tumors
o Mx replacement of steroids and thyroid and sex hormones and GH
o Must always replace steroids first then thyroid to prevent adrenal crisis
o Steroid replacement may unmask diabetes insipidus

176

80. Addisons disease


Clinical
o Weakness, LOA and LOW
o Hyperpigmentation
Crease of the palms
Mouth and lips
Nipples, belt, straps, rings
o Sparse axillary hair, pubic hair, postural hypotension
o Associations vitiligo, polyglandular (hypoparathy, DM, thyroid)
o Dy/Dx Nelsons syndrome = look for abdominal scar and visual field, Liver, renal
Ix
o Confirm dx with synacthen test
o Confirm level
ACTH
Prolonged ACTH test
Will respond if there is suppression by exogenous steroids or ACTH deficiency
Imaging
o If adrenals
AXR (calcification)
CT adrenals
CXR : TB
Adrenal Antibodies
Mx
o Replace steroids
o Fludrocortisone
o Steroid card
Notes
o Causes of hyperpigmentation
Addisons, Nelsons, ectopic ACTH
Liver PBC, hemochromatosis
Uremia
Race, suntan
Porphyria cutanea tarda
o Causes of Addisons
AI (21 hydroxylase)
TB
Mets
HIV
o Association
Graves, Hashimoto, Pernicious anemia
AI polyglandular syndromes
Type 1: Addisons, hypoparathy, chronic mucocutaneous candidiasis
Type 2: Addisons, hypothy, DM
81. Gynaecomastia
Physiological
o Newborn
o Adolescence
o Ageing
Pathological
o Cirrhosis of the liver, renal failure
o Hyperprolactinoma, thyrotoxicosis
o Klinfelters
o Malignancy (HCC, Lung CA, Testicular)
Pharmacological
o Digoxin, spironolactone, cimetidine, methyldopa, diazepam

177

DERM!
82. Dermatology Overview
rd
Note: Number in brackets refer to the page numbers of 250 Cases In clinical Medicine by R.R Baliga, 3 Edition.
Generalised
Maculopapular rash (389)
Erythema multiforme (423)
Steven Johnson Syndrome (423)
Dy/Dx Staphylococcal Scalded Syndorme
Purpura (see notes)
Henoch Scholein Purpura (399)
Systemic Lupus Erythematous (417)
Bullous eruption (396)
Dermatitis herpetiformis (456) (dy/dx scabies)
Herpes zoster
Herpes labialis
Urticaria (496)
Vasculitis
Urticaria pigmentosa (500)
Eczema (475, 484)
Kaposi (460)
Mycosis fungoides (498)
Psoriasis (see notes)
Dy/dx (mycosis fungoides and Bazex syndrome)
Lichen Planus (see notes)
Purple hue dermatomyositis, sarcoid
Erythroderma (see notes)
Sezary syndrome (mycosis fungoides)
Multiple lumps
NF
Dercums disease
Multiple exostosis
Gardners syndrome

178

Face
Alopecia (472)
Scarring (infective/Discoid lupus, Lichen planus)
Non-scarring (alopecia areata, totalis, universalis/telogen effluvium/male balding)
Acne (470)
Rosacea (454)
SLE (417)
Dermatomyositis (see notes)
Mitral stenosis
Seborrheic dermatitis (493)
Telengiectasia (403)
Sturge Weber (468)
Peutz Jeghers (463)
Hereditary Haemorrhagic Telengiectasia (403)
Shovlin Criteria (epistaxis, telangiectasia, visceral and AD)
Dy/dx (CLD, AI, rosacea, lupus pernio,radiation)
Lupus pernio (435)
Nose disfigurement
Lupus pernio
Lupus vulgaris
Leprosy
Rhinophyma
Xanthelesma (See Limbs)
Hirsutism (428)
Radiation marks (444)

179

Limbs
Ulcers of Lower Limbs (477,478)
Pyoderma gangrenosum (466)
Loose skin
Pseudoxanthoma elasticum (451) (also has grouped papules)
Ehlers Danlos (508)
Hyperextensible Joints
Ehlers Danlos (508)
Marfan (see rheumatology)
MEN type 2B
Klinefelters

Ichthyosis (401)
Inherited or malignancy(Breast, haematological)
Raynauds phenomenon (415)
Phleblitis migrans (421)
Erythema ab igne and livedo reticulais (426)
Bad nails
Psoriasis
Lichen planus
Alopecia areata (472)
Fungal (482)
Lipoatrophy (433)
Xanthelasma (439)
Tendon xanthomaa (446)
Eruptive xanthoma (448) (dy/dx- pseudoxanthoma elasticum, molluscum contagiosum 494)
Palmar xanthoma (450)
Tuberous xanthomas
Shin lesions
Necrobiosis lipodica diabeticorum (442)
Pretibial myxedema (514)
Erythema nodosum (480)
DM dermopathy
Erythema ab igne
Livedoreticularis
Melanoma (490)
Hypopigmentation
Vitiligo (412)
Post inflammatory
Pityriasis alba
Tinea versicolor

180

83. Psoriasis Locomotor (10% of Psoriasis with Joint involvement)


Presentation
Sir, this patient has psoriatic arthropathy affecting the hands of the
6. Arthritis mutilans type (bilateral deforming arthropathy, telescoping of the digits)
7. RA type (symmetrical joint involvement)
8. OA type (asymmetrical terminal joint involvement)
9. mono/oligoarticular type
10. AS type (Sacroilitis, but the syndesmophytes arise from the lateral and anterior surface and not at the margins unlike AS)
With
7.
8.
9.
10.
11.

Bilateral deforming polyarthropathy, and joint deformities, tender (activity)


sausage shaped fingers, tenosynovitis
wasting dorsal guttering and wasting of the thenar and hypothenar eminence
nails pitting, onycholysis, subungal hyperkeratosis, discoloration of the nails (80% involvement with arthropathy)
Skin patches well circumscribed plagues on the extensor surfaces of the elbows and scalp, with salmon pink hue and
silvery scales
12. surgical scars

Joint function
6. Impaired or preserved
7. able to grip and do pincer movement
8. coarse function turn a doorknob
9. fine function cap a pen, transfer coins, unbutton clothes
10. able to abduct and internally rotate her shoulder joints which are important for her ADLS
Treatment complications Steroids for arthritis
Mention no evidence of Gout (as this is associated with Psoriasis)
Complete my examination by
examining for other joint involvement
Skin especially scalp, knees, natal cleft, intragluteal folds, submammary folds, Koebners phenomenon
Enquire on aggravating factors
Questions
What are the types of skin lesions?
Plague
Guttate (numerous small papular, hx of streptococcal infection
Pustular (localized or generalized, superficial pustules may stud the plagues)
Erythrodermic (generalized erythema and scaling which may be life threatening)
Inverse psoriasis (plagues evolving in the intertriginous area without typical silvery scales due to moisture and
maceration)
Where are the typical sites of distribution?
Extensor surfaces of knees, elbows, scalp, navel, natal cleft, submammary and intragluteal folds
How do you assess severity?
Psoriasis Area and Severity Index area, thickness, redness and scaling
Total score 72 - <10, 10-50, >50 for mild, moderate and severe respectively
What are the types of joint involvement in psoriasis?
OA
RA
AS
Oligo/mono
Arthritis mutilans
Radiological features of psoriatic arthritis?
Periostitis fluffy
Destruction of small joints
Pencil in cup appearance
Non marginal syndesmophytes in AS type
What are the unique characteristics of psoriatic lesions?
Salmon pink hue with silvery scales
Koebners - New psoriatic skin lesions at site of cutaneous trauma
Moist red surface on removing of scales (Bulkeleys membrane)
Auspitzs sign capillary bleeding when silver scales are picked from the plague

181

What is Koebners phenomenon and which other conditions is it seen?


New skin lesions at the site of cutaneous trauma
Occurs in 30% of patients with psoriasis, usually occurring 10-20 days postTrauma, ranges from 3 to 2 years
Also occurs in eczema, lichen planus, vitiligo and lichen sclerosus et atrophicus
What is the pathology?
Hyperproliferation of the epidermis with inflammation of the dermis and epidermis.
What are the differential diagnoses for onycholysis?
Psoriasis
Fungal infection
Thyrotoxicosis (Plummers nails)
Lichen Planus
What are the aggravating factors?
Emotional stress
Alcohol
Drugs beta blockers, ACE inhibitors, Indocid, Lithium & antimalarials
Streptococcal infection (classically associated with guttate psoriasis)
Injury to the skin mechanical injury and sunburn
What are the principles of management?
Education
Avoidance of aggravating factors
Topical WSP, salicyclic, aqueous cream
Topical Topical steroids, coal tar, Dithranol, Calcipotriol (Vit D3 which acts to increase keratinocytes differentiation
as a result of increased extracellular calcium therefore decreased cellular proliferation and scaling), topical retinoids
Systemic UVB, MTX, Retinoids, systemic steroids, cyclo, tacrolimus and MMF
Novel immunodulators (infliximab, etarnacept)
What is the prognosis?
Deforming and erosive in 40%
10% are disabled by the arthropathy
What other joint pathology can patients have especially if disease is active?
Gout because of hyperproliferation
Others
30% have family history
Psoriasiform lesions on the fingers, toes, nose and ears exclude SCC of the Oropharynx, tracheobronchial tree and
esophagus Bazex syndrome.

182

84. Lichen Planus


Presentation
Sir, this patient has Lichen planus as evidenced by
Cs
Grouped and confluent
Flat-topped, polygonal, sharply defined papules
Violaceous
Wickhams striae
Koebnerisation
Pruritic scratch marks
Post-lesional hyperpigmented macules
Distribution
Flexor areas of the ULs wrists and forearms
Sacral area, groin
Palms and soles
Nails
Dystrophy of nail plates
Longitudinal ridging
Longitudinal melanonychia
Subungal hyperkeratosis
Onycholysis
Pterygium
Complete loss of nail bed
Scalp
Scarring/cicatrical alopecia
Mucosal
White lacy-like lesions, asymptomatic
Ulcers for erosive form
There are no surgical scars noted and patient is not jaundiced with no stigmata of chronic liver disease which may suggest
Hepatitic C infection
I would like to complete the examination
Drug history
Antihypertensive B-blockers, thiazides, methyldopa
Antimalarials quinine
Anti-diabetic (Tolbutamide)
Phenothiazines, Gold
Occupational history
Color film developer
In summary this patient has lichen planus affecting his upper limbs, nails and oral mucosa and is troubled by pruiritus.

183

Questions
What are the types of Lichen Planus?
Hypertrophic (plague-like lesions on the tibia; Afro-carribean)
Erosive (mouth ulcers with risk of SCC)
Bullous
Follicular
Guttate
How would you Ix?
Skin biopsy with IF
Dx is triad of
Typical skin lesions
T-cell infiltration of the dermis in a band pattern
IgG and C3 immunofluorescence at the basement membrane of the dermis
How would you manage?
Education
Px skin lesions are not premalignant; oral ulcers can progress to SCC
Most resolves within 6-18 months
Pharmological
Steroids topical, intralesional and systemic
Cyclosporin topical for mouth lesions, systemic
MMF
Retinoids
PUVA
What is Pterygium of the nails?
Cuticle invades the nail bed; Cs of Lichen planus
What are the differential diagnoses for white lesions of the mouth?
Lichen Planus
Candidiasis
Secondary syphilis
Leukoplakia
Squamous papilloma
What are the differential diagnoses of oral ulcers?
Erosive Lichen planus
Pemphigus vulgaris
SJS
Drug eruptions
Infective HSV
Inflammatory bowel disease
Aphthous ulcers
Behcets disease
What are the differential diagnoses for violaceous lesions?
Lichen planus
Sarcoid (lupus pernio)
Dermatomyositis
(eruptive xanthomas of the skin may mimic the polygonal lesions of LP)

184

85. Neurofibromatosis (von Recklinghausens disease)


Examination
Spot diagnosis
Look at the arms for caf-au-lait spots, axilla for freckles
Look at the face
o Eyes Lisch nodules (brown pigmentation of the iris)
o Ears deafness
Lower limbs
o Bowed legs, pseudoarthrosis
Request
o Chest caf-au-lait spots, axillary freckling, kyphoscoliosis
o Fundi optic gliomas, retinal harmatomas
o Abdomen auscultate for renal bruit
o Pulses for coarctation
o BP hypertension in renal artery stenosis, coarctation, phaeochromocytoma
o Family Hx
Presentation
Sir, this patient has neurofibromatosis type 1 as evidenced by presence of multiple neurofibromas which are subcutaneous nodules
some of which are pedunculated with a generalised distribution associated with cafe-au-lat spots which are brown macules present
on the upper limbs and chest as well as axillary freckling.
On examination of his eyes, there are Lisch nodules detected and no deafness on screening (acoustic neuroma). Examination of
the lower limb does not reveal any bowed legs or pseudoarthrosis.
I would like to complete the examination by:
o Chest caf-au-lait spots, axillary freckling, kyphoscoliosis, lung fibrosis
o Abdominal examination
o Pulses
o Fundi optic gliomas, retinl harmatomas
o Cranial nerve examination V, VI, VII, VIII, Cerebellar
o BP hypertension in renal artery stenosis, coarctation, phaeochromocytoma
o Family Hx
Questions
What are the type of neurofibromatosis?
Type 1 and 2
What are the diagnostic criteria?
Type 1 - 2 or more of:
Caf-au-lait spots (6 or more, each >15mm in diameter)
Neurofibroma (2 or more) or plexiform
Freckles in the axillae or inguinal (Crowes sign)
Bone lesions Sphenoid dysplasia
Lisch nodules
FHx 1 or more first degree relative
Type 2 Either
1. Bilateral eight nerve palsy on CT/MRI OR
2. First degree relative with type 2 and either
a. unilateral eighth nerve mass or
b. 2 or more: neurofibroma, glioma, schawnomma, meningioma or
juvenile posterior subcapsular lenticular opacity.
What are Lisch Nodules?
These are melanocytic harmatomas of the iris that appear as well-defined, dome-shaped elevations projecting from the surface of
the iris which are yellow or brown in color. Incidence increases with age and by 20 years, 100% have them.
Why are the possible causes of hypertension?
Renal artery stenosis
Pheachromocytoma
Coarctation of the aorta
What are the histology of the skin tumors?
Schwanommas
Neurofibromas

185

What is the mode of inheritance and on which chromosome?


Autosomal dominant
Type 1 Chromosome 17 (Neurofibromin which is a tumor supp gene)
Type 2 Chromosome 22 (tumor supp gene)
If a parent is affected, child has 50% of being affected
If family has history of type 2, others should be screen with hearing tests and
brainstem auditory evoked response.
How would you investigate? Diagnosis is clinical.
How would you manage?
Education and counselling
Most do not require treatment
Managing complications hypertension, excision of painful neurofibroma,
radiation of optic glioma.
What are the associations and complications of the disease?
1. Childhood leukaemia
2. CNS complications
a. Mental retardation
b. Epilepsy
c. Obstructive hydrocephalus sec to stenosis of aqueduct of Sylvius
d. Optic gliomas, acoustic neuromas, meningiomas
e. Cord compression form spinal nerve root involvement
3. Sarcomatous changes
4. Bony complications
a. Intraosseous bone cysts
b. Bowed legs secondary to thinning of the cortices of long bones
c. Pseudoarthrosis of the tibia
d. Rib notching
e. Sphenoidal dysplasia
5. Lung cysts
6. Hypertension
a. Coarctation
b. Renal artery stenosis
c. Phaeochromcytoma (5%)
What can be a possible differential diagnosis?
Dercums disease.
What are the other neurocutaneous conditions that you are aware of?
Tuberous sclerosis (spot diagnosis) (= Bournevilles or Pringles disease)
o
Hx of epilepsy or seizures, FHx Aut dominant
o
Face adenoma sebaceum (angiofibromas distributed in a butterfly pattern over the cheeks, chin and forehead (dy/dx
acne, Cushingoid?)
o
Chest and back
Shagreen patches (leathery thickenings localised patches over the lumbosacral region)
Ash-leaf hypopigmentation
Caf-au-lait macules
o
Hands subungal fibromata
o
Systemic
CVS CCF and arrythmias, cardiac rhabdomyomas
Resp fibrosis
Abdomen polycystic kidneys, renal angiomyolipomas
CNS retinal harmatomas
o
Mx
Education
Rx seizures
Sturge-weber syndrome
o
Spot Dx
o
Hx seizures, hemiparesis, hemisensory, mentally retarded
o
Signs
Port wine stains in V1 and V2 distribution
Hypertrophy of area involved
Hemangiomas of the iris
Fundus for choroidal haemangiomas
BP for hypertension secondary to phaeochromocytoma
o
Ix SXR tramline calcification parietal-occipital lobe
o
Mx
Seizures control
Skin photothermolysis
Eye screen for glaucoma and Mx choroidal angiomas
Von-Hippel-Lindau disease

186

86. Purpura
(Approach: Establish purpura, rule out anaemia and neutropenia, establish cause)
Examination
Introduce, thank pt, ask for pain and request to undress, note any nasal speech (Wegeners)
General inspection
Age
Cushingoid
Renal failure, CLD
Extent ULs, LLs trunk
Examine individual lesions in the ULs or LLs
Palpable = vasculitis
Central hemorrhagic necrosis of HSP
Petechiae, ecchymosis
Cockscrew hair, perifollicular haemorrhages
Thin skin
Upper limbs
Hands
RA/SLE/Scleroderma
IE signs Oslers nodes, splinters, clubbing
Nails involvement
CLD stigmata
Elbows
RA nodules, thickened nerves (leprosy)
Face
Jaundice
Conjunctival pallor (haematological disease)
Malar rash
Mouth Ulcers, rashes, bleeding gums (scurvy for elderly patient)
Chest
CLD stigmata
Lower limbs
Arthritis of knees and ankles
Examine the feet
Requests
LNs
Abdominal examination hepatosplenomegaly
Peripheral neuropathy
Temperature chart
Urine dipstik hematuria in vascultis with renal involvement
Drug history
Presentation
Sir, this patient has
Purpura/palpable purpura as evidenced by non blanchable, well-demarcated reddish/purplish patches
Presence/absence of petechiae, ecchymosis
Distribution and extent
Anaemia and mouth ulcers (neutropenia)
Cause (purpura)
Age (Mention perifollicular haemorrhages and cockscrew hair if elderly)
Cushingoid
renal failure
Liver failure
Chest scars anticoagulation
Obvious haemarthrosis (haemophilia)
Ehlers Danlos
Cause (palpable purpura)
AI conditions
Infections
Malignancy
Drugs
Treatment (Cushingoid can be cause of purpura or treatment for vasculitic rash)
Complete examination for spleen, liver and LNs

187

Questions
What are your differential diagnoses for purpura?
Thrombocytopenia
o ITP
o BM infiltration haematological malignancies
o BM aplasia
o CLD
Capillary fragility
o Senile purpura
o Chronic steroid ingestion
o Vasculitis eg HSP
o Renal failure
Coagulation factors (ecchymoses)
o Haemophilia
o Christmas disease
o Anticoagulation
o CLD
What are the causes of a palpable purpura/vasculitis rash?
Autoimmune
o SLE/RA/SSc
o Churg-Strauss/PAN/Wegeners
o PBC/UC/Cryoglobulinaemia
Mitotic
o Solid
o Haematological Lymphoproliferative, paraproteinaemia
Infective
o Viral HIV, Hep B, Hep C, EBV, influenzae
o Bacterial IE, TB, leprosy, Streptococcal
Drug
o Aspirin
o Antibiotics (penicillins, sulphonamide)
o Allopurinol
o Anticoagulant
o Phenytoin, Gold
Idiopathic
What are the common causes of purpura?
Senile purpura
Secondary to steroids and anticoagulants
Thrombocytopenia from leukaemia or marrow aplasia
How would you investigate this patient?
After taking a detailed drug history
Blood Ix FBC, biochemistry, liver function test, coagulation profile and protein electrophoresis (rule out paraproteinaemia),
ANAs, dsDNA, ANCAs
Skin Bx small vessel vasculitis
Others Bone marrow biopsy, trephine biopsy of the iliac crest

188

87. Dermatomyositis
Examination
Examine this patients skin/face/hands
Look and proceed
This patient has dysphagia, please examine her
(Similar to sun-exposed rash)
Face
Helitrope rash
Neck and shoulder shawl sign
Weakness of neck flexion
Conjunctival pallor (associated with myeloproliferative or GI malignancies)
SLE or SScl for overlap syndrome
Hands
Grottrons sign
Vasculitis, capillary loops at the base of fingernails
Raynauds phenomenon
Calcinosis (usually in children)
SLE or SScl or RA for overlap syndrome
Upper limbs
Elbows for rashes
Tenderness of muscles
Test power, demonstrating proximal weakness
Loss of reflexes
Show no loss of sensory
Knees for rash
Request to screen for underlying mitotic lesions such as breast, respiratory and abdominal examination and screen for
interstitial fibrosis.
Presentation
Sir, this patient has got dermatomyositis.
There is the presence of heliotrope rash, which is a purplish-blue rash, around the eyelids and periorbital area and on the dorsum
of the hands. This erythematous rash is also present on the neck and the shoulders, ie in a shawl distribution as well as on the
sun-exposed areas. There is also involvement of the extensor surfaces of the elbow and knees. There is also periorbital edema.
Examination of the hands reveals also presence of Grottrons papules, which are flat-topped, violaceous papules over the dorsum
of the knuckles and interphalangeal joints. The erythematous rash spares the phalanges. There is presence of nailfold vasculitis
and telengiectasias. The cuticles are irregular, thickened and distorted. There is hyperkeratosis of the palms which resembles a
mechanics hands. I did not notice any Raynauds phenemenon. There is also no calcinosis.
There is tenderness of the muscles with proximal weakness. There is no sensory loss. There is also weakness of neck flexion.
I did not detect any clinical features of Systemic sclerosis or systemic lupus erytthromatosis or rheumatoid arthritis to suggest an
overlap syndrome. I would like to complete the examination by screening for any associated underlying mitotic lesion.
There are also no features of chronic steroid use.

189

Questions
What is dermatomyositis?
It is an idiopathic inflammatory myopathy with characteristic cutaneous findings.
How do you diagnose DM/PM?
4 out of 5 criteria:
Progressive, proximal, symmetrical muscle weakness
Raised CK
Cs EMG findings
Cs findings on muscle Bx
Compatible dermatological findings
What are the types of dermatomyositis?
Dermatomyositis
Polymyositis
Amyopathic dermatomyositis (no muscle involvement, just skin features)
How do you classify?
5 Groups, Group 1 to 5 respectively
Idiopathic polymyositis
Idiopathic dermatomyositis
A/w neoplasia
Childhood a/w vasculitis
A/w collagen vascular disease
How would you investigate this patient?
Creatinine kinase levels raised and reflects disease activity
ANA levels, anti-Mi-2, anti-Jo1
EMG myopathic changes which are spontaneous fibrillations, salvos of repetitive potentials and short duration of polyphasic
potentials of low amplitudes
Muscle biopsy necrosis and phagocytosis of muscle fibres, with interstitial and perivascular infiltration of inflammatory cells.
Ba swallow for atonic dilated esophagus (if stem statement states patient has dysphagia)
Other Ix to rule out malignancy (breast, lungs and GIT, ovaries) and mixed CT disease
What is your differential diagnosis for myositis with raised CK levels?
Statin, chloroquine and colchicine
What are some disorders associated with myositis?
Drugs
Infectious Lymes disease, CMV
Eosinophilic myositis
Outline your management.
Educate and counselling
Treat underlying malignancy
General meausures
Skin sun avoidance and sunscreens
Muscle bedrest, PT and OT, ST and bed elevation if dysphagia
Medical treatment
Steroid treatment (prednisolone 1mg/kg/day)
IVIG, methotrexate or azathioprine
Calcium channel blockers eg diltiazem for calcinosis
What is the prognosis?
Depends on
Presence of underlying malignancy
Severity of myopathy
Presence of cardiopulmonary involvement

190

EYE!
88. Diabetic Retinopathy
Presentations (Can still use the older classification)
Sir, this patient has:
(a)Type and location
1.

2.
3.

4.

Background diabetic retinopathy affecting the inferior/superior temporal/nasal quadrants of the retina (usually seen in the
posterior pole, ie area between the superior and inferior temporal quadrants) as evidenced by
a. Microaneurysms
b. Dot, blot or flame shaped haemorrhages
c. Hard exudates
Preproliferative diabetic retinopathy affecting the inferior/superior temporal/nasal quadrants of the retina
a. Cotton-wool spots
b. Venous dilatations, beading, looping or segmentation
Proliferative retinopathy diabetic retinopathy
a. Neovascularisation
i. At the disc
ii. Affecting the inferior/superior nasal/temporal quadrants of the retina
Diabetic maculopathy
a. Circinate formatiuon of hard exudates at or near the macula
b. Macular edema (cannot be seen by direct ophthalmoscopy)

(b)Treatment for (3) and (4)


1.
2.
3.

Focal photocoagulation scars


Panretinal or scatter photocoagulation scars
Macular photocoagulation scars

(c)Complications (for proliferative)


1.
2.
3.

Vitreous haemorrhages
Fibrosis with traction retinal detachment
Optic atrophy (for all)

(d)Associations (mention if present)


1.
2.
3.
4.

Xanthelesma
Cataracts
Hypertensive changes
Robeosis irdis

191

Questions
What are microaneurysms?
They are well-defined red dots seen in the superficial retinal layers which represents outpouching of the retinal capillaries;
earliest sign of diabetic retinopathy
Can also be seen in
o Hypertensive retinopathy
o Collagen vascular disease
o Severe anaemia
o Dysproteinaemia
What are flame-shaped haemorrhages?
Superficial bleed shaped by nerve fibres into a fan shape which points towards the disc
What are dot and blot haemorrhages?
Formed as a result of rupture of microaneurysms with bleeding into the deep layer of the retina
How do you differentiate between dot haemorrhages and microaneurysms?
This is difficult and differences includes:
Microaneurysms are well defined and last for months to years whereas dot haemorrhages tend to be have an
irregular outline and disappears within a few days
Fluoroscein angiography of which microaneurysms are hyperfluoroscent whereas dot haemorrhages are
hypofluoroscent
What are hard exudates?
These are minute, yellow, well defined deposits of lipo-protein and lipid-laden macrophages
What are cotton-wool spots?
Build up of axoplasmic material due to interrupted flow caused by ischaemia from capillary occlusion in the retinal nerve
fibre layer
What are IRMAs?
It stands for intraretinal microvascular abnormalities. They are remodelled capillary beds without proliferative changes and
are collateral vessels that do not leak on fluoroscein angiography. Usually found on the borders of non-perfused retina
What is neovascularisation?
Formation of abnormal new vessels on the retinal surface and at the optic disc as a result of ischaemia
These are fragile and tend to bleed into the vitreous leading to vitreous haemorrhages and fibrous tissue formation with
resultant traction retinal detachment
What is clinically significant macular edema?
Thickening of the retina at or within 500 microns of the centre of the macula
Areas of thickening 1 disc area or larger, any part of which is within 1 disc diameter of the centre of the macula
Hard exudates at or within 500 microns of the center of the macula, if associated with thickening of the adjacent retina
What is the pathogenesis of diabetic retinopathy?
Earliest stages are characterised by increased vascular permeability, leading to fluid accumulation in the retina (seen by
leakage of fluoroscein dye into the vitreous humor)
Later there is vascular closure causing retinal ischaemia leading to neovascularisation of the retina
These new vessels are prone to complications of vitreous haemorrhages, fibro-proliferative changes, retinal detachment
and neovascular glaucoma
How can diabetes mellitus affect the eye?
Eyelids xanthelasma (association)
Extraocular mononeuritis multiplex, diabetic third (spares the pupils and associated with headache; resolves within 3
months) or sixth nerve palsies
Anterior chamber neovascular/rubeotic glaucoma
Iris rubeosis irdis
Pupils Argyll Robertson pupil, RAPD
Lens cataracts(higher incidence and occurs at a younger age), refractor errors (occurs due to fluctuation in the blood
sugar level especially when starting treatment; it is a benign condition)
Vitreous body - haemorrhages
Retina DM eye changes, hypertensive, CRAO, lipaemia retinalis
Optic nerve optic atrophy, ischaemic papillitis
Orbit mucormycosis

192

How can patients present clinically?


NPDR asymptomatic
PDR
o Asymptomatic
o Reduced VA or blindness as a result of complications
CSME
o Asymptomatic
o Reduced in VA
o Paracentral scotoma
o Decrease in central vision
What conditions can cause blindness in diabetic eye disease?
Macular edema
Retinal detachment
Vitreous haemorrhages
3.6% of type 1 and 1.6% of type 2 were legally blind according to the WESDR
When should we screen for diabetic eye disease?
Type 1 DM within 3-5 years of Dx of DM
Type 2 DM at diagnosis
Pregnancy with pre-existing DM prior conception and first trimester
How should we screen for diabetic eye disease?
Fundal photography
Indirect ophthalmoscopy with slit-lamp biomicroscopy
Direct ophthalmoscopy through dilated pupils
How frequent should patients be followed up?
No retinoapthy annually
NPDR
o Mild and no retinal edema 6 to 12 monthly
o Presence of retinal edema 4 to 6 monthly
o Macula affected or severe 1-4 monthly
PDR 1 to 4 monthly
How soon must you refer a patient with diabetic eye disease to the ophthalomologist?
All patients with DM retinopathy needs a referral to an ophthalmologist
Immediately (1 day)
o Sudden loss of vision
o Retinal detachment
Urgently (within 1 week)
o Neovasculariastion
o Pre-retinal or vitreous haemorrhages
o Rubeosis irdis
Soon (within 4 weeks)
o Pre-proliferative changes
o Macular diabetic changes
o Unexplained drop in VA
What are the risk factors for diabetic eye disease?
Poor glucose control
o WESDR (Wisconsin Epidemiologic study of DR)
o DCCT (Diabetes Control and Complications trial)
Hypertension
o UKPDS (UK prospective diabetic study)
o Aim < 130/80 mmHg
Hyperlidaemia
Renal disease aggressive treatment of renal disease may slow progression of DM retinopathy and prevent neovascular
glaucoma
Cigarette smoking
Duration of diabetes (non-modifiable)
o After 20yrs, nearly all patients with type 1 and 60% of type 2 will have DR
Pregnancy (non-modifiable)

193

How do you manage patients with diabetic eye disease?


Management of the diabetic eye includes:
o Prompt referral to the ophthalomologist (see above)
o Macular edema focal or grid macular laser
o NPDR none or consider scatter laser if severe
o PDR scatter laser
o Vitreous surgery
Non-resorbing vitreous opacities
Traction retinal detachment threatening or involving the macula
Progressive fibro-proliferative diabetic retinopathy
Combined rhegmatogenous and traction retinal detachment
Pay attention to
o Glycaemic control
o Blood pressure
o Quit smoking
o Screen for other DM complications especially DM renal disease
o Control hyperlidaemia
Engage patient
o Education
o Importance of regular follow up
How would you manage a patient who requires laser therapy but has cataracts?
If fundal visibility permits, laser treatment administered prior to cataract surgery
If not, cataract surgery followed by prompt laser treatment
How effective is laser photocoagulation?
For visual salvage in maculopathy
o Effective in 50-60% of cases
For reduction of already formed abnormal new vessels on the retina
o It can abolish new vessels in up to 80% of patients with PDR and follow up showed that disease had stabilised or
cured
o In pan-retinal photocoagulation, it reduces the ischaemic and hypoxic retina, reducing angiogenic factors and
neovascularisation
o It can alsobe used to treat microaneurysms
There may be loss of peripheral vision

194

89. Hypertensive Retinopathy


Presentation
Sir, this patient has got grade 4 hypertensive retinopathy as evidenced by
Presence of narrowed, tortous and irregular retinal arteries with increased light reflex, demonstrating the silver/copper wiring
There is arterio-venous nipping
With evidence of flame shaped and blot haemorrhages and cotton wool spots and hard exudates, especially around the
macula, forming a macular star
Associated with papilloedema
I did not notice any concomitant features of diabetic retinopathy. However the patient does have xanthelesma.
There are no signs of chronic renal failure such as sallow appearance. Patient does not have any cushingoid or acromegalic or
polycythaemic features which are conditions associated with hypertension.
I would like to complete my examination by taking the blood pressure of this patient as well as examine his
Cardiovascular system
RR/RF delay coarctation of the aorta (state if young)
Evidenced of LVH
S4 if BP>180/110 (state if grade 3 or 4 changes noted)
Abdominal examination for renal bruit (renal artery stenosis) or ballotable kidneys (polycystic kidney disease) or
palpable adrenal masses
Urine dipstick for proteinuria, casts, glycosuria
CNS for signs of previous CVA

195

Questions
How do you grade hypertensive retinopathy?
Keith Wagner Classification
Grade 1 Arteriolar narrowing, tortousity, irregular calibre with copper/silver wiring
Grade 2 Arteriovenous nipping
Grade 3 flame-shaped and blot haemorrhages, cotton wool spots and hard exudates
Grade 4 Papilloedema
Clinical features and prognosis of grade 3 and 4 are the same
Explain their physical appearance?
Young patients, retinal arterioles react to hypertension via constriction, hence arteriolar constriction or narrowing
In older patients, there is arteriosclerosis hence irregular calibre. Also the thickened walls shows a widening of the normal light
reflex, giving the blood column a copper appearance (copper wiring) or silver appearance (silver wiring)
At arteriovenous crossing, the thickened arteriolar walls displace and constricts the veins, resulting in AV nipping
The arteriolar may be damaged by necrosis leading to flame shaped haemorrhages, cotton wool spots caused by
microinfarcts, as well as retinal edema
Chronic retinal edema at the macula results in hard exudates radiating from the macular, ie macular star
Finally papilloedema results
How do patients normally present?
Normal vision
Except when there is associated macular involvement
What are the causes for retinal haemorrhages?
Diabetic retinopathy
Hypertensive retinopathy
CRVO
Severe anaemia, leukaemia
What are the causes for cotton wool spots?
DM preproliferative retinopathy
Hypertensive retinopathy
Anaemias, leukaemias
HIV, infective endocarditis
What are the causes for hard exudates?
Diabetic retinopathy
Hypertensive retinopathy
What are the causes of hypertension?
90% are essential hypertension
10% are secondary
o Renal causes Chronic renal disease, polycystic kidneys, renal artery stenosis
o Endocrine Cushings, Acromegaly, Phaechromocytoma, Conns, hyperparathyroidism
o Others Coaractation, OCP usage, pre-eclampsia, polycythaemia
How would you investigate a patient with hypertension?
Urea and electrolytes
Urine for protein, glucose and cast
Fasting lipids and glucose
ECG, CXR
If indicated clinically, ie
Young hypertensive <50
Requiring >2 antihypertensive
Sudden deterioration in control of BP
Features suggestive of secondary causes on clinical examination
How would you manage?
Lifestyle- Exercise, eat healthily, stop smoking
Pharmocotherapy

196

90. Optic atrophy


Examination
Fundus
o Papilloedema
o Optic cup (Glaucoma)
o DM changes
o Retinitis pigmentosa
o Attenuated arterioles and veins as in CRAO
Eye
o RAPD (MS)
o Argyll-Robertson pupil
o INO (MS)
o Nystagmus (MS and FA)
Head
o Tender temporal arteritis
o Pb lines in the gums
o Pagets facies
Hands
o Cerebellar
o AF
Presentation
Sir, this patient has unilateral/bilateral optic atrophy. On examination of his fundus, I did not detect any presence of papilloedema,
deep optic cup, DM changes or RP. The vessels are also not attenuated.
I would like to complete the examination by testing patients visual acuity and visual fields as well as for features of MS with RAPD,
INO, nystagmus/cerebellar signs, palpate the temporal artery and examine the gums for Pb lines and pulse for AF

197

Questions
What are your differential diagnoses?
Unilateral
o Demyelinating disease
o Compression (Tumor, aneurysm, Pagets)
o Glaucoma
o Ischaemic
Thromboembolic
Vasculitis temporal arteritis, tertiary syphilis
Bilateral
o Toxic
Nicotine, alcohol
Drugs (ethambutol, chloroquine, methanol, Pb, Arsenic)
o Metabolic
B12 deficiency, B1 and B6
Diabetes mellitus
o Hereditary
FA
Lebers (mitochondrial dz with pt mutations)
DIDMOAD (DI, DM, Optic atrophy, Deafness) rare recessive
o Others
sec to papilloedema
sec to retinitis pigmentosa
How do patients present?
o Reduction in visual acuity
What would you find on visual filed testing?
o Central scotoma
What is Lebers optic atrophy or Lebers hereditary optic neuropathy (LHON)?
o Hereditary condition affecting males
o Progressive visual loss
o Onset form second decade onwards
o Mitochondrial disease with point mutations
What investigations would you do?
o Electroretinography and pattern evoked visual response
o Blood test
Blood glucose
ESR
VDRL
B12 levels
o Imaging
SXR enlarged sella turcica pit tumor
CT scan SOL
MRI demyelinating disease
o (History for toxic and hereditary causes)

198

91. Papilloedema
Examination
On noticing papilloedema
Attempt to identify the different stages of papilloedema present
Increase in venous calibre and tortusity
Optic cup pinker with disappearance of vessels over the disc
Disc is suffused and slightly elevated with blurring of margins; optic cup is filled and presence of haemorrhages around the
disc
Look at the retina
Features of hypertension (flame-shaped haemorrhages, cotton wool spots and hard exudates)
Features of CRVO (heamorrhages)
Severe anaemia (haemorrhages)
Check for
Pallor (severe anaemia)
Obvious proptosis
Graves ophthalmopathy
Cavernous sinus thrombosis
Spectacles (for hypermetropia)
Requests to examine the other eye if told to examine one eye only (bilateral papilloedema vs Foster-Kennedy syndrome)
Requests
VA
Visual fields
Color testing
Pupillary reflex (may not be possible if dilated)
Eye movements
Pain on eye movements
VI nerve palsy
Palpate the temporal region if elderly for tenderness (temporal arteritis)
Blood pressure
Presentation
Sir, this patient has papilloedema affecting his right eye as evidenced by a suffused and slightly elevated optic disc associated with
blurring of the disc margins with filling in of the optic cup and dilated tortuos veins.
There was no evidenced of hypertensive retinopathy such as silverwiring of the blood vessels, arterio-venous nipping, flamedshaped haemorrhages or exudates. I could not detect any haemorrhages on the retina to suggest severe aneamia or CRVO.
I noticed that there was no conjunctival pallor and no obvious proptosis of the eye.
I would like to complete my examination by examining the other eye for features of papilloedema or optic atrophy; checking his
blood pressure; testing his VA and VF and asking him about color vision loss; eye movements for VI nerve palsy and pain on eye
movement as well as RAPD.
Questions
What are the differential diagnoses of optic nerve swelling?
Papilloedema
Papillitis
Ischaemic optic neuropathy
Pseudopapilloedema
Hypermetropia (margins is blurred)
Drusen (yellowish-white deposits at the optic disc)
Myelinated nerve fibres
Bergmeisters papilla (whitish elevation of the center of disc; common, seen in all ages, races and equal sex distribution)
How do you differentiate beteween papilloedema and pseudopilloedema?
Fundal fluroscein angiography
What are the differences between papilloedema and papillitis?
Papilloedema
Papillitis
VA
Preserved
Reduced
VF
Enlargement of blind spot; loss of
Central scotoma
peripheral vision
Color testing
Normal
Loss of red
Pupillary reflex
Not affected
RAPD
Eye movements
No pain on movements
Pain on movement
Others
Bilateral
Unilateral
Absent of venous pulsation
Venous pulsations present
*Retrobulbar neuritis presents exactly like papillitis without the optic nerve head swelling appearance

199

What are the causes of papilloedema? (6)


Space Occupying lesion
Malignancy SOL tumor
Abscess
Hematoma
Hydrocephalus
Obstructive (ventricle, aqueduct, outlet to 4th ventricle)
Communicative
Increased CSF formation (rare Choriod plexus papilloma)
Reduced CSF absorption
Meningitis
SAH
Benign intracranial hypertension
Idiopathic
Addisons disease
Drugs Nitrofurantoin, tetracycline, vit A, steroids, OCPs
Hypertension
CRVO
Others(3s)
Metabolic
Hypoparathyroidsm
CO2 retention
Graves congestive ophthalomopathy
Hamotological
Severe anaemia
Polycythaemia rubra vera/leukaemia/Multiple myeloma
Sagittal/caverbous sinus thrombosis
Sarcoid
GBS (impaired CSF absorption due to elevated protein content)
Pagets disease, Hurlers syndrome
What are the stages of papilloedema?
Stage 1 increase in venous size and tortousity with loss of venous pulsation
Stage 2 Optic cup is pinker and less distinct with disappearance of vessels over the disc
Stage 3 Blurring of the disc on the nasal side
Stage 4 Disc is suffused and elevated with blurred margins, optic cup filled in and haemorrhages around the disc
What is ischaemic optic neuropathy?
Ischaemia to optic nerve head from arteriolar sclerosis or temporal arteritis
Elderly with sudden visual loss
Swollen optic disc
What is Foster-Kennedy Syndrome?
Characterised by papilloedema in one eye and optic atrophy in the other eye
Caused by a tumor of the frontal lobe, classically subfrontal olfactory groove meningioma
Results in optic atrophy due to compression of that optic nerve by the tumor and papilloedema in the contralateral eye from raised ICP
Look for features of NPH and loss of smell
What is Benign intracranial hypertension?
Dandys diagnostic criteria
Patient is alert
Clinical features of raised ICP
No localising signs except VI CN palsy
LP opening pressure >20cm H2O and CSF composition normal
Normal ventricle size and normal CT head
Management
Discontinue drugs, weight loss
Diuretics, carbonic anhydrase inhibitors
Serial LPs or lumbopertoneal shunt
Optic nerve fenestration or subtemporal decompression

200

92. Central and Branch Retinal Vein Occlusion


Presentation
Central Retinal Vein Occlusion
Sir, this patient has got central retinal vein occlusion affecting the right eye as evidenced by grossly tortuous and engorged retinal
veins, especially near the disc. There are also numerous haemorrhages seen scattered all over the retina of varying shape and
sizes with presence of cotton wool spots (blood and thunder appearance). There is also papilloedema.
There is no evidence of rubeosis irdis.
The left eye is unaffected but has presence of hypertensive retinopathy grade 2 with silver wiring and irregular retinal arterioles
associated with arteriovenous nipping. (Look for causes for in the other eye eg hypertensive or diabetic changes)
I would like to complete my examination by
VA and Visual fields
Checking her BP
Urine dipstick for glycosuria
Urine for Bence Jones protein for multiple myeloma
Look for LNs and hepatosplenomegaly, bruising and purpura for macroglobulinaemia
Ask about h/o of glaucoma or use of OCPs (in females of reproductive age group)
Branch Retinal Vein Occlusion
Sir, this patient has a branch retinal vein occlusion affecting the superior/inferior temporal/nasal vein in his right eye as evidenced
by a fan-shaped distribution of retinal flame-shaped haemorrhages with cotton wool spots which radiate from the arteriovenous
crossings. These changes have affected the macula of the right eye.
There is presence of hypertensive changes of grade 2 with silver wiring and irregular retinal arterioles with arteriovenous nipping.
There is no exudates noted and the disc margins are sharp.
I would like to complete my examination by
Checking his VA (normal unless affecting the macula) and VF (nomal or presence of a quandrantonopic fieled loss)
Blood pressure
Urine dipstick for glycosuria
Questions
What are the vessels that are involved in retinal vein occlusion?
CRVO central retinal vein (occlusion is behind the cribiform plate)
BRVO in front on the cribiform plate
o
Most commonly the superior temporal retinal vein
o
Followed by inferior temporal retinal veins
o
The nasal retinal veins
How do patients present?
CRVO
Diminished visual acuity to counting fingers (incomplete loss)
Reduced visual fields
Floaters
Acute secondary glaucoma 3 months later from rubeosis irdis from an ischaemic retina
Although retinal changes may improve in weeks, VA fails to be restored
BRVO
VA normal unless macula affected
Can have sudden visual loss if there is vitreous haemorrhage
Normal VF or monocular quadrantic field defect
Floaters
How do you differentiate between CRVO and papilloedema?
CRVO has reduced VA whereas papilloedema affected only in late stages
CRVO unilateral involvement in contrast to papilloedema
What are the usual sites of occlusions in BRVO
AV crossings
Along the main veins (DM)
Edges of the optic disc, resulting in hemispheric involvement
Peripherally as in sickle cell disease

201

What are the causes of retinal vein occlusion?


CRVO
o
Hypertension
o
Diabetes mellitus
o
Glaucoma
o
Hyperviscosity syndrome
Waldenstroms macroglobulinaemia
Multiple myeloma
OCPs
BRVO
o
Diabetes mellitus
o
Hypertension
o
Need to differentiate with retinal vasculitis
How would you investigate?
CRVO
BP
Fasting blood glucose
Screen for glaucoma
Screen for multiple myeloma (BJ proteins) and macroglobulinaemia
Enquire regarding OCPs and stop with alternate use of contraceptive methods
BRVO
o
BP
o
Fasting blood glucose, urine dipstick
o
ESR, CRP and ANA and ANCA for vasculitis
How would you manage this patient?
CRVO
o
Identify causes and risk factors and correct or treat them
o
Fluoroscein angiography to identify areas of retinal ischaemia
o
Panretinal photocoagulation to prevent rubeotic glaucoma
BRVO
o
Identify and treat causes and risk factors
o
Fluoroscein angiography to identify areas of retinal ischaemia
o
Laser photocoagulation may be necessary to ablate damaged vessels if there is macular edema or risk of vitreous
haemorrhages from abnormal new vessels
What is the prognosis?
CRVO
o
Mild to severe
o
Mild cases, asymptomatic
o
In severe cases, markedly reduced VA to counting fingers with complications of neovascularisation of the retina,
rubeosis iridis(1 month) and robeotic glaucoma (3 months)
BRVO
o
Mostly asymptomatic
o
Macula affected with VA loss but can recover
o
Occassionally leads to gradual visual loss

202

93. Central Retinal Artery Occlusion


Presentation
Sir, this patient has a right sided CRAO on fundoscopy as evidenced by
RAPD
Pale retina
Foveola cherry red spot
Attenuated retinal vessels
Intra-arterial emboli (10-20%)
I did not notice any laser scar marks to suggest laser panretinal photocoagulation and there are no complications of rubeosis iridis
Complete the examination
Visual acuity
Pulse (AF), DM dermopathy, xanthelasma, BP
Palpate the temporal area for tenderness in GCA
Questions
What is the epidemiology of CRAO?
1 in 10 000
Male 2X
Unilateral in 99%
How do patients present?
Sudden, painless visual loss (counting fingers to light perception)
10% has a history of amaurosis fugax TIA affecting the retinal artery and lasting for minutes and classically described as a
loss of vision in a curtain descending fashion
Significant cardiovascular risk factors
What are the causes?
Emboli (Hollenhorst plague = cholesterol embolus within the arteriole)
Arteritis (GCA, SLE, PAN)
Why is there a Cherry red spot?
Ischaemia of the retina at the posterior pole renders the retina pale, white and milky; thus the choroid is seen through the fovea
as a cherry red spot
How would you manage?
No effective treatment
o Within 100mins, severe retinal cell injury
o Total irreversible damage occurs 4 hrs after onset
Within 48 hrs, may attempt
o Digital massage for at least 15 mins to dislodge any emboli
o Administration of carbogen therapy (95% O2 and 5% CO2)
o Medications to lower intraocular pressure (IV acetazolamide)
o Anterior chamber paracentesis
Laser panretinal photocoagulation to reduce risk of progression to neovascular glaucoma
Manage cardiovascular risk factors
o DM, hypt, lipids, smoking
o Echo and USS carotids
o Antiplatelets
What are the differential diagnoses for sudden painless loss of vision?
CRAO
CRVO
Retinal detachment
Submacular haemorrhage form age-related macular degeneration
Vitreous haemorrhage commonly from DM retinopathy

203

94. Retinitis Pigmentosa


Examination
Upon noting the pigmented bony spicules at the eye peripheries
Macular edema or bulls-eye maculopathy
Attenuated arterioles
Way pallor of the optic disc
Cataracts
External ophthalmoplegia
Ptosis
Deafness
Hands for polydactyly
Sallow appearance for renal impairment
Short Stature
Presentation (think in terms of 3s)
Sir, this patient has retinitis pigmentosa as there are bony spiculated pigmentation on the peripheries of the retina bilaterally.
This is not associated with any macular edema or bulls eye maculopathy, attenuated arterioles or a waxy pale optic disc.
I also noted that the patient has cataracts, which maybe associated with retinitis pigmentosa. There is no ptosis that I noted and
the patient is not sallow in appearance and does not have short stature.
I would like to complete the examination by examining his visual field and acuity, enquire about night blindness and assess eye
movements to look for external ophthalmoplegia. I would also liked to screen for deafness as well as examine his hands for
polydactyly and his limbs for spinocerebellar degeneration.
Questions
What is retinitis pigmentosa?
It is a dystrophy of the photoreceptors and pigment epithelium with an incidence of 1 in 4000. Occurs bilaterally and begins in childhood or
young adults with a progressive course.
What is the mode of inheritance?
9% X linked, 16% is AR and 22% is AD; the rest or approx 50% is sporadic.
What are the clinical features of RP?
Commoner rod-cone type
o
Ring scotoma,
o
Peripheral visual field loss, tunnel vision
o
Night blindness
Cone-rod type
o
Visual acuity loss
o
Loss of color discrimination
o
Day vision problems
What are the causes of RP?
Primary where no known cause
Secondary to inflammatory retinitis
Associated with other syndromes
o
Ushers syndrome (RP with hearing loss)
o
Alports syndrome (RP with hearing loss and nephritis)
o
Refsums disease (RP with deafness, hypertrophic peripheral neuropathy and cerbellar ataxia- it is a phytanic acid storage
disease)
o
Abetalipoproteinaemia (RP, with fat malabsorption and spinocerebellar degeneration)
o
Friederichs ataxia
o
Kearnes Sayre Syndrome (RP with ext ophthalmoplegia, ptosis and heart block)
o
Laurence-Moon-Biedl Syndrome (RP with short stature, polydactyly, renal dysfn)
What are the associated ocular abnormalities? Posterior subcapsular cataracts, Myopia, Keratoconus, Open angle glaucoma
How would you Ix?
Careful FHx and rule out phenothiazine toxicity
Formal visual fields testing, color testing (Ishihara charts) and electroretinogram (ERG)
Ix if cause is suspected, eg, ECG for heart block in Kearnes Sayre syndrome, lipids and protein elctrophoresis for
abetalipoproteinaemia, serum phytanic acid for Refsums disease
How would you manage?
Education and counselling especially genetic counselling
Impaired vision training and aids for ADLs and job retraining
Medication such as high dose of Vit A which slow the progress of RP by 2% a year and acetazolamide for complications of cystoid
macular edema.

204

95. Visual Field Defects


Examination
Stem statements
o Examine visual fields
o Examine eyes
o Patient complain of knocking into objects
General
o Acromegaly
o Hemiparesis
o Dysphasia
Visual fields
o Introduce
o Sit about an arms length
o Can you see my whole face
o Test for gross VA counting fingers (wear spectacles!)
o Test for gross visual fields using finger movements as well as for visual inattention
o Patient to cover his right eye with right hand and instructed to look straight into my left eye
o Test using white hat pin from all quadrants
o If single eye defect
Proceed with fundoscopy
BRAO, haemorrhages, chorioretinitis
Optic atrophy, glaucoma, RP
Possibilities
Constricted field
o Chronic papilloedema
o Chronic glaucoma
o Retinitis pigmentosa
o Chorioretinitis
o Hysteria (visual field does not widen as object is brought further away from the patient cf to
organic cause)
Scotoma (red hat pin)
o Retinal haemorrhage or infarct (paracentral or peripheral scotomas)
Does not cross the horizontal midline
o Optic nerve (pale in atrophy, normal in retrobulbar neuritis and pink and swollen in
papillitis) resulting in central scotomas
Compression tumor, aneurysm, Pagets
Glaucoma
Neuritis
MS
Ischaemic (C/BRAO, syphilis, temporal arteritis and idiopathic)
Toxic (methanol, tobacco, Pb, arsenic)
B12 defeiciency
Hereditary Friederichs ataxia, LHON
Secondary to retinitis pigmentosa
Altitudinal defects
o Retina infarcts
o Ischaemic optic neuropathy
Totally blind in one eye
o Retina
o Optic nerve
o If bilateral peripheral field loss
Bilateral retinal lesion
Bilateral optic nerve lesion
o If bitemporal defect
Upper> lower = inferior chaismal: Pituitary tumor, Suprasellar meningioma
Lower > upper: Craniopharyngioma
Other causes: Aneursym, Metastasis, glioma
o If homonymous hemianopia (infarcts, haemorrhages or tumor)
Left or right homonymous hemianopia = right or left lesion respectively
Incongruous: Optic tract
Congruous
Upper quandrantonopia: Temporal lobe
Lower quandrantonopia: Parietal lobe
Macula sparing (test with a red hat pin): Occipital cortex
No macula sparing: Optic radiation
Note any DM dermopathy, xanthelasma and AF, hemiparesis

205

Presentation
Sir this patient has
A left/right
o Eye blindness
Unable to perceive light
o Scotoma
o Constricted visual field defect
o Upper/lower, temporal/nasal field
Bitemporal hemianopia
o Mention any acromegalic features
o Request to screen for hypopituitarism
o Causes includes (see above)
o Investigate
Lateral SXR enlarged sella turcica, calcification for cranipharingioma
CT or MRI head
Formal field perimetry
Serum prolactin
Left/right, upper/lower homonymous quandrantonopia
Left or right homonymous hemianopia, incongruous or congruous, macula sparing
o Mention obvious signs
Hemiparesis
Dysphasia (for right homonymous hemianopia)
Visual inattention
o Request for neurological examination for CVA and tumor
o Look for
CVA risk factors DM dermopathy, xanthelasma, AF
Tumor
Cachexia, clubbing for metastatic disease
o Ix
CT head
Formal field testing, perimetry

206

Other Eye Conditions


96. Visual Acuity
Examine each eye with finger counting
o If unable to do so, proceed with finger movement and then light perception
o If able to do so, proceed with Snellen chart
Determine unilateral or bilateral, acute or chronic
Causes
o Bilateral Acute front (methyl alc poisoning) vs back( occipital lobe infarction trauma)
o Bilateral chronic glaucoma, cataracts, DM, bilateral nerve damage or compression)
o Unilateral acute
CRVO, CRAO, arteritis, non arteritic isch optic neuritis
Retinal detachment
Vitreous hemorrhage
97. Cataracts
Causes
o Systemic
Senile cataracts
DM
In prroly controlled younf type 1 DM, can get snowflakes cataracts
Hypoparathyroidism
Drugs
Steroids (>10mg/day of prednisolone > 1year)
Chloroquine
Chlorpromazine
o Local
Trauma to the eye
Glaucoma
Radiation
o Hereditary
Dystrophia myotonica (stellate)
Wilsons diseae (sunflower cataracts)
Refsums disease
98. Nystagmus
Rule out nystagmus at extremes of gaze which is physiological
Obvious type of nystagmus
o Pendular congenital, macular disease
o Rotatory only central causes
o Upbeat nystagmus (fast phase upwards)
Upper brainstem MS, stroke, Wernickes ( triad of confusion, ophthalmoplegia and nystagmus, ataxia a/w
Korsakoffs Psy)
o Downbeat nystagmus
Cervicomedullary junction AC malformation, syringobulbia,MS
o Ocular bobbing pontine lesions
Jerky nystagmus
o Occurs at primary gaze (means central)
Cerebellar
Vestibular (MS or stroke)
o Occurs on horizontal gaze
Multidirectional gaze evoked nystagmus
Central cerebellar or vestibular
Right or left horizontal gaze evoked nystagmus
Central or
Peripheral
o Vestibular neuronitis, Menieres
o Ataxic nystagmus ie INO
NB: To differentiate between central and peripheral, central is sustained and peripheral can be fatigued and often associated with
severe vertigo

207

99. Pupillary defects


Large pupil
o Differential diagnoses
RAPD
III nerve palsy
Holmes Adie pupil
Unilateral
Slow reaction to bright light and incomplete constriction to convergence
Young women
Reduced or absent reflexes
Degeneration of ciliary ganglion
Mydriatic drugs
Sympathetic overdrive (drugs)
Small pupil
o Argyll Robertson pupil
Characteristic
Small (2mm), irregular pupils
Absent light reflex
Intact accommodation reflex
Does not dilate with mydiatrics
Sign of tertiary syphilis
Begins unilaterally and involves both pupils with time (months to years)
Pathophysiology unknown
Differential diagnoses for light-near dissociation
Syphilis
DM
Pituitary tumors
Midbrain lesions
Adies tonic pupil
Dystrophia myotonica
Aberrant regeneration of CN III
Familial amyloidosis
o Horners syndrome
o Long Standing Adies tonic pupil (initially large pupil)
o DM
o Encephailitis
o Sarcoidosis
o Lymes disease
o Parinauds (triad of psuedo AG pupil, vertical gaze palsy and nystagmus on convergence and causes include MS,
vascular and pinealoma)

208

100. History-taking Station


Steps
1.

5 mins - Read carefully and prepare headings as in (2)

2.

12 mins Examination itself


a. Introduce, shake hands
b. Are you comfortable? Positioning!
c. I understand from the GP letter that
d. Past history, drug allergy
e. Present complaints
i. Address individual problems
ii. Etiology and differential diagnoses
iii. Associations of condition
iv. Complications of disease and treatment
v. Conditions that may affect treatment
f. Women Menstrual history, pregnancy
g. Social history
i. Work job nature, boss, colleagues
ii. Family husband, children
iii. Others Church activities, exercise
h. Family History
i. Smoker, Alcoholic
j. ICE Ideas, Concerns and Expectations, Feelings

3.

2 mins Wrap Up
a. Any other significant things you would want to bring up
b. Summarise
c. Assure
d. Management plan
i. Investigations
ii. Symptomatic treatment
iii. Letter to employer
iv. Arrange next consult with family and significant others
v. Discuss the case with the consultant

4.

1 min Gather your thoughts


a. List patients problems
i. Medical ( Disease, associated conditions, side effects)
ii. Social
iii. Concerns
b. What is the diagnosis or differential diagnoses for the presenting complains?
c. Summarise

5.

5 mins Discussion of case with examiners


a. Answer the above (4)
b. Investigations
c. Management

209

You might also like